Principles of Biology Chapter 1, Principles of Biology Chapter 15, Principles of Biology Chapter 14, Principles of Biology Chapter 13, Principles of Biology Chapter 12, Principles of Biology Chapter 11, Principles of Biology Chapter 10, Principles of...

Pataasin ang iyong marka sa homework at exams ngayon gamit ang Quizwiz!

What are the 3 major principles of the cell theory?

1. A cell is the basic unit of life. 2. Organisms are made up of cells. 3. New cells arise only from pre-existing cells.

The site inside a eukaryotic cell where photosynthesis takes places is the... A.) Chloroplast. B.) Cytoplasm. C.) Nucleus. D.) Golgi complex.

Chloroplast.

Define and distinguish between: atomic number and atomic mass.

Atomic number- number of protons Atomic mass- number of protons added to number of neutrons

Which of the following molecules can ionize and release hydrogen ions to make the solution more acidic? A.) Carbonyl B.) Phosphate C.) Sulfhydryl D.) Carboxyl E.) Hydroxyl

Carbonyl

In a recessive disorder, individuals who have that recessive allele but not the disorder are referred to as.. A.) Autosomal B.) Codominant C.) Phenotypes D.) Genotypes E.) Carriers

Carriers

What is the function of a vector in genetic engineering? A.) Carry DNA into a new cell B.) Make millions of copies of specific segments of DNA C.) Separate fragments of DNA by their length and electrical charge D.) Cut DNA into many fragments E.) Link together newly joined fragments of DNA

Carry DNA into a new cell

The function of transfer RNA is to... A.) Deaminate amino acids. B.) Carry amino acids to ribosomes. C.) Transfer nucleotides to the nucleus. D.) Act as the site for protein synthesis. E.) Turn DNA on and off.

Carry amino acids to ribosomes.

The process of lowering the activation energy for a chemical reaction is called... A.) Respiration. B.) Catalysis. C.) Oxidation. D.) Photosynthesis.

Catalysis.

Living organisms are characterized by a hierarchy of increasing complexity, such as: atoms --> molecules --> macromolecules --> organelles --> cells --> tissues --> organs At which of these levels of complexity does the property of metabolism first emerge?

Cell

The basic unit of structure and function for all life is the...

Cell

What are 2 other names for the cell's plasma membrane?

Cell membrane and cytoplasmic membrane are two other names for the cell's plasma membrane.

Which of the following processes does not expend ATP (energy)? A.) Muscle contraction B.) Cell movement C.) Cell growth D.) Cell respiration

Cell respiration

At which level of taxonomy do humans and plants share common characteristics? A.) Order B.) Kingdom C.) Phylum D.) Domain E.) Class

Domain (Eukarya)

Which of the following is the main component of cell membranes? A.) Cellulose B.) Sucrose C.) Phospholipids D.) Triglycerides E.) Glycogen

Phospholipids

The role of enzymes in cellular metabolism is to... A.) Speed up the biochemical reactions in cells B.) Slow down the biochemical reactions in cells C.) Transport oxygen and carbon dioxide in cells D.) Protect the cell from osmotic pressure E.) Transport nutrients into cells via facilitated diffusion

Speed up the biochemical reactions in cells

Regarding the replication of DNA in cells, we know that a double helix... A.) Reforms exactly as it was after replicating. B.) Splits into two single strands, and each one then acts as a template to build its complement. C.) Fragments into small chunks that duplicate and reassemble. D.) Becomes a quadruple helix.

Splits into two single strands, and each one then acts as a template to build its complement.

During which phase of the cell cycle is DNA synthesized?

(S) Synthesis phase

What is the smallest unit of matter that has the properties of a chemical element?

The atom

What are the products of the Krebs cycle?

6- NADH 2- FADH2 2- ATP 4- CO2

Photosynthesis takes place in this organelle

Chloroplasts

All cellular organisms possess a genetic system that is based on...

DNA

If one strand of DNA is CCAATTG, then the complementary strand is...

GGTTAAC

What percent of the human genome is coding DNA? A.) 1-2% B.) 5-7% C.) 9.5-15% D.) 34-55% E.) It changes during a person's life.

1-2%

What are the 3 stages of translation?

Initiation, elongation, termination

Approximately how many genes are in the human genome?

3 billion base pairs (3,200 Mbp) 20,000-25,0000

Does water or oxygen have a double bond?

Oxygen

Add the complementary strand of DNA to the one below: 5'-A T T G C A T-3'

3'-T A A C G T A-5'

Photosynthesis and Cell Respiration involve ______ reactions.

Redox

Energy flows into the biological world from the...

Sun

What model describes the way that enzymes slightly alter their shape to bind to their substrates (enzyme-substrate complex)?

Induced fit model

Mesophyll cells of plant leaves contain chloroplasts and perform photosynthesis. TRUE or FALSE?

True

Mesophyll cells of plant leaves contain chloroplasts and perform photosynthesis. True or False?

True

Meiosis results in a change in chromosome number which can be written as A.) "3n to 2n." B.) "2n to n." C.) "3n to n." D.) "4n to 3n." E.) "4n to 2n."

"2n to n."

During which phase of the cell cycle is DNA replicated/synthesized?

(S) Synthesis phase

What is a cancer? And, in what ways are cancer cells different from normal cells?

- Abnormal growth of cells caused by mutations to genes that encode checkpoint regulations. - Cancer cells have uncontrolled cell proliferation and tissue invasion.

What is the first stage of cellular respiration? Where does it take lace in the cell?

- Glycolysis - Cytoplasm

Mutations provide the raw material for natural selection, and thus evolution. True or False?

True

In a large population of flowers, the frequency of a trait's recessive allele is 0.4. What is the frequency of the trait's dominant allele? A.) 0.16 B.) 0.6 C.) 0.4 D.) 0.36

0.6

The degree of earlobe attachment in humans is inherited as a simple dominant allele of free earlobes, or as a recessive allele of attached earlobes. What is the probability that a woman with attached earlobes will have children with attached earlobes if their father has free earlobes, but his father had attached earlobes? A.) 1 in 2 B.) 3 in 4 C.) 1 in 10 D.) None

1 in 2

What is the probability of getting a pea plant with recessive wrinkled seeds when two heterozygous pea plants, both having the dominant round-seeded phenotype, are crossed with each other? A.) 1 in 6 B.) 1 in 4 C.) About half D.) None

1 in 4

Lists the steps in the scientific method in their typical order

1) Making observations 2) Formulating a hypothesis 3) Performing experiments and making observations 4) Coming to a conclusion

If a healthy person heterozygous (Tt) for the Tay-Sachs disease autosomal recessive allele marries a healthy individual who is homozygous (TT) for the dominant normal gene, the chances of them having a child with the disease is... A.) 0% chance B.) 25% chance C.) 50% chance D.) 99% chance E.) 100% chance

0% chance

Refer to the genetic code table in your textbook to answer the following questions 1.) What would be the corresponding amino acid sequence (use abbreviation), if the nucleotide sequence of a segment of mRNA read: AUG ACC GGA GGU CCU UAU AAA UGA GCC 2.) What would be the outcome if that mRNA sequence was to be changed to: AUG ACC GGA GGU CCC UAU AAA UGA GCC 3.) What would be the outcome if that mRNA sequence was to be changed to: AUG ACC GGA GGU CGU UAU AAA UGA GCC

1.) TAC TGG CCT CCA GGA ATA TTT ACT CGG 2.) TAC TGG CCT CCA GGG ATA TTT ACT CGG 3.) TAC TGG CCT CCA GCA ATA TTT ACT CGG

If 0.9% NaCl were isotonic compared to a cell, then A.) 0.7% NaCl would by hypertonic to cell. B.) 1.0% NaCl would be hypertonic to cell C.) 1.0% NaCl would be hypotonic to cell. D.) 1.9% NaCl would be hypotonic to cell.

1.0% NaCl would be hypertonic to cell.

A solution with a pH of 4 has _____ times the concentration of H+ present compared to a solution with a pH of 5.

10

Most multicellular eukaryotes have between ______________ chromosomes in their body cells. A.) 1 and 15 B.) 10 and 50 C.) 100 and 200 D.) 200 and 300

10 and 50

Plant and animal cells have an average size in the range of... A.) 1-10 nanometers B.) 10-100 nanometers C.) 1-10 micrometers D.) 10-100 micrometers E.) 1-10 millimeters

10-100 micrometers

White flower color is recessive to purple. What is the genotypic ratio when two heterozygous pea plants are crossed?

1:2:1

Tall is dominant to dwarf in pea plants. If two dwarf pea plants were crossed, their offspring will be...

1:2:1 TT:Tt:tt

After meiosis I ________ cells are formed and after meiosis II, ________ cells result. A.) 2 / 3 B.) 4 /4 C.) 2 / 4 D.) 4 / 8 E.) 2 / 8

2 / 4

How many net ATP are produced in glycolysis?

2 ATP are produced in glycolysis

How many net ATP are produced in glycolysis?

2 ATP are produced in glycolysis.

The first electron shell in any atom contains one orbital which may contain as many as... A.) 2 electrons. B.) 8 protons. C.) 8 electrons. D.) 4 neutrons.

2 electrons

A plant species has its DNA analyzed. The results show that 28% of its nitrogen bases are guanine. What is the percent represented by thymine?

22%

Both parents are heterozygous for the autosomal recessive Tay-Sachs disease. If three children in a row were born with Tay-Sachs disease, what are the chances that the next child will have Tay-Sachs disease?

25%

The ability to taste PTC is a dominant trait. If a heterozygous woman has a child with a man who is not a taster, what is the chance their child will be a taster? A.) 100% B.) 75% C.) 50% D.) 25% E.) 0%

25%

If a healthy person heterozygous for an autosomal recessive allele for a disease marries another healthy individual who is also heterozygous for that gene, the chances of them having a child with the disease is... A.) 0% chance B.) 25% chance C.) 50% chance D.) 99% chance E.) 100% chance

25% chance

Below is a diagram for a chemical reaction in which hydrogen peroxide is converted to water and oxygen catalyzed by the enzyme catalase. Which chemical(s) is the reactant(s) (or substrate of the enzyme) and which chemical(s) is the product(s)? Catalase 2H2O2 --> 2H2O + O2 (Hydrogen peroxide --> water + oxygen)

2H2O is the reactant. 2H2O + O2 are the products.

The number of different possible gametes that an organism can produce due to independent assortment is 2^n, where n is the number of chromosome pairs. How many different kinds of gametes can be produced by humans due to independent assortment only? A.) 23^2 = 529 B.) 46^2 = 2116 C.) 23^4 = 279, 841 D.) 2^23 = 8, 388, 608 E.) 2^46 = 7.04 X 1013

2^32 = 8, 388, 608

If the mRNA codon for alanine is 5'—GCA—3', then the correct anticodon would be A.) 3'—GCA—5' B.) 3'—CGT—5' C.) 3'—ACG—5' D.) 3'—CGU—5' E.) 3'—UGC—5'

3'—CGU—5'

Transcription of a template strand of a DNA molecule with a nucleotide sequence of 5' - A-A-A-C-A-A-C-T-T -3' results in a mRNA molecule with which complementary sequence? A.) 3'—U-U-U-G-T-T-G-A-A—5' B.) 3'—C-C-C-A-C-C-T-C-C—5' C.) 3'—G-G-G-A-G-A-A-C-C—5' D.) 3'—T-T-T-G-A-A-G-C-C—5' E.) 3'—U-U-U-G-U-U-G-A-A—5'

3'—U-U-U-G-U-U-G-C-C—5'

Arrange the following events in the proper sequence in the development of cancer A. Angiogenesis - blood vessels arise and service the tumor cells B. Metastasis - the tumor cells invade the blood and establish secondary tumor C. Tumor cells grow in an uncontrolled and disorganized manner, invading nearby tissues D. Cell acquires mutations in proto-oncogenes and tumor suppressor genes

3, 4, 2, 1

If an organism's diploid number is 64, its haploid number would be...

32

How many molecules of ATP can be generated from the complete cellular aerobic respiration of one molecule of glucose?

36 ATP molecules can be generated from the complete cellular aerobic respiration of one molecule of glucose

How many molecules of ATP can be generated from the complete cellular aerobic respiration of one molecule of glucose?

36 ATP molecules can be generated from the complete cellular aerobic respiration of one molecule of glucose.

At what temperature do most human enzymes work best at?

37 degrees Celsius or 98 degrees Fahrenheit

How much of the available energy stored in glucose is converted to ATP? A.) 68% B.) 7% C.) 26% D.) 50% E.) 39%

39%

The diploid number of chromosomes in humans is _________. A. 76. B. 24. C. 23. D. 46.

46

The diploid number of chromosomes in humans is __________. A.) 76. B.) 24. C.) 23. D.) 46.

46.

In eukaryotic cells, a primary-mRNA (also called pre-mRNA) must be processed to become a mature mRNA. What 3 things occur during mRNA processing?

5' end is capped 3' end has a poly-A tail added Splicing

Below is a nucleotide sequence of the sense strand (also known as the coding strand or the positive strand) of DNA for a gene. Which would represent the correct mRNA made from this gene? 5'—C G A T A C G G A C T—3' A.) 5'—A C U A U A C C U A U A—3' B.) 5'—C G A U A C AG G A C U C—3' C.) 5'—G C T U T G C C T G U G—3 D.) 5'—G C T U T G C C U G A G—3'

5'—G C T U T G C C T G U G-3'

About how many times will a normal human cell divide in laboratory culture? A.) 5 B.) 10 C.) 50 D.) 100

50

Earlobe attachment is inherited as a dominant allele of free earlobes (F) or a recessive allele of attached earlobes (f). What is the probability that a woman with attached earlobes will have children with attached earlobes if her husband has free earlobes, but his father had attached earlobes? A.) 0% that the children will have attached earlobes B.) 10% that the children will have attached earlobes C.) 25% that the children will have attached earlobes D.) 50% that the children will have attached earlobes E.) 100% that the children will have attached earlobes

50% that the children will have attached earlobes

What are the products of the Krebs cycle?

6 CO2 ATP Electron carriers

The second shell in an atom can hold a total of up to______ electrons A.) 3 B.) 4 C.) 8 D.) 12

8

In guinea pigs, B = black, b = brown, S = short hair, s = long hair. What would be the expected results if a BbSs mated with a BbSs? A.) 9 black short hair, 6 black long hair, 3 brown long hair, 1 brown short hair B.) 9 black short hair, 3 black long hair, 3 brown short hair, 1 brown long hair C.) 9 black long hair, 3 black short hair, 3 brown long hair, 1 brown short hair D.) 9 black short hair, 3 black long hair, 3 brown long hair, 1 brown short hair E.) 6 black short hair, 6 black long hair, 2 brown short hair, 2 brown long hair

9 black short hair, 3 black long hair, 3 brown short hair, 1 brown long hair

An inactive X chromosome which is found in females is referred to as... A.) A Barr body. B.) A histone. C.) A nucleosome. D.) A repressible chromosome. E.) An inducible chromosome.

A Barr body.

In a solution, if the hydroxide ion concentration exceeds the hydrogen ion concentration the solution is considered to be... A.) A buffer. B.) An acid. C.) A neutral solution. D.) A base.

A base.

A chemical which resists pH changes in a solution is... A.) A hydrophilic solution. B.) A base. C.) A buffer. D.) A hydrophobic solution. E.) An acid.

A buffer.

In plants, cytokinesis occurs with the formation of... A.) A cell plate B.) A set of asters and centrioles C.) A cleavage furrow in the cell wall D.) A cleavage furrow in the cytoplasm E.) A cleavage furrow in the nuclear membrane

A cell plate

In biology, evolution is defined as... A.) A progressive "ladder" of changes from most primitive to most advanced organisms. B.) A change in a physical trait of an individual during its lifetime. C.) A change in the frequency of alleles in a population over time. D.) A change in the frequency of a physical trait in a population over time. E.) Survival of the fittest.

A change in the frequency of alleles in a population over time.

Many people think the term "theory" means someone's idea about something. Describe the scientific use of the term theory, especially as it relates to the biological concepts.

A concept that seems to be true or proven to be right Deemed to be right, not an educated guess

The removal of a hydroxyl group and a hydrogen atom from two molecules to join them together is known as... A.) A dehydration reaction. B.) A hydrophilic reaction. C.) A monomer reaction. D.) A polymer reaction. E.) A hydrolysis reaction.

A dehydration reaction.

The plasma membrane is... A.) A carbohydrate layer that surrounds groups of cells to protect them. B.) A double lipid layer with proteins inserted in it, which surrounds every cell individually. C.) A thin sheet of structural proteins that encloses cytoplasm. D.) Composed of proteins that form a protective barrier.

A double lipid layer with proteins inserted in it, which surrounds every cell individually.

Modern corn is considered an allotetraploid, which means: A.) The offspring contain half the number of chromosomes as the parent plant. B.) The offspring contain double the number of chromosomes as the parent plant. C.) A doubling of chromosomes has occurred during hybridization. D.) A halving of chromosomes has occurred during hybridization. E.) The number of chromosomes is highly variable from generation to generation.

A doubling of chromosomes has occurred during hybridization.

One source of genetic variation resulting from meiosis is the independent assortment of chromosomes. Independent assortment refers to the fact that... A.) A gamete can receive any combination of the maternal and paternal chromosomes. B.) A zygote can be formed by any combination of the trillions of gametes formed. C.) A gamete will always receive only the haploid number of chromosomes. D.) A gamete will receive chromosomes that have been combined from crossing over.

A gamete can receive any combination of the maternal and paternal chromosomes.

Some organisms are able to reproduce both sexually and asexually. True or False?

True

During the beginning stages of allopatric speciation... A.) Gene flow continues between subpopulation. B.) Reproduction between the two species continues. C.) A geographic separation occurs between the subpopulation. D.) Wide phenotypic differences appear between the subpopulations. E.) Geographic isolation has not occurred.

A geographic separation occurs between the subpopulation.

The result of breeding two closely related species is referred to as... A.) An alloploid. B.) A polyploid. C.) A hybrid. D.) An autoploid. E.) A sub-species.

A hybrid.

When we digest proteins in our food to obtain amino acids, we are carrying out which of the following chemical reactions? A.) An isomeric reaction B.) A hydrolysis reaction C.) A dehydration reaction D.) A hydrophobic reaction E.) A hydrophilic reaction

A hydrolysis reaction

The notion that organisms that are more distantly related should have time to accumulate more biochemical differences than those more closely related can best be illustrated using... A.) Radioactive isotope dating. B.) Gel electrophoresis. C.) The geologic time scale. D.) A molecular clock.

A molecular clock.

What is a trait that is controlled by a combination of genes plus environmental influences? A.) A polygenic phenomenon B.) A multifactorial trait. C.) A polygenic trait. D.) A phenotypic trait. E.) A codominant trait.

A multifactorial trait.

Which type of mutation could be passed on to your child? A.) A mutation in one of your somatic cells B.) A mutation in one of your germ-line cells

A mutation in one of your germ-line cells

According to the fluid-mosaic model, a cell membrane is assembled from.. A.) A phospholipid bilayer with embedded proteins B.) Charged and uncharged fatty acids C.) A bilayer of glycolipids on top of cholesterol D.) Microtubules that keep it intact

A phospholipid bilayer with embedded proteins

Define the term: photoautotroph

A photoautotroph is any organism that derives its energy for food synthesis from light, is capable of using carbon dioxide, and is capable of using its principle source of carbon.

When there is an unequal and asymmetric sharing of electrons between two atoms joined together by a covalent bond, what will be the result? A.) A nonpolar molecule, such as methane. B.) An ionic compound, such as salt. C.) An electronegative molecule, such as calcium ion. D.) A polar molecule, such as water.

A polar molecule, such as water.

A scientist who studies a group of field mice all living in the same area would be studying... A.) A biosphere. B.) A species. C.) An ecosystem. D.) A population. E.) A community.

A population.

A scientist observed a cell which is stopped at the M checkpoint for an unusually long time. The scientist should guess that... A.) A problem has occurred with chromosome distribution. B.) The cell hasn't finished synthesizing DNA. C.) Chromatin has not condensed yet. D.) Organelles are blocking spindle formation. E.) There are not enough microtubules.

A problem has occurred with chromosome distribution.

Imagine a human gene that encodes for a protein with the following amino acid sequence- [methionine - isoleucine - alanine - alanine - proline - serine]. If you made transgenic bacteria by transferring that gene into bacterial cells, the bacteria would produce... A.) A protein with a different amino acid sequence. B.) A protein with the same amino acid sequence. C.) Either the same protein or a different one depending on the type of bacteria it was. D.) Cannot tell with the given information. E.) Nothing from that gene.

A protein with the same amino acid sequence.

When mRNA exits a eukaryotic cell's nucleus, it next becomes associated with _________ in order to be translated. A.) A lysosome B.) The cytoplasmic membrane C.) A ribosome D.) RNA polymerase E.) mRNA does not exit the nucleus

A ribosome

Triglycerides function as....

A rich source of chemical energy for cells

Cells are usually rather small. What does this ensure that they have? A.) A sufficient surface area-to-volume ratio B.) An exchange of genetic information C.) A need for lipid-based membranes D.) A reduction in size due to division E.) A group of new cells by division

A sufficient surface area-to-volume ratio

What is the fate of the NADH and FADH2 made during glycolysis and the Krebs cycle? A. They carry electrons to the electron transport chain. B. They carry electrons to water. C. They carry electrons to pyruvate. D. They carry electrons to the Calvin cycle to reduce carbon dioxide. E. They carry electrons to the cytoplasm.

A. They carry electrons to the electron transport chain

What happens inside a normal cell when a stimulatory growth factor binds to its receptor of the cell membrane? A.) A transcription factor turns on a proto-oncogene, which inactivates the cell cycle. B.) A transcription factor turns on a tumor suppressor gene, which inactivates the cell cycle. C.) A transcription factor turns on mechanisms that cause mutations in tumor suppressor genes. D.) A transcription factor turns on a proto-oncogene, which activates the cell cycle. E.) A transcription factor turns on a tumor suppressor gene, which activates the cell cycle.

A transcription factor turns on a proto-oncogene, which activates the cell cycle.

What happens in a normal cell when an inhibitory growth factor binds to its receptor? A.) A transcription factor turns on mechanisms that cause mutations in tumor suppressor genes. B.) A transcription factor turns on a proto-oncogene, which activates the cell cycle. C.) A transcription factor turns on a proto-oncogene, which inactivates the cell cycle. D.) A transcription factor turns on a tumor suppressor gene, which activates the cell cycle. E.) A transcription factor turns on a tumor suppressor gene, which inactivates the cell cycle.

A transcription factor turns on a tumor suppressor gene, which inactivates the cell cycle.

Under what environmental conditions do yeast carry out fermentation? A. Absence of oxygen B. High temperature C. Low pH D. High osmotic pressure E. All of the choices are correct.

A. Absence of oxygen

What is produced when yeast are used to make beer? A. CO2 and ethanol B. Oxygen and ethanol C. Water, hydrogen gas, and ethanol D. Lactate and ethanol

A. CO2 and ethanol

Which of the following becomes reduced as a result of the Calvin cycle? A. carbon dioxide is reduced to glucose B. oxygen is reduced to water C. light is reduced to fire D. water is reduced to acid

A. Carbon dioxide is reduced to glucose

Depending on the body's or cell's needs, acetyl-CoA is either directed into the Krebs cycle or... A. channeled into fat synthesis. B. fermented into alcohol. C. sent back to glycolysis. D. sent to the electron transport chain.

A. Channeled into fat synthesis

The substance that initially traps solar energy in photosynthesis is... A. chlorophyll. B. RuBP. C. water. D. glucose. E. pyruvate.

A. Chlorophyll

The site inside a eukaryotic cell where photosynthesis takes place is the... A. chloroplast. B. cytoplasm. C. nucleus. D. Golgi complex.

A. Chloroplast

What causes cells to become cancerous? A. Damage to genes controlling cell division B. Chemical damage to cell membranes C. UV damage to transport proteins D. A disconnect between the two G phases E. Skipping the S phase in the cell cycle

A. Damage to genes controlling cell division

Select the correct sequence depicting the breakdown of glucose during cellular respiration. A. Glycolysis - pyruvate - acetyl - CoA - electron transport chain - Krebs cycle B. Glycolysis - pyruvate - acetyl - CoA - Krebs cycle - electron transport chain C. Glycolysis - acetyl - CoA - pyruvate - electron transport chain - Krebs cycle D. Glycolysis - acetyl - CoA - pyruvate - Krebs cycle - electron transport chain

A. Glycolysis - pyruvate - acetyl - CoA - electron transport chain - Krebs cycle

Which of the following result from mitosis? (Select all that apply) A. healing of a cut in the skin B. growth of bacteria C. a plant growing longer roots D. growth of cancer E. production of gametes (sperm/egg)

A. Healing of a cut in the skin C. a plant growing longer roots D. Growth of cancer

Which of these is an electron carrier molecule? A. NADH B. ATP C. Adenylate kinase D. Carbonic acid

A. NADH

In the light-independent ("synthesis") reactions of photosynthesis, glucose is formed from carbon dioxide. The source of electrons and hydrogen to form the new covalent bonds in glucose come from... A. NADPH B. glucose C. ATP D. salt E. X-rays

A. NADPH

Cellular respiration... A. oxidizes glucose and reduces oxygen. B. oxidizes oxygen and reduces glucose. C. releases energy and reduces water. D. requires energy and releases NADH.

A. Oxidizes glucose and reduces oxygen

Proto-oncogenes encode proteins that... A. stimulate cell division. B. suppress tumors. C. restrict cell division. D. control programmed cell death.

A. Stimulate cell division

Which is incorrect about photosynthesis? A. The light reactions take place in the cytoplasm and the dark reactions occur in the chloroplast. B. The light-independent reactions don't require light but can occur in its presence. C. The light-dependent reactions produce NADPH and ATP. D. The light-dependent reactions release oxygen. E. Photosystems act as antennae to capture photons.

A. The light reactions take place in the cytoplasm and the dark reactions occur in the chloroplast

Diploid cells have homologous pairs of chromosomes. Which of the following is an example of a homologous pair of chromosomes? A. the two chromosome 1's you have in each of your body cells B. the chromosome 1 and chromosome 22 you have in each of your body cells C. the X and Y chromosome D. maternal chromosome 1 after it is has been duplicated and joined together at the centromere E. human chromosome 1 and mouse chromosome 1

A. The two chromosome 1's you have in each of your body cells

The carbon atoms in the carbohydrate produced by photosynthesis come from carbon dioxide. TRUE or FALSE?

True

A biologist is studying the formation of carbohydrate during photosynthesis, so the best part of the chloroplast to study would be what? A. chloroplast stroma B. outer chloroplast membrane C. thylakoid space D. thylakoid membrane

A. chloroplast stroma

When a person exercises strenuously and their muscle cells are not receiving enough oxygen, the cells may begin to produce lactic acid via a process called _____________. A. fermentation B. aerobic respiration C. photosynthesis D. exocytosis

A. fermentation

The first stage of cellular respiration, called ___________, takes place in the cytoplasm of the cell and needs no oxygen. A. glycolysis B. Krebs cycle C. photorespiration D. oxidation

A. glycolysis

Which of the following is NOT a stage of mitosis? A. Interphase B. Prophase C. Anaphase D. Metaphase E. Telophase

A. interphase

Which of the following best represents the "synthesis" part of photosynthesis? A. production of glucose from CO2, ATP, and NADPH B. production of glucose from light and H2O C. production of ATP and NADPH from light and H2O D. production of ATP from glucose, H2O, and O2 E. Sorry, nothing is synthesized during photosynthesis

A. production of glucose from CO2, ATP, and NADPH

The full range of energy in sunlight can best be described as... A. the electromagnetic spectrum. B. visible light. C. ultraviolet light. D. infrared radiation.

A. the electromagnetic spectrum

Name the proper phase of mitosis to match each description below A.) The sister chromatids are separating and moving towards the two poles of the cell. B.) The nuclear envelope forms and the chromosomes de-condense. C.) The nuclear envelope disappears, and the chromosomes become distinct from condensation. D.) The chromosomes are lined up on the equator of the cell by spindle fibers. E.) The cell cytoplasm is cleaved, resulting in 2 daughter cells.

A.) Anaphase B.) Telophase C.) Prophase D.) Metaphase E.) Cytokinesis

Name the proper stage of the eukaryotic cell cycle to match each description below A.) The cell recovers from a previous cell division, and if it commits to dividing again, it increases in size, doubles its organelles, and accumulates nucleotides for DNA synthesis. B.) The DNA is copied into two duplicates. At the end of this stage, each chromosome is composed of two identical sister chromatids. C.) The cell carries out its normal metabolic processes of living and functioning, but it is not preparing for growth / reproduction / proliferation. D.) The nucleus and chromosomes are divided into two, passing onto the 2 daughter cells. E.) The cell cytoplasm is cleaved, resulting in 2 daughter cells.

A.) G1 stage B.) S stage C.) G0 stage D.) Mitosis E.) Cytokinesis

Biochemist Erwin Chargaff found that in DNA there was a special relationship between individual bases that we now refer to as Chargaff's rule. His observation was that... A.) A = T and G = C B.) C = T and A = G C.) A + T = C + G D.) G = T and C = A

A=T and G=C

As a result of the light-dependent ("photo") reactions of photosynthesis, _______ is generated by chemiosmosis (a proton-motive force). A.) Amino acids B.) H2O C.) Glucose D.) Chlorophyll E.) ATP

ATP

The energy released by the catabolism of glycogen into glucose can be converted into another form of chemical energy, called ________, which is used directly to fuel your body's anabolic reactions.

ATP

The ATP which is made during the electron transport chain is made at which site? A.) ATP reductase B.) ATP cytochrome complex C.) ATP cytochrome oxidase D.) ATP synthase complex E.) ATP coenzyme

ATP synthase complex

What would occur if the thylakoid membrane was highly permeable to hydrogen ions? A.) Chemiosmosis would occur more rapidly. B.) ATP synthesis would be greatly enhanced. C.) ATP synthase complexes would not work. D.) Water would not be split in the light reactions. E.) The electron transport chain would function more efficiently.

ATP synthase complexes would not work.

If the electron transport chain stopped during photosynthesis, what would be the direct result? A.) Water would not be split. B.) The plant could not absorb light. C.) ATP would not be produced. D.) No electrons would be excited in the photosystem. E.) No pigment molecules would be made.

ATP would not be produced.

Approximately how much of the radiant energy from the sun is incorporated into organic compounds in the process of photosynthesis?

About 25%

Under what environmental conditions do yeast carry out fermentation? A.) Absence of oxygen B.) High temperature C.) Low pH D.) High osmotic pressure E.) All of the choices are correct.

Absence of oxygen

Chlorophylls are important to many plants because these pigments are able to... A) Remove carbon dioxide from the air. B) Absorb water so that hydrolysis can be carried out in the chloroplasts. C) Absorb red and blue wavelengths of light very effectively. D) Store electrons from use during the "dark" reaction of photosynthesis.

Absorb red and blue wavelengths of light very effectively.

What is acid rain? Provide some examples of the negative side effects acid rain has on ecosystems?

Acid rain is precipitation with a pH of 5 or less. The negative side effects that acid rain has on an ecosystem include its ability to kill fish, kill plants, and cause asthma.

From smallest to largest, what is the correct size order of the cytoskeleton fibers? A.) Microtubules, intermedia filaments, actin filaments B.) Actin filaments, microtubules, intermediate filaments C.) Actin filaments, intermediate filaments, microtubules D.) Intermediate filaments, actin filaments, microtubules E.) Microtubules, actin filaments, intermediate filaments

Actin filaments, microtubules, intermediate filaments

Maltose induces expression of the mal operon by... A.) Activating the Maltose activator protein to bind DNA and recruit RNA polymerase. B.) Activating the Maltose activator protein to bind DNA and block RNA polymerase. C.) Inhibiting the Maltose activator protein from binding DNA thereby allowing RNA polymerase to proceed. D.) Inhibiting the Maltose activator protein from binding DNA thereby preventing RNA polymerase from binding.

Activating the Maltose activator protein to bind DNA and recruit RNA polymerase.

Even with exergonic reactions, some energy input is required to destabilize existing chemical bonds and to initiate a chemical reaction that will finally release a net gain of energy. This extra initial energy that must be added to get the reaction going is called... A.) Exergonic energy. B.) Endergonic energy. C.) Kinetic energy. D.) Activation energy.

Activation energy.

The Maltose activator protein binds to a(n)... A.) Operates downstream of the promoter. B.) Operator upstream of the promoter. C.) Activator binding site downstream of the promoter. D.) Activator binding site upstream of the promoter.

Activator binding site upstream of the promoter.

Describe 2 mechanisms by which a transcription factor can induce / activate transcription of a gene in prokaryotes.

Activator protein bound near a weak promoter may bend the DNA forming a better contact site for binding RNA polymerase. Activator protein may directly bind/interact with the RNA polymerase to recruit it to the promoter

In the absence of lactose, the lac repressor is... A.) Active and can bind to the operator. B.) Active and cannot bind to the operator. C.) Inactive and can bind to the operator. D.) Inactive and cannot bind to the operator.

Active and can bind to the operator.

The site on an enzyme where the reactant fits is referred to as the...

Active site

The site on the surface of a chemical reactant where it binds to an enzyme is called the...

Active site

What process uses both a carrier protein and energy to move a solute from an area of low concentration to an area of high concentration? A.) Active transport B.) Pinocytosis C.) Diffusion D.) Osmosis E.) Facilitated diffusion

Active transport

Gregor Mendel's work was different from that of other researchers working on inheritance in his day because he alone... A.) Was a religious man. B.) Worked with pea plants. C.) Knew how to grow plants. D.) Actually counted the numbers and types of offspring.

Actually counted the numbers and types of offspring.

The differences in the 14 species of finches that inhabit the Galapagos Islands and Cocos Island are the result of... A.) Extinction. B.) Population growth. C.) Adaptive radiation. D.) Convergent evolution.

Adaptive radiation.

Which of the following nitrogenous bases would be correctly paired in a strand of DNA? A.) Adenine-cytosine; guanine-thymine B.) Adenine-guanine; thymine-cytosine C.) Adenine-uracil; guanine-cytosine D.) Adenine-thymine; guanine cytosine

Adenine-thymine; guanine-cytosine

What term describes the tendency of water molecules to cling to the wall of a blood vessel?

Adhesion

Water moving up into a paper towel is attributable to... A.) Heat storage. B.) Electronegativity. C.) Cohesion. D.) Adhesion

Adhesion.

An organisms that has more or less than the normal number of chromosomes is called A.) Haploid. B.) Tetraploid. C.) Aneuploid. D.) Triploid.

Aneuploid.

Organisms using this type of catabolism could grow more rapidly and with less food intake. A.) Fermentation B.) Anaerobic respiration C.) Aerobic respiration D.) All of the above are equivalent

Aerobic respiration

Which of the following catabolic pathways generates the most ATP from a molecule of glucose? A.) Photosynthesis B.) Fermentation C.) Arginine anabolism D.) Aerobic respiration

Aerobic respiration

Compare aerobic respiration, anaerobic respiration, and fermentation in terms of energy yield per molecules of glucose.

Aerobic respirstion forms 32 ATP molecules. Anaerobic respiration forms 2 ATP molecules. Fermentation forms 2 ATP molecules.

Compare aerobic respiration, anaerobic respiration, and fermentation in terms of energy yield per molecule of glucose.

Aerobic respirstion forms 32 ATP molecules. Anaerobic respiration forms 2 ATP molecules. Fermentation forms 2 ATP molecules.

Albinism is recessive to normal pigmentation in animals. If two albino animals were crossed their offspring will be...

Albino

Compare ethanol and lactic acid fermentation with regard to their products and the types of cells in which they occur.

Alcohol fermentation occurs in yeast and other bacterial forms. Its end product is ethanol and carbon dioxide. Lactic Acid fermentation occurs in human muscles as well as bacteria found in yogurt. Its end product is lactic acid.

The final stimulus that convinced Darwin to publish his findings came from ________ who had come to the same conclusion as Darwin from his own observations. A.) Thomas Malthus B.) Alfred Russell Wallace C.) John Henslow D.) Charles Lyell

Alfred Russell Wallace

The ability of cells to regulate gene expression allows for... (select all that apply) A.) Bacteria to make enzymes to catabolize certain sugars only when needed. B.) Human bodies to make different cell types (like skin cells versus muscle cells). C.) Plants to have different cell types (like those of the leaves vs. cells of the roots). D.) Cells reproduce in response to growth factors. E.) ATP synthesis via chemiosmosis.

All except plants to have different cell types (like those of the leaves vs. cells of the roots) and ATP synthesis via chemiosmosis

Which of the following depends on mitosis? (select all that apply) A.) Healing of a cut in your skin B.) A plant growing longer roots C.) The growth of a cancer D.) Growth of a mold E.) Growth of bacteria

All except the growth of bacteria

Life can be generally defined by a group of characteristics possessed by all living things. Which is NOT one of these basic shared characteristics? A.) All living things can take in and use energy. B.) All living things can reproduce. C.) All living things are composed of one or more cells. D.) All living things possess an inherited information base. E.) All living things are able to move from one place to another.

All living things are able to move from one place to another.

Where are ribosomes found inside the eukaryotic cell? A.) Rough endoplasmic reticulum B.) Cytosol / cytoplasm C.) Mitochondria D.) All of the above

All of the above

Which of the following types of living organisms are capable of performing photosynthesis? A.) Algae B.) Plants C.) Certain bacteria D.) All of the above E.) A and B only

All of the above

Which products made during photosynthesis are needed by animals to live and grow? A.) Carbon dioxide B.) Oxygen C.) Sugar D.) All of the above E.) B and C

All of the above

What are major differences between mitosis and meiosis? A.) Mitosis results in two diploid cells, but meiosis results in four haploid cells B.) Meiosis involves a reduction division C.) Meiosis involved two cell divisions, whereas mitosis involved one cell division D.) All of the above are true differences between mitosis and meiosis E.) A and B only

All of the above are true differences between mitosis and meiosis.

In order for an enzyme to work properly, A.) It must have its particular 3-D shape. B.) The temperature must be within certain limits. C.) The pH must be within certain limits. D.) All of the above.

All of the above.

RNA polymerase... A.) Is needed for adding nucleotides during mRNA synthesis. B.) Synthesizes new RNA only in the 5' to 3' direction. C.) Cannot add nucleotides to the 5' end of an RNA strand. D.) Synthesizes an RNA from a template DNA strand running in the 3' to 5' direction. E.) All of the choices are correct.

All of the choices are correct.

Which of the following macromolecules can be broken down into intermediate products that enter cellular respiration? A.) Proteins B.) Lipids C.) Nucleic acids D.) Carbohydrates E.) All of the choices are correct.

All of the choices are correct.

Which of the following problems exist when performing human gene therapy? (Select all that apply) A.) The virus vector used to carry the gene may elicit a strong immune response, leading to rejection of that added gene. B.) The virus vector may insert its DNA into the middle of a gene which could lead to cancer. C.) The genes carried by the virus vector could be used to produce more viruses. D.) The virus vector may not deliver the gene into enough cells to have a physiologic effect.

All of the choices are correct.

Which of the following can cause cancer? A.) UV rays B.) Viruses C.) Chemicals D.) Environmental contaminants E.) All of the choices can cause cancer.

All of the choices can cause cancer.

Genetic engineering in agriculture has been employed to make crops... A.) Resistant to insect pests. B.) Resistant to certain bacteria. C.) More nutritious. D.) All of these are correct.

All of these are correct.

Which of the following is NOT produced during the Krebs cycle? A.) Carbon dioxide B.) NADH C.) FADH2 D.) ATP E.) All of these are produced during the Krebs cycle.

All of these are produced during the Krebs cycle.

Darwin's evidence that evolution occurs included... A.) The existence of fossils that closely resembled living specimens. B.) A distinctive distribution of plants and animals in lands with similar climates that were not physically connected. C.) Unique but similar species on several nearby islands. D.) All of these were Darwin's evidence for evolution.

All of these were Darwin's evidence for evolution.

Artificial selection has been used on corn to produce plants with increased growth rates and greater resistance to fungus infection. Although farmers have continued to select for these traits over successive plant generations, the productivity of the selected corn strain is no longer increasing. This result suggests that A.) The corn population size has been decreasing. B.) All or most of the natural variation for these traits has been eliminated. C.) Gene migration is a major evolutionary agent in corn. D.) Long-term disruptive selection may lead to speciation. E.) Artificial selection is not as strong as natural selection.

All or most of the natural variation for these traits has been eliminated.

In Mendel's experiments, two true-breeding pea plants were crossed, one with purple flowers (PP) and the other with white (pp). Their offspring were...

All purple (heterozygous)

Alternative forms of a gene that influence the same trait and are found at the same location in homologous chromosomes are called ________. A.) Alleles B.) Clusters C.) Phenotypes D.) Chromosomes

Alleles

Alternative forms of a gene that influence the same trait and are found at the same location in homologous chromosomes are called ____________.

Alleles

In a population, some genes exist as alternative forms called _________. The gene for flower color in pea plants is an example in which there is the purple flower color version and the white flower color version of the gene

Alleles

When two different but related species of plants hybridize, this can produce a condition referred to as... A.) Symploidy. B.) Gametoploidy. C.) Autoploidy. D.) Alloploidy. E.) Zygoploidy.

Alloploidy.

A variety of genotypes and phenotypes in a population is useful because it... A.) Makes life more interesting. B.) Makes genetic drift an unlikely occurrence. C.) Means that the gene pool is constant and unchanging. D.) Allows species to survive if the environment changes. E.) Makes gene flow in and out of a population unlikely.

Allows species to survive if the environment changes.

Cells from the hypothalamus and thyroid gland contain the same calcitonin gene, but the calcitonin proteins made in the two tissues differ. This results from A.) Transcriptional repression B.) Chromatin remodeling C.) Alternative pre-mRNA splicing D.) Protein degradation E.) Protein phosphorylation

Alternative pre-mRNA splicing

You observe a cell that has lysosomes and a nucleus, but no chloroplasts or cell wall. Based on these observations, you conclude that the cell is most likely of what kind? A.) Bacterial cell B.) Archaean cell C.) Animal cell D.) Plant cell E.) Prokaryotic cell

Animal cell

The carbon atoms in the carbohydrate produced by photosynthesis comes from carbon dioxide. True or False?

True

Three consecutive bases (codon) in the DNA of a gene represent the code for one... A.) Protein. B.) Nucleotide. C.) Amino acid. D.) Purine.

Amino acid.

A sequence of nucleotides in DNA/RNA can specify the sequence of ______________ in a protein.

Amino acids

As a food source, carbohydrates provide you will all the following EXCEPT: A.) Energy B.) Carbon C.) Monomers to make cell structural polysaccharides D.) Fiber E.) Amino acids

Amino acids

Proteins are made up of_______ strung together by ________ bonds. A.) Amino acids / peptide B.) Monosaccharide / nucleotide C.) Monomers / ionic D.) Polymers / hydrogen

Amino acids / peptide

Flat leaves in plants are an example of what to capture more light? A.) An adaption B.) A homeostasis C.) A mutation D.) A gene E.) A cell

An adaption

How was the cloning of Dolly the sheep first performed? A.) An adult nucleus was inserted into an enucleated egg cell. B.) An egg nucleus was inserted into a sperm cell. C.) A sperm and egg nucleus was inserted into an enucleated egg cell. D.) An egg nucleus was inserted into an enucleated adult cell. E.) A sperm nucleus was inserted into an enucleated egg cell.

An adult nucleus was inserted into an enucleated egg cell.

Foxes, cats, and rabbits have enzymes that are heat-sensitive, causing a seasonal variation in coat color due to.... A.) An environmental effect. B.) Multiple alleles. C.) Pleiotropy. D.) Incomplete dominance.

An environmental effect.

A biological molecule which speeds a reaction by bringing the reactants together is referred to as... A.) A glycerol. B.) An enzyme. C.) A lipid. D.) A carbohydrate. E.) A nucleic acid.

An enzyme

What is an isotope? Are all isotopes radioactive?

An isotope is an atom of a single element that differ in number of neutrons. No, not all isotopes are radioactive.

Which of the metabolic reactions, catabolic or anabolic, would require the most ATP?

Anabolic

If you observed the sister chromatids separating during cell division, what phase of mitosis would you be observing? A.) Telophase B.) Prophase C.) Metaphase D.) Cytokinesis E.) Anaphase

Anaphase

Sister chromatids are separated and pulled to opposite poles during _________ of mitosis.

Anaphase

Sister chromatids are separated and pulled to opposite poles during _____________ of mitosis.

Anaphase

In meiosis, when does separation of homologous chromosomes occur? A.) Prophase I B.) Anaphase I C.) Interphase D.) Anaphase II

Anaphase I (Meiosis I)

What structure joins the cells in heart tissue together, so that the cells work in a coordinated fashion? A.) Tight junctions B.) Gap junctions C.) Extracellular matrix D.) Anchoring junctions E.) Microvilli

Anchoring junctions

Which kingdom contains non-photosynthetic multicellular organisms that digest their food internally? A.) Fungi B.) Plantae C.) Animalia D.) Protista E.) Archaea

Animalia

Why can anti-microtubule agents be used as cancer chemotherapeutic drugs?

Anti-microtubules agents can be used as cancer chemotherapeutic drugs because they stop cell division.

The _______ of a tRNA molecule will bind to the codon of an mRNA molecule. A.) RNA polymerase B.) Anticodon C.) Promoter site D.) DNA sequence E.) Amino acid binding site

Anticodon

Select the scientific name from the following names. A.) Robin B.) Tiger C.) Winged beetle D.) Apis mellifera

Apis mellifera

When you get a sunburn as the result of absorbing too much solar radiation, badly damaged cells can be eliminated by... A.) Interphase. B.) Apoptosis. C.) Spindle formation. D.) Sloughing off. E.) Immune reactions.

Apoptosis.

Mendel's law of segregation implies that the two members of an allelic pair... A.) Are segregated pair wise. B.) Are distributed to the same gamete. C.) Are distributed to separate gametes. D.) Are always code for the identical trait or feature. E.) Are assorted dependently.

Are distributed to separate gametes.

In aerobic cellular respiration, oxygen is used... A.) As an enzyme to catalyze chemical reactions B.) To donate electrons to electron carriers in the electron transport chain C.) As the final electron acceptor of the electron transport chain yielding water as a product. D.) As the final electron acceptor during fermentation to make alcohol. E.) To fill up the gaseous cytoplasm of our cells to give them shape.

As the final electron acceptor of the electron transport chain yielding water as a product.

In aerobic respiration, oxygen is used... A.) As an enzyme to catalyze chemical reactions. B.) To donate electrons to electron carriers in the electron transport chain. C.) As the final electron acceptor of the electron transport chain. D.) As the final electron acceptor during fermentation to make alcohol. E.) To fill up the gaseous cytoplasm of cells to give them shape.

As the final electron acceptor of the electron transport chain.

DNA fingerprinting is a technique employed by forensic scientists to... A.) Assist in the identification of individuals by their respective DNA restriction fragment profiles. B.) Transfer good genes into people with a genetic disorder. C.) Determine whether a person carries a certain disease allele. D.) To amplify a few copies of a piece of DNA into thousands to millions of copies. E.) Combine one piece of DNA with another one.

Assist in the identification of individuals by their respective DNA restriction fragment profiles.

The molecular evidence of how the cytochrome c gene changes with time suggests that this gene evolves... A.) At a constant rate. B.) In spurts. C.) Hardly at all. D.) More rapidly than any other gene.

At a constant rate.

Which of the following scientific theories is a basic theory of chemistry and physics, but not of biology? A.) Atomic B.) Cell C.) Homeostasis D.) Ecosystem E.) Gene

Atomic

In what ways is the Krebs cycle involved in the anabolic pathways of protein synthesis? (hint: consider the intersections of metabolic pathways and the metabolic pool concept)

Attached amino acid from the Krebs cycle is used to produce proteins.

Where are autosomal traits located in an organism's genome?

Autosomes (non-sexual chromosomes)

Which outcome might you expect to happen with a population of wild rabbits if you took them out of the wild and bred them in the absence of any predators for a thousand generations? A.) Average running speed would be increased B.) Average running speed would be decreased C.) Average running speed would stay the same D.) All of the above are equally the same E.) A or B, but not c

Average running speed would be decreased

Most multicellular eukaryotes have between ___________ chromosomes in their body cells. A. 1 and 15 B. 10 and 50 C. 100 and 200 D. 200 and 300

B. 10 and 50

The major goal of the Calvin cycle is to produce what material? A. ATP B. carbohydrate C. NADPH D. RuBP E. NADP+

B. Carbohydrate

During the photo reactions of photosynthesis, ATP molecules are made as a direct result of... A. the Calvin cycle B. Chemiosmosis C. The splitting of water D. light striking the chlorophyll molecules

B. Chemiosmosis

The complex of DNA and histones formed by eukaryotic chromosomes is referred to as... A. genes B. chromatin C. nucleic acid D. membrane

B. Chromatin

In prokaryotic cells, glycolysis and the Krebs cycle take place in the _____________ and the electron transport chain takes place in the ___________. A. mitochondrial matrix / inner mitochondrial membrane B. cytoplasm / cell plasma membrane C. intermembrane space of the mitochondria / mitochondrial matrix D. cytoplasm / mitochondria E. mitochondria / cytoplasm

B. Cytoplam/cell plasma membrane

In eukaryotes, DNA within chromosomes is wrapped around proteins with positive charges called... A. chromatids. B. histones. C. centromeres. D. kinetochores.

B. Histones

Which is incorrect about fermentation? A. It is anaerobic. B. It generates oxygen. C. In yeast, ethanol is produced. D. When human muscles are lacking oxygen, lactate is produced. E. It recycles NADH back to NAD+.

B. It generates oxygen

During which stage(s) of cellular respiration is CO2 produced? A. Glycolysis and Krebs cycle B. Krebs cycle only C. Electron transport chain D. Oxidation of pyruvate to acetyl-CoA and Krebs cycle E. Krebs cycle and electron transport chain

B. Krebs cycle only

All of the following are correct about cancer except: A. malignant tumors are invasive. B. malignant tumors can produce metastases. C. cancer cells can sometimes spread into the bloodstream. D. an oncogene is a cancer cell. E. cancers are the direct result of mutations in growth-regulating genes.

B. Malignant tumors can produce metastases

In the light-independent ("synthesis") reactions of photosynthesis, glucose is formed from carbon dioxide. The source of electrons and hydrogen to form the new covalent bonds in glucose come from... A. ATP B. NADPH C. sunlight D. glucose E. salt

B. NADPH

In cells, DNA replication begins at a site on the DNA called the... A. cleavage furrow. B. origin of replication. C. cell plate. D. binary fission.

B. Origin of replication

During which stage of mitosis do replicated chromosomes condense and the nuclear envelope disappear? A. Interphase B. Prophase C. Telophase D. Metaphase E. Anaphase

B. Prophase

Most of the carbon dioxide that is made during aerobic respiration is from... A. glycolysis. B. the Krebs cycle. C. the electron transport chain. D. the Calvin cycle

B. The Krebs cycle

Most of the atmospheric oxygen occurs as a result of photosynthesis. From which of the following molecules is the oxygen derived? A. Carbon dioxide B. Water C. Glucose D. Chlorophyll E. NADH

B. Water

Light energy arrives at earth in tiny packets called... A. rays. B. photons. C. light waves. D. wavelengths.

B. photons

The energy to power the Calvin cycle come from... A. cellular respiration B. the light-dependent reactions of photosynthesis C. the light-independent reactions of photosynthesis D. oxygen E. chlorophyll

B. the light-dependent reactions to photosynthesis

Bacteria and archaea cells divide by... A.) Mitosis. B.) Fusion. C.) Binary fission. D.) Meiosis.

Binary fission.

The formation of cancers often involves angiogenesis, the formation of new blood vessels. True or False?

True

Which of the following mating pairs, indicated by letters referring to the diploid blood type genotypes, CANNOT produce a child with type O blood? A.) OO x OO B.) BO x AB C.) BO x OO D.) AO x BO

BO x AB

Meiosis occurs in all of the following EXCEPT A.) Plants B.) Animals C.) Fungi D.) Bacteria E.) Algae F.) Protozoa

Bacteria (archaea)

A biologist looks through a microscope at a cell which contains no nucleus. Based solely on this information, to which domain(s) might this organism belong? A.) Bacteria only B.) Archaea only C.) Eukarya only D.) Bacteria or Archaea but not Eukarya E.) Bacteria, Archaea, or Eukarya

Bacteria or Archaea but not Eukarya

A scientist observes a cell that has a cell wall and ribosomes but no ER or nucleus. Based on these observations, he can conclude that the cell is most likely a(n)... A.) Animal cell B.) Bacterial cell C.) Plant cell D.) Archean cell E.) Prokaryotic cell

Bacterial cell

The first genome to be sequenced was... A.) Bacterial. B.) Viral. C.) Human. D.) Plant.

Bacterial.

Starch, glycogen, and cellulose are all polymers of glucose. So then, how can they be distinct polysaccharides with different names?

Banded differently and structural difference

What type of solution has a pH of 8.2? A.) Neutral B.) Acid C.) Ionic D.) Buffer E.) Base

Base

Which type of chemical substance lowers the H+ ion concentration in a solution? A.) Ice B.) Acid C.) Base D.) Water

Base

Imagine a case in which brown eyes (B) are dominant over blue eyes (b). If a brown-eyed woman has a blue-eyed child what would her genotype be? A.) Bb B.) BB C.) Bb D.) BBB E.) Cannot tell with the given information

Bb

Why is crossing over not possible in meiosis II? A.) Because homologous chromosomes are no longer in the same cell. B.) Because the enzymes needed for it to occur are no longer present. C.) Because prophase only occurs in meiosis I. D.) Because meiosis II is much shorter than meiosis I.

Because homologous chromosomes are no longer in the same cell. Only one chromosome

How was Gregor Mendel's work different from that of others studying inheritance of traits before and during his time?

Before Mendel, people had observed inheritance of dominant and recessive traits by following many generations of plants and animals. Mendel kept careful count of his results, including mathematical analysis. Prior to Mendel's work, breeders thought both sexes contributed equally to offspring.

A study of how different courtship patterns lead to reproductive isolation is a study of what? A.) Habitat isolation B.) Mechanical isolation C.) Gamete isolation D.) Behavioral isolation E.) Temporal isolation

Behavioral isolation.

Describe 4 mechanisms by which a transcription factor can induce / activate transcription of a gene in eukaryotes.

Bend DNA, making the promoter more accessible for general transcription components. Directly interact with general transcription factor apparatus. Indirectly interact with general transcription factor apparatus through recruitment of co-activators (acting as a bridge) Recruit co-activators that have activity involved in chromatin remodeling.

What is the role of the general transcription factors? A.) Binding heterochromatin to prevent transcription B.) Binding promoter of a gene to start translation C.) Binding promoter of a gene to start transcription D.) Binding operator to prevent transcription E.) Binding repressor to prevent transcription

Binding promotor of a gene to start transcription

Translation repressor proteins may shut down translation of processed mRNA transcripts by... A.) Binding within the 5' or 3' UTRs B.) Resetting the reading frame C.) Reinserting introns into the transcript D.) Excising a short sequence of nucleotides

Binding within the 5' or 3' UTRs

Define the term Biodiversity.

Biodiversity is the variety of life on Earth. Scientists have estimated that between 10 and 50 million species may exist in all.

How can bioinformatics be applied in functional and comparative genomics?

Bioformatics is the application of computer technologies to the study of biological information (genomes). An example of this is BLAST, which is computer software used to compare genes and can also be used to help trace evolution.

The ___________________ concept determines if a group of organisms makes up a separate species based on whether or not it can interbreed with other such groups.

Biological species

The regulators of transcription are...

Block/Decrease transcription (Repression or negative regulation) Activate/Increase transcription (Activation or positive regulation)

When something appears blue, it is absorbing all colors except _______. A.) Yellow B.) Red C.) Blue D.) Green E.) Orange

Blue

What is Bt Corn?

Bt-corn is a type of genetically modified organism, termed GMO. A GMO is a plant or animal that has been genetically modified through the addition of a small amount of genetic material from other organisms through molecular techniques. The GMOs on the market today have been given genetic traits to provide protection from pests, tolerance of pesticides, or improve its quality. Genetically modified plants to reduce pesticide use in farming. 1.) Corn plants destroyed by butterfly larvae (caterpillars). 2.) Bacterial gene coding for Bt crystals, which are poisonous to the caterpillars, is inserted directly into the corn plant's DNA. 3.) Bt crystals—toxic to caterpillars—are now produced by the corn plant itself and present in all cells, reducing the amount of pesticides the farmer must use.

How are newly made biomolecules transported from one compartment of the endomembrane system to another? A.) Budding and fusion of membrane transport vesicles B.) Sent out into the cytosol through membrane carrier proteins C.) Teleportation D.) Through conjugation bridges that form between the organelles

Budding and fusion of membrane transport vesicles

What keeps are blood from becoming too acidic?

Buffers (bicarbonate)

The overall purpose of the Calvin cycle is to... A.) Generate molecules of ATP. B.) Generate NADP. C.) Give off oxygen for animal use. D.) Build organic molecules.

Build organic molecules.

The fusion of gametes is called fertilization. True or False?

True

About how many times will a normal human cell divide in laboratory culture? A. 5 B. 10 C. 50 D. 100

C. 50

What would occur if the thylakoid membrane was highly permeable to hydrogen ions? A. Chemiosmosis would occur more rapidly. B. ATP synthesis would be greatly enhanced. C. ATP synthase complexes would not work. D. Water would not be split in the light reactions. E. The electron transport chain would function more efficiently

C. ATP synthase complexes would not work.

If the electron transport chain stopped during photosynthesis, what would be the direct result? A. Water would not split B. The plant could not absorb light C. ATP would not be produced D. No electrons would be excited in the photosystem E. No pigment molecules would be made

C. ATP would not be produced

If the electron transport chain stopped during photosynthesis, what would be the direct result? A. Water would not be split. B. The plant could not absorb light. C. ATP would not be produced. D. No electrons would be excited in the photosystem. E. No pigment molecules would be made.

C. ATP would not be produced

Organisms using this type of catabolism could grow more rapidly and with less food intake. A. fermentation B. anaerobic respiration C. aerobic respiration

C. Aerobic respiration

In aerobic respiration, oxygen is used... A. as an enzyme to catalyze chemical reactions. B. to donate electrons to electron carriers in the electron transport chain. C. as the final electron acceptor of the electron transport chain. D. as the final electron acceptor during fermentation to make alcohol. E. to fill up the gaseous cytoplasm of cells to give them shape.

C. As the final electron acceptor of the electron transport chain

Bacteria and archaea cells divide by... A. mitosis. B. fusion. C. binary fission. D. meiosis.

C. Binary fission

The Calvin cycle is directly dependent upon a supply of ... A. water and carbon dioxide. B. carbon dioxide and NADPH. C. carbon dioxide, NADPH, and ATP. D. oxygen and carbon dioxide. E. glucose and carbon dioxide

C. Carbon dioxide, NADPH, and ATP

During which stage of the cell cycle do daughter cells form? A. Metaphase B. Prophase C. Cytokinesis D. S phase E. G1

C. Cytokinesis

Which one of the three stages of cellular respiration generates the most ATP molecules through the production of a proton-motive force (chemiosmosis)? A. Glycolysis B. Krebs cycle C. Electron transport chain D. Calvin cycle

C. Electron transport chain

A proton gradient across a membrane can also be called a(n) ______ gradient. A. Pigment B. Sodium ion (Na+) C. Hydrogen ion (H+) D. Electron E. Water

C. Hydrogen ion (H+)

As the electron carriers of the respiration electron transport chain transfer electrons, they actively pump __________ into the mitochondrial intermembrane space, setting up a concentration gradient called the proton-motive force. A. ATP B. phosphate C. hydrogen ions D. oxygen

C. Hydrogen ions

Cancer cells arise from... A. an inhibition of chlorophyll function. B. an inhibition of glycolysis. C. mutations in the DNA of growth factor genes. D. a disruption of the cell membrane. E. cells to undergo cell death.

C. Mutations in the DNA of growth factor genes

Which of the following is not a pathway that cells use to get energy from food/nutrient molecules? A. Cellular respiration B. Fermentation C. Photosynthesis D. Anaerobic respiration

C. Photosynthesis

In chemiosmosis, a(n) _____ gradient drives the production of ATP using ATP synthase channels. A. Pigment B. Chlorophyll C. Proton D. Electron D. NADPH

C. Proton

In the process of glycolysis, glucose is cleaved into a pair of three-carbon ________ molecules. A. lactate B. acetyl-CoA C. pyruvate D. alcohol

C. Pyruvate

A protein encoded by a tumor-suppressor gene normally acts to... A. mutate the cell. B. stimulate cell division. C. restrict cell division. D. promote metastases.

C. Restrict cell division

Replicate copies of each chromosome are called ________ and are joined at the _________. A. homologues / centromere B. sister chromatids / kinetochore C. sister chromatids / centromere D. homologues / kinetochore

C. Sister chromatids/centromere

Why is a molecule that binds to and blocks the action of ATP synthase lethal to plants? A. ATP synthase splits water and releases oxygen; without the production of oxygen, the plants will die. B. ATP synthase is the enzyme responsible for carbon dioxide fixation. C. The ATP produced by ATP synthase powers the Calvin cycle, the ultimate source of all organic molecules made by the plant. D. When ATP synthase is blocked, the electron transport chain moves electrons backwards to the reaction center. E. If ATP synthase is blocked, chemiosmosis cannot occur, and as a result the plant will run out of water and dehydrate.

C. The ATP produced by ATP synthase powers the Calvin cycle, the ultimate source of all organic molecules made by the plant.

The spindle is... A. the new cell membrane as it reforms across the cytoplasm during cytokinesis. B. a point of constriction on each chromosome containing repeated DNA sequences. C. the network of protein cables that will pull the chromatids to opposite ends of the cell. D. the protein that DNA winds itself around.

C. The network of protein cables that will pull the chromatids to opposite ends of the cell

Mitosis is the part of the eukaryotic cell cycle in which ... A. the DNA is transcribed into mRNA B. cells grow in size preparation for division C. the nucleus divides into two D. the cell/cytoplasm is divided into two E. sorry, mitosis is not part of the cell cycle

C. The nucleus divides into two

How many valence electrons does carbon (C) have? How many valence electrons does oxygen (O) have? How many covalent bonds will each of these be able to form?

Carbon----4-----4 covalent bonds Oxygen-----6-----2 covalent bonds

Chlorophylls are important to many plants because these pigments are able to... A. remove carbon dioxide from the air. B. absorb water so that hydrolysis can be carried out in the chloroplasts. C. absorb red and blue wavelengths of light very efficiently. D. capture UV radiation that is harmful to the DNA in the nucleus of plant cells. E. store electrons for use during the "dark" reaction of photosynthesis.

C. absorb red and blue wavelengths of light very efficiently.

In aerobic cellular respiration, oxygen is used... A. as an enzyme to catalyze chemical reactions B. to donate electrons to electron carriers in the electron transport chain C. as the final electron acceptor of the electron transport chain yielding water as a product. D. as the final electron acceptor during fermentation to make alcohol. E. to fill up the gaseous cytoplasm of our cells to give them shape.

C. as the final electron acceptor of the electron transport chain yielding water as a product

When something appears blue, it is absorbing all colors except __________. A. yellow B. red C. blue D. green E. orange

C. blue

Electrons taken from pyruvate during the Krebs cycle are transferred to NADH and FADH2 which then carry them to the redox reactions of the _____________. A. photo reactions B. centromere C. electron transport chain D. Krebs cycle

C. electron transport chain

Plant leaves absorb _____ and ______ wavelengths of light and reflect ____ wavelengths. A. green/red/blue B. green/blue/red C. red/blue/green D. white/green/blue E. white/green/red

C. red/blue/green

Within chloroplasts, the fluid-like matrix in which the Calvin cycle occurs is called... A. thylakoids. B. grana. C. stroma. D. mesophyll.

C. stroma

The energy to power the Calvin cycle comes from... A. cellular respiration. B. the light-dependent reactions of photosynthesis. C. the light-independent reactions of photosynthesis. D. oxygen. E. chlorophyll.

C. the light-dependent

Write a chemical reaction that represents the overall chemical reaction of cellular respiration

C6H12O6 (glucose) + 6O2 --> 6CO2 + 6H2O + energy

What is produced when beer is made? A.) CO2 and ethanol B.) Oxygen and ethanol C.) Water, hydrogen gas, and ethanol D.) Lactate and ethanol

CO2 and ethanol

Which ions found specifically in bones and teeth are important muscle contractions and nerve conduction? A.) Bicarbonate B.) Sodium C.) Chloride D.) Calcium E.) Potassium

Calcium

Inheritance of an autosomal dominant disorder (like Huntington's disease) differs from inheritance of an autosomal recessive disorder (like Albinism) in that a dominant disorder: A.) Is more often seen in females. B.) Is evidence only if the child inherits two affected (mutant) alleles. C.) Can be passed on to children only if both parents are affected by the disorder. D.) Can be passed on to children even if only one parent is affected by the disorder.

Can be passed on to children even if only one parent is affected by the disorder.

(a) What is a cancer? (b) What is its root cause? And (c) In what ways are cancer cells different from normal cells?

Cancer is the abnormal growth of cells. Cancer is caused by mutations to genes that encode checkpoit regulations. Cancer cells have uncontrollecell proloferation and tissue invasion.

The major goal of the Calvin cycle is to produce what material? A.) ATP B.) Carbohydrate C.) NADPH D.) RuBP E.) NADP+

Carbohydrate

Which of the following is NOT true regarding a cell's metabolic pool? A.) Carbohydrate intake can result in the storage of fat. B.) Some metabolites of the Krebs cycle are used to synthesize amino acids. C.) Fatty acids can be oxidized to acetyl-CoA, which enters the Krebs cycle. D.) Carbohydrates represent the most energy-rich form of stored energy for an organism

Carbohydrates represent the most energy-rich form of stored energy for an organism.

Cellular respiration can transform chemical energy from ______ into the chemical energy of ATP. A.) Carbohydrates, fats, and proteins B.) Carbohydrates and proteins only C.) Fats only D.) Carbohydrates only E.) Proteins only

Carbohydrates, fats, and proteins

Match each type of biomolecule with the subunits that makes up that biomolecule: Carbohydrates Lipids Proteins Nucleic acid

Carbohydrates- monosaccharides Lipids- glycerol and fatty acids Proteins- amino acids Nucleic acid- nucleotides

What are the 6 major elements making up life?

Carbon Hydrogen Nitrogen Oxygen Phosphorus Sulfur

The reactants for photosynthesis are light energy, water, and...

Carbon Dioxide

The reactants for photosynthesis are light energy, water, and....

Carbon dioxide

Which of the following is reduced as a result of the Calvin cycle? A.) Carbon dioxide B.) Oxygen C.) Light D.) Water

Carbon dioxide

What would happen if the molecule RuBP disappeared from a plant? A.) The electron transport chain would stop. B.) The plant could no longer split water. C.) The plant could no longer absorb light. D.) Carbon dioxide fixation would stop. E.) Pigment production would stop.

Carbon dioxide fixation would stop.

The Calvin cycle is directly dependent upon a supply of... A.) Water and carbon dioxide. B.) Carbon dioxide and NADPH. C.) Carbon dioxide, NADPH, and ATP. D.) Oxygen and carbon dioxide. E.) Glucose and carbon dioxide.

Carbon dioxide, NADPH, and ATP.

Carbohydrates are made up of what elements?

Carbon, oxygen, hydrogen

The ________ is a rigid structure, made up of polysaccharides, surrounding a cell that provides for structural support, shape, and protein. A.) Cell membrane B.) Cell wall C.) Ribosome D.) Flagellum

Cell wall

Why is the cell wall important when a plant cell is placed into a hypotonic solution? A.) Active transport occurs through the cell wall. B.) Cell wall prevents the cell from bursting. C.) Cell wall prevents the cell from shrinking. D.) Diffusion occurs through the cell wall.

Cell wall prevents the cell from bursting.

The basic unit of structure and function for all life is the... A.) Atom. B.) Organ. C.) Cell. D.) Molecule. E.) Organism.

Cell.

Cell theory includes the principle that... A.) Cells are the smallest living things; nothing smaller than a cell is considered alive. B.) All cells are surrounded by cell walls that protect them. C.) All organisms are made up of many cells arranged in specialized functional groups. D.) All cells contain membrane-bounded structures called organelles.

Cells are the smallest living things; nothing smaller than a cell is considered alive.

What would happen to a cell if it stopped making ATP?

Cellular respiration would stop working so the cell would die

What would happen to a cell if it stopped making ATP?

Cellular respiration would stop working so the cell would die.

The carbohydrate that is present in plant cell walls for structural purposes is... A.) Starch. B.) Cellulose. C.) Chitine. D.) Fructose.

Cellulose.

What generated the sickle-cell anemia allele of hemoglobin?

Change in amino acid, single base change in DNA, Valine (val) appears at one spot in hemoglobin to form semirigid rods and cells become sickle-shaped

Depending on the body's or the cell's needs, acetyl-CoA is either directed into the Krebs cycle or... A.) Channeled into fat synthesis. B.) Fermented into alcohol. C.) Sent back to glycolysis. D.) Sent to the electron transport chain. E.) Fermented to lactic acid.

Channeled into fat synthesis.

In 1859, this naturalist suggested an explanation for why evolution occurs.

Charles Darwin

Molecules which stimulate or inhibit cell division are types of... A.) Control molecules B.) Signaling molecules C.) Contact molecules D.) Checkpoint molecules

Checkpoint molecules

The making or breaking of chemical bonds is called... A.) Entropy reactions. B.) Chemical reactions. C.) Thermodynamic reactions. D.) Catalysis reactions.

Chemical reactions.

In the electron transport chain, proton pumps drive protons out across the mitochondrial membrane into the intermembrane space. This produces a membrane potential and source of energy referred to as _________.

Chemiosmosis

In the electron transport chain, proton pumps drive protons out across the mitochondrial membrane into the intermembrane space. This produces a membrane potential and source of energy referred to as the _________________.

Chemiosmosis

During photosynthesis, ATP molecules are directly generated by... A.) The Calvin cycle. B.) Chemiosmosis. C.) The splitting of water. D.) Light striking the chlorophyll molecules.

Chemiosmosis.

Energy from electron transfers in the redox reactions of the electron transport chain is used to establish what potential energy source?

Chemiosmotic force (proton-motive force) is the potential energy source established from energy from electron transfers in the redox reactions of the electron transport chain.

The substance that initially traps solar energy in photosynthesis is... A.) Chlorophyll. B.) RuBP. C.) Water. D.) Glucose. E.) Pyruvate.

Chlorophyll.

Which of the following organelles is NOT present in both plant and animal cells? A.) Nucleus B.) Endoplasmic reticulum C.) Golgi apparatus D.) Mitochondria E.) Chloroplast

Chloroplast

A biologist is studying the formation of carbohydrate during photosynthesis, so the best part of the chloroplast to study would be what? A.) Chloroplast stroma B.) Outer chloroplast membrane C.) Thylakoid space D.) Thylakoid membrane

Chloroplast stroma

Name 3 structures or organelles unique to plant cell anatomy as compared with animal cell anatomy. Define the function of each of these structures/organelles.

Chloroplast- An organelle of plants and algae where photosynthesis occurs. Its functions include generating cellular energy (ATP) from sunlight and water, and using that energy to produce glucose from carbon dioxide. Cell wall- The layer surrounding the plasma membrane. Its functions includes structural strength and protection. Vacuole- Functions include storage and physical support.

Two organelles that are thought to have once been free-living bacterial cells are... A.) Ribosomes and nuclei B.) Golgi bodies and vesicles C.) Peroxisomes and lysosomes D.) Chloroplasts and mitochondria

Chloroplasts and mitochondria

Human cells have this sterol in their cell membranes to strengthen it and improve its fluidity

Cholesterol

An excess of what 2 types of lipids in the body can increase plaque buildup in blood vessels?

Cholesterol and triglycerides

The complex of DNA and histones in eukaryotic chromosomes is referred to as... A.) Genes B.) Chromatin C.) Nucleic acid D.) Membrane

Chromatin

Modification of histones that cause their interaction with the DNA to change or the movement of a nucleosome from one position to another are events in a process called A.) Chromatin immunoprecipitation B.) Chromatin modulation C.) Chromatin remodeling D.) Chromatin supercoiling

Chromatin remodeling

Which cytoskeletal structures on the lining of the respiratory tract are needed to sweep dust and debris away? A.) Flagella B.) Actin filaments C.) Pseudopods D.) Cilia E.) Intermediate filaments

Cilia

When observing an animal cell in telophase, you would expect to observe the formation of a(n)... A.) Cleavage furrow. B.) Cell plate. C.) Spindle. D.) Interphase junction. E.) Anaphase plate.

Cleavage furrow.

With transcription, the mRNA is identical in sequence to the __________ of the DNA (except for the fact that T's in the DNA are replaced by U's in the RNA.) A.) Template strand B.) Coding strand C.) Non-coding strand D.) Anti-sense strand

Coding strand

When two different dominant alleles are both fully expressed in the phenotype, it is referred to as A.) Incomplete dominance. B.) Codominance. C.) Genotype dominance. D.) Polygenic dominance. E.) Pleiotropic dominance.

Codominance.

In the human ABO blood grouping, there are four basic blood types, type A, type B, type AB, and type O. The blood proteins A and B are... A.) Simple dominant and recessive traits. B.) Codominant. C.) Incompletely dominant. D.) Sex-linked traits.

Codominant.

A three base sequence on mRNA that specifies an amino acid is called a(n)... A.) Codon. B.) Exon. C.) Anticodon. D.) Intron. E.) Trifecta

Codon.

The specific sequence of three bases in mRNA that encodes a particular amino acid is the ____________ whereas the complementary sequence of three bases in tRNA that bind to the mRNA is best known as the ___________. A.) Codon; anticodon B.) Anticodon; codon C.) Codon; DNA triplet D.) Gene; complement

Codon; anticodon

What are cofactors? And what are coenzymes?

Cofactors- inorganic, nonprotein component, activator Coenzymes- organic, nonprotein component, carbon-hydrogen structure

Which term describes the tendency of water molecules to cling to other water molecules?

Cohesion

Which line of evidence developed since the time of Darwin has strengthened support for his theory of natural selection? A.) Comparative anatomy B.) Biogeography C.) Fossil record D.) Comparative embryology E.) Comparative biochemistry

Comparative biochemistry

Which of the following types of evidence for evolution was not available during Darwin's time? A.) Fossil record B.) Comparative anatomy C.) Comparative embryology D.) Comparative genomics

Comparative genomics

When an inhibitor blocks the active site of an enzyme, this is referred to as... A.) Noncompetitive inhibition B.) Competitive inhibition

Competitive inhibition

Two pieces of wood, which have shapes that exactly fit together to make a whole, are like the two strands of DNA because both pairs of things can be said to be __________ to one another.

Complementary

What is Golden Rice?

Containing beta-carotene, a precursor to Vitamin A Increased Vitamin A by almost 25-fold

Which of the following is not an example of a genetic modification in a plant? A.) Strawberries which are resistant to freezing B.) Potatoes which are resistant to blight disease and beetles C.) Strawberries which are resistant to freezing D.) Corn that has built in nitrogen fertilization E.) Soybeans which make a desired fatty acid (oleic acid)

Corn that has built in nitrogen fertilization

What types of crops have been produced using genetic engineering in agriculture?

Corn, tobacco, rice, cotton, potato, etc.

The hydrolysis reaction of ATP is called a(n) _______ reaction, because the process _______. A.) Endergonic, must be supplied with energy B.) Kinetic, keeps molecules moving C.) Output, releases products D.) Exergonic, gives off energy E.) Coupled, involves two molecules

Coupled, involves two molecules

The type of bond that forms between two atoms when electrons are shared is a(n) _____ bond. A.) Hydrogen B.) Covalent C.) Kinetic D.) Ionic

Covalent

What type of bond joins oxygen and hydrogen atoms together in a molecule of water?

Covalent bond (polar)

Define the following types of chemical bonds: covalent, ionic, and hydrogen

Covalent bond- Forms when two atoms come together and share electrons so they each have a full outer shell. They can be polar or non polar. Ionic bond- Forms between ions of opposite charge. Hydrogen bond- Forms by the attractive force of a hydrogen. The hydrogen is attached to an electronegative atom of one polar molecule and an electronegative atom of a different polar molecule.

Which of the following organisms uses chitin for structure and support? (select all that apply) A.) Bacterium B.) Human C.) Crab D.) Tree E.) Insect

Crab and insect

Meiosis leads to tremendous genetic variation in the daughter cells. One source of this genetic variation is the exchange of genetic information between homologous chromosomes known as... A.) Random fertilization. B.) Crossing over. C.) Lining up at the equator. D.) Independent assortment.

Crossing over.

The exchange of genetic information between homologous chromosomes is known as... A.) Random fertilization. B.) Independent assortment. C.) Crossing over. D.) Lining up at the equator.

Crossing over.

Cyanide is a poison that binds to the final carrier in the electron transport chain. Why does this property make cyanide deadly?

Cyanide prevents the reduction of oxygen and stops the electron transport chain.

Which of the following disorders is characterized by thick, viscous mucus production in the lungs and digestive tract? A.) Sickle-cell disease B.) Huntington disease C.) Tay-Sachs disease D.) PKU E.) Cystic fibrosis

Cystic fibrosis

Which of the following are part of the electron transport chain's membrane-associated electron carriers? A.) Pyruvate molecules B.) Acetyl groups C.) Cytochrome molecules D.) NADH molecules E.) FADH2 molecules

Cytochrome molecules

During which stage of the cell cycle do daughter cells form? A.) Metaphase B.) Prophase C.) Cytokinesis D.) S phase E.) G1

Cytokinesis

If you were studying the semifluid interior of the cell you would be studying the A.) Cytoplasm B.) Cytoskeleton C.) Centrosome D.) Chromatin E.) Centrioles

Cytoplasm

In prokaryotic cells, glycolysis and the Krebs cycle takes place in the _________ and the electron transport chain takes place in the ______. A.) Mitochondrial matrix / inner mitochondrial membrane B.) Cytoplasm / cell plasma membrane C.) Intermembrane space of the mitochondria / mitochondrial matrix D.) Cytoplasm / mitochondria E.) Mitochondria / cytoplasm

Cytoplasm / cell plasma membrane

In eukaryotic cells, glycolysis takes place in the _________ and the Krebs cycle and electron transport chain takes place in the _________. A.) Mitochondrial matrix / inner mitochondria membrane B.) Cytoplasm / endoplasmic reticulum C.) Intermembrane space of the mitochondria / mitochondrial matrix D.) Cytoplasm / mitochondria E.) Mitochondria / cytoplasm

Cytoplasm / mitochondria

Before any eukaryotic cell can divide into two daughter cells, it must... A. produce sufficient quantities of ATP. B. duplicate its DNA. C. synthesize more organelles (such as mitochondria). D. All of the above E. Only A and C

D. All of the above

Which of the following types of living organisms are capable of performing photosynthesis? A. Algae B. Plants C. Certain bacteria D. All of the above E. A and B only

D. All of the above

Which products made during photosynthesis are needed by animals to live and grow? A. carbon dioxide B. oxygen C. sugar D. All of the above E. B and C

D. All of the above

What would happen if the molecule RuBP disappeared from a plant? A. The electron transport chain would stop. B. The plant could no longer split water. C. The plant could no longer absorb light. D. Carbon dioxide fixation would stop. E. Pigment production would stop.

D. Carbon dioxide fixation would stop.

Which of the following processes does not expend ATP (energy)? A. muscle contraction B. cell movement C. running a 5K race D. cell respiration E. cell growth

D. Cell respiration

In eukaryotic cells, glycolysis takes place in the _____________ and the Krebs cycle and electron transport chain take place in the ___________. A. mitochondrial matrix / inner mitochondrial membrane B. cytoplasm / endoplasmic reticulum C. intermembrane space of the mitochondria / mitochondrial matrix D. cytoplasm / mitochondria E. mitochondria / cytoplasm

D. Cytoplasm/mitochondria

Which of the following statements accurately reflects the process of glycolysis? A. Glycolysis uses molecular oxygen, however it occurs in anaerobic environments. B. Glycolysis uses molecular oxygen and occurs in aerobic environments. C. Glycolysis uses no molecular oxygen and occurs in the mitochondria. D. Glycolysis uses no molecular oxygen, however it can occur in aerobic environments.

D. Glycolysis uses no molecular oxygen, however it can occur in aerobic environments

If a molecule has been reduced, which of the following statements must be true? A. It has lost electrons. B. The number of electrons in the molecule has stayed the same. C. It has lost oxygen. D. It has gained electrons. E. It has gained oxygen

D. It has gained electrons

What happens to pyruvate, the end product of glycolysis, before it enters the Krebs cycle? A. It is reduced. B. It is phosphorylated. C. It is carboxylated. D. It is oxidized.

D. It is oxidized

Yogurt is a food produced by fermentation of milk. Fermentation of lactose by bacteria produces ____________, which acts on milk proteins to give yogurt its texture and characteristic tang. A. sulfate B. propane C. glucose D. lactic acid E. water

D. Lactic acid

During the S-phase of the cell cycle, the chromosomes are _________, forming two identical copies of each chromosome called sister chromatids. A. degraded B. oxidized C. cleaved D. replicated

D. Replicated

Which of the following does not conduct photosynthesis? A. Algae B. Mesophyll cells of plants C. Cyanobacteria D. Root cells of plants

D. Root cells of plants

Which of the following catabolic pathways generates the most ATP from a molecule of glucose? A. photosynthesis B. fermentation C. arginine anabolism D. aerobic respiration

D. aerobic respiration

Once a plant has produced glucose from photosynthesis, the glucose can be catabolized for energy (to make ATP) in the plant cell's... A. nucleus. B. lysosome. C. plasma membrane. D. mitochondria. E. chloroplasts

D. mitochondria

After hydrogens and electrons are stripped from NADPH, the regenerated NADP+ can be used again as a reactant in the ... A. antenna complex. B. the Calvin cycle. C. the Krebs cycle. D. the light-dependent reactions. E. the light-independent reactions.

D. the light-dependent reactions

The portion of the electromagnetic spectrum that is visible to the human eye and is also the source of energy harvested by plants is known as ___________. A. Gamma rays B. X-rays C. UV light D. Visible light E. Infrared

D. visible light

A molecule from a new organism contains adenine, cytosine, guanine, and thymine. What is the unknown molecule? A.) DNA B.) RNA C.) Lipid D.) Protein E.) Carbohydrate

DNA

A frame shift is caused by ________ mutations. A.) Missense and insertion B.) Missense and nonsense C.) Nonsense and deletion D.) Deletion and insertion

Deletion and insertion

The use of DNA probes to produce unique banding patterns of DNA on a film for the identification of a person is called... A.) PCR. B.) cDNA amplification. C.) DNA fingerprinting. D.) Recombinant DNA.

DNA fingerprinting.

During the process of transcription, the information in... A.) RNA is converted into DNA information. B.) RNA is converted into protein information. C.) DNA is converted into RNA information. D.) Protein and converted into RNA information. E.) DNA is converted into protein information.

DNA is converted into RNA information.

Okazaki fragments are attached to the growing end of the lagging strand by... A.) DNA ligases. B.) DNA polymerases. C.) DNA helicases. D.) DNA gyrases. E.) Primases.

DNA ligases.

If a scientist wanted to determine which genes are being expressed in a cell of a specific tissue at a specific time, what technology would she use? A.) Restriction digest B.) Polymerase chain reaction C.) Thermocycling D.) DNA microarray E.) DNA sequencing

DNA microarray

The enzyme that is used to join successive DNA nucleotides together is which? A.) Ribozyme B.) DNA polymerase C.) RNA polymerase D.) Helicase

DNA polymerase

Which enzyme is responsible for building the mRNA transcript from individual nucleotides? A.) RNA Ligase B.) Ribozymes C.) DNA polymerase D.) Helicase E.) RNA polymerase

DNA polymerase

Although DNA replication of one cell involves billions of nucleotides, why is this process so accurate? A.) DNA polymerase proofreads and corrects mistakes in base-pairing. B.) Semiconservative replication means that the process is very careful and conservative. C.) RNA polymerase corrects any mistakes before doing RNA transcription. D.) DNA primers can replace any mistakes in base pairing. E.) Unwinding the double helix would remove any mistakes in base pairing.

DNA polymerase proofreads and corrects mistakes in base-pairing.

The enzymes that can proofread replicating DNA, detect incorrect bases, excise them, and correctly replace them are... A.) DNA ligases. B.) DNA polymerases. C.) DNA helicases. D.) DNA gyrases. E.) Primases.

DNA polymerases.

In cells, the flow of genetic information is from... A.) RNA to protein to DNA B.) RNA to DNA to protein C.) DNA to RNA to protein D.) DNA to protein to RNA E.) Protein to RNA to DNA

DNA to RNA to protein

The significance of the Griffith, Avery, and Hershey-Chase experiments was that they demonstrated that... A.) Genes were located on chromosomes. B.) Enzymes transformed bacteria. C.) Proteins were what genes were composed of. D.) DNA was the molecule of heredity.

DNA was the molecule of heredity.

List 3 features that are different between DNA and RNA (structure or function)

DNA- double-helix strand, H (deoxyribose), thymine RNA- single strand, OH (ribose, extra oxygen), uracil

What causes cells to become cancerous? A.) Damage to genes controlling cell division B.) Chemical damage of cell membranes C.) UV damage to transport proteins D.) A disconnect between the two G phases E.) Skipping the S phase in the cell cycle

Damage to genes controlling cell division

Exposing cells to a chemical or radiation that ________ can increase the rate of mutation in that population. A.) Damages DNA B.) Dissolve lipids C.) Denatures proteins D.) Cannot be metabolized

Damages DNA

Red-green colorblindness is a sex-linked recessive trait. Imagine a case of a woman with normal color vision, whose father was colorblind. She mates with a man with normal color vision. What chance do each of the children have being colorblind?

Daughter - 25% Son - 50%

A change in protein structure due to high heat is called... A.) Denaturation. B.) Saturation. C.) Synthesis. D.) Dehydration.

Denaturation.

Which of the following macromolecules is important for making up the inherited information base (genetic material) of living organisms? A.) Deoxyribonucleic acid B.) Triglyceride C.) Protein D.) Polysaccharide E.) Phospholipid

Deoxyribonucleic acid

What was the primary goal of the Human Genome Project? A.) Determine the nucleotide sequence of all DNA in a human cell B.) Determine the number of genes in a human cell C.) Determine the amino acid sequence of all proteins in a human cell D.) Determine the number of proteins in a human cell

Determine the nucleotide sequence of all DNA in a human cell

Darwin's studies of the Galapagos finches and the studies done later by Peter and Rosemary Grant of Princeton University proved that... A.) Beak size adjusted within a given year depending on individual nutrition status. B.) Differences in beak size were the result of natural selection and were inherited. C.) Beak sizes were completely random and no pattern was determined. D.) Beak sizes made no difference as to the food being eaten.

Differences in beak size were the result of natural selection and were inherited.

Which of the following processes does NOT use ATP? A.) Muscle contraction B.) Active transport of biomolecules across the cell membrane C.) Body heat production D.) Diffusion of oxygen into the blood from air in the lungs E.) Transport of biomolecules from one place to another in the cytoplasm

Diffusion of oxygen into the blood from air in the lungs

Define the terms diffusion and osmosis.

Diffusion- all molecules have a tendency to diffuse by moving from areas of high concentration to areas of low concentration (equilbrium) Osmosis- specifically the diffusion of water molecules across a membrane

Some cancer chemotherapeutic drugs disrupt microtubules from functioning properly. How would these drugs affect the cancer cells? A.) Stop enzyme production in the cell B.) Disrupt cell shape and cell reproduction C.) Prevent the storage of water in the cytosol D.) Prevent the cancer cells from eating bacteria E.) Inhibit ATP synthesis

Disrupt cell shape and cell reproduction

Some gene-specific transcription factors recruit transcriptional coactivators with histone acetylase activity in order to... A.) Condense the DNA into tightly coiled chromatin. B.) Block the translocation of RNA polymerase. C.) Disrupt the interaction between histones and the DNA. D.) Prevent the initiation of translation by the ribosome.

Disrupt the interaction between histones and the DNA.

When natural selection operates to eliminate intermediate phenotypes, it is referred to as... A.) Directional selection. B.) Disruptive selection. C.) Stabilizing or balancing selection. D.) Random chance.

Disruptive selection.

Human embryos that are missing one or that have an extra copy of an autosome are called monosomics or trisomics, respectively. These individuals generally... A.) Do not survive embryonic development. B.) Develop and grow up normally. C.) Develop characteristics of the opposite sex. D.) Develop into healthy but unusually short adults.

Do not survive embryonic development.

A testcross occurs when an individual with the _______ is crossed with an individual having the _________. A.) Homozygous dominant, heterozygous recessive B.) Dominant phenotype, dominant phenotype C.) Dominant phenotype, recessive phenotype D.) Heterozygous dominant, homozygous recessive E.) Homozygous recessive, homozygous recessive

Dominant phenotype, recessive phenotype

Mendel observed that alleles can be dominant or recessive, and the ______________ allele typically masks the expression of the __________ allele.

Dominant, other

All living things use _________ as the genetic material (the blueprint passed on from one generation to the next). A.) Single-stranded DNA B.) Double-stranded DNA C.) Single-stranded RNA D.) Double-stranded RNA

Double-stranded DNA

A geneticist finds three copies of chromosomes 21 in the karyotype of a patient, leading to the conclusion that the patient has what anomaly? A.) Patau syndrome B.) Down syndrome C.) Turner syndrome D.) Klinefelter syndrome

Down syndrome

Before any eukaryotic cell can divide into two daughter cells, it must... A.) Produce sufficient quantities of ATP. B.) Duplicate its DNA. C.) Synthesize more organelle (such as mitochondria). D.) All of the above. E.) Only A and C.

Duplicate its DNA.

A geneticist found that the gene order is ABCDEFG on one chromosome and the order is ABCDEDEFG on the other homologous chromosome. That means that there was a change in the chromosome structure called a(n) A.) Duplication. B.) Inversion. C.) Translocation. D.) Trisomy. E.) Deletion.

Duplication.

Why do animals need to breathe in air with oxygen (O2) gas?

During cellular respiration, animal cells combine oxygen with food molecules to release energy to live and function. They then release carbon dioxide. In order to carry out cellular respiration, animals need to breath in air with oxygen.

When does the duplication of DNA occur in meiosis? A.) Twice, before meiosis I and meiosis II. B.) Before meiosis II only. C.) During interphase prior to prophase I. D.) During prophase I.

During interphase prior to prophase I.

As a result of the light-dependent ("photo") reactions of photosynthesis, _________ is generated by chemiosmosis (a proton-motive force). A. amino acids B. H2O C. glucose D. chlorophyll E. ATP

E. ATP

Which of the following macromolecules can be broken down into intermediate products that enter cellular respiration to make ATP? A. Proteins B. Lipids C. Nucleic acids D. Carbohydrates E. All of the choices are correct.

E. All of the choices are correct

Which of the following can cause cancer? A. UV rays B. Viruses C. Chemicals D. Environmental contaminants E. All of the choices can cause cancer.

E. All of the choices can cause cancer

Which of the following is not produced during the Krebs cycle? A. Carbon dioxide B. NADH C. FADH2 D. ATP E. All of these are produced during the Krebs cycle.

E. All of these are produced during the Krebs cycle

Which represents the overall chemical reaction of cellular respiration? A. 6 carbon dioxide + 12 water + sunlight > glucose + 6 oxygen + 6 water B. glucose > 2 pyruvate C. glycerol + 3 fatty acids + ATP > triglyceride D. amino acids > protein E. glucose + 6 oxygen > 6 carbon dioxide + 6 water + energy (heat and ATP)

E. Glucose + 6 oxygen -> 6 carbon dioxide + 6 water + energy (ATP and heat)

What is(are) the end product(s) of the light reactions in photosynthesis? A. ATP only B. NADP+ and ATP C. NADP+ and ADP D. NADPH and ADP E. NADPH and ATP

E. NADPH and ATP

Photosynthetic pigments are embedded into the membranes of these flattened sacs within chloroplasts. A. Matrices B. Golgi Bodies C. Stroma D. Cristae E. Thylakoids

E. Thylakoids

What biology theory describes the interactions within a population and with other populations in their environment?

Ecosystems

Which of the following has the strongest resolving power? A.) Compound light microscope B.) Electron microscope C.) Magnifying glass D.) Human eye

Electron microscope

Which stage of cell respiration generates the majority of ATP?

Electron transport chain

In a multicellular organism, different tissues that function together are grouped into A.) Organisms. B.) Cells. C.) Organs. D.) Tissue systems.

Organs.

Which one of the three stages of cellular respiration generates the most ATP molecules through the production of a proton-motive force (chemiosmosis)?

Electron transport chain generates the most ATP molecules through chemiosmosis.

NADH and FADH2 molecules are oxidized in which stage of the cell respiration process?

Electron transport is the stage of the cell respiration process where NADH and FADH2 molecules are oxidized.

The ______ of atoms determine how atoms will chemically react with each other.

Electrons

________ possess both potential and kinetic energy. Potential energy is the energy of their position in different orbital levels, whereas kinetic energy is that due to their motion. A.) Protons B.) Neutrons C.) Electrons

Electrons

Much of the evolutionary history of vertebrates can be seen in the way in which their ________ develop. A.) Digest tracts B.) Embryos C.) Brains D.) Appendages

Embryos

Define the following Endergonic chemical reaction- Exergonic chemical reaction-

Endergonic chemical reaction- energy must enter in order for this reaction to occur Exergonic chemical reaction- energy exits from cellular respiration and is used to build up ATP

An extensive system of internal membranes, called the _______, is where carbohydrates and lipids are manufactured, and proteins are made that will be exported from the cell. A.) Golgi complex B.) Endoplasmic reticulum C.) Nucleolus D.) Mitochondria

Endoplasmic reticulum

In addition to being used to make RNA, ATP is used as an _____ carrier.

Energy

The Second Law of Thermodynamics states that

Energy cannot be changed from one form to another without a loss of usable energy.

The First Law of Thermodynamics states that

Energy cannot be created or destroyed, but it can be changed from one form to another.

Define the term energy:

Energy is the capacity to bring about movement against an opposing force. It is also known as the capacity to do work.

In eukaryotic cells, gene-specific transcription factors that activate transcription bind to these regulatory sites A.) Enhancers B.) Repressors C.) Operators D.) Promoters

Enhancers

Compare enzyme inhibition by competitive inhibition versus non-competitive inhibition. Which one depends on allosteric regulation?

Enzyme inhibition by competitive inhibition occurs when an inhibitor and the substrate compete for the active site of an enzyme. Product forms only when the substrate, not the inhibitor, is at the active site. Enzyme inhibition by noncompetitive inhibitor binds to the enzyme at a location other than the active site. When an inhibitor is at the site, called the allosteric site, the enzyme is inhibited because it is unable to bind to its substrate.

Proteins are often simply described as the biological macromolecules that do most of the "work" in cells. Provide one example of a general type of protein and its function to illustrate this statement?

Enzymes are a general type of protein. Their functions include maintaining metabolism and the digestion of lactose.

Cooking food to a proper temperature can kill bacteria and prevent food-borne illness. The effect of high temperature on bacteria cell life is due to the fact that... A.) Enzymes denature at high temperatures. B.) Chemical reactions slow down when the temperature rises. C.) Heat can cause an aqueous solution to become acidic. D.) Bacteria can only live at very cold temperatures.

Enzymes denature at high temperatures.

In a neutral atom, protons are always... A.) Equal to the electrons. B.) Close to the electrons. C.) Equal to the neutrons. D.) Combined with the electrons to calculate the atomic mass.

Equal to the electrons.

Which is the loosely condensed chromatin where genes are expressed? A.) Euchromatin B.) Nucleosomes C.) Heterochromatin D.) Histones

Euchromatin

Which life forms have an endomembrane system: bacteria, archaea, or eukarya?

Eukarya

Explain how the term "compartmentalization" can be applied to eukaryotic cell anatomy more so than prokaryotic cell anatomy.

Eukaryotic cells can be broken up into more "rooms" compared to the prokaryotic cell.

Darwin's main tenet of natural selection was that...

Every organism has the potential to produce more offspring than can survive and those individuals that are better suited to their environment are more likely to survive.

A doctor studying in the field of gene therapy has removed tissue from the patient, inserted normal genes into the tissue and then reinserted the tissue back into the patient's body, in an example of A.) Vectored gene therapy. B.) Xenotransplantation gene therapy. C.) Ex vivo gene therapy. D.) Translational gene therapy. E.) Transcriptional gene therapy.

Ex vivo gene therapy.

For treating children with familial hypercholesterolemia, a piece of the liver is surgically removed from the patient. A virus with the normal needed gene is used to infect the liver. Then the liver is replaced back into the patient. What type of treatment is this called? A.) Germline gene therapy. B.) Functional gene therapy. C.) Subdermal gene therapy. D.) Ex vivo gene therapy. E.) Xenotransplantation gene therapy.

Ex vivo gene therapy.

The process in which inappropriately paired nucleotides are removed and the correct bases added is called... A.) Transduction. B.) Excision repair. C.) Frame shift. D.) Back-mutation. E.) Transformation.

Excision repair.

When a cytoplasmic secretory vesicle fuses with the plasma membrane, expelling its contents outside the cell, the process is known as... A.) Diffusion B.) Exocytosis C.) Endocytosis

Exocytosis

In multi-cellular organisms, this provides the physical microenvironment in which the cells exist. It provides a substrate for cell anchorage, serves as a tissue scaffold, guides cell migration during embryonic development and wound repair, and has a key role in tissue formation. A.) Enzymes B.) Extracellular matrix C.) Gap junctions D.) Photosynthesis

Extracellular matrix

The offspring of the P generation are referred to as the... A.) True recessives. B.) Test cross. C.) F1 generation. D.) F2 generation.

F1 generation.

________ is when solute molecules move through a cell's plasma membrane from an area of greater concentration to an area of lesser concentration through a carrier protein. A.) Osmosis B.) Simple diffusion C.) Facilitated diffusion D.) Active transport

Facilitated diffusion

What is the role of general transcription factors in eukaryotic cells?

Factors assemble into initiation complex, assist in binding of RNA polymerase

Since the same DNA and genes are found in each cell in the body, all the proteins made and in each cell of the body will be the same. True or False?

False

The human proteome is the complete collection of proteins that humans produce. True or False?

False

True or False? Evolution and Natural Selection refer to the same thing.

False

When enzyme activity stops due to a buildup of end product, this control is called... A.) Feedback inhibition (negative feedback). B.) Competitive inhibition. C.) Enzyme inhibition. D.) Enzyme repression.

Feedback inhibition (negative feedback).

When enzyme activity stops rapidly due to a buildup of an end product, this control is called... A.) Feedback inhibition (negative feedback). B.) Competitive inhibition. C.) Enzyme induction. D.) Enzyme repression.

Feedback inhibition (negative feedback.)

When a person exercises strenuously and their muscle cells are not receiving enough oxygen, the cells may begin to produce lactic acid via a process called ______. A.) Fermentation B.) Aerobic respiration C.) Photosynthesis D.) Exocytosis E.) Calvin cycle

Fermentation

During the process to move an organelle inside a cell, the motor protein dynein must bind the organelle and use energy to walk along a microtubule tract. How is the energy to the dynein supplied? A.) From the endergonic reaction of ATP breakdown B.) From the exergonic reaction of ATP breakdown C.) From the exergonic reaction of ATP formation

From the exergonic reaction of ATP breakdown

The punctuated equilibrium model of speciation predicts which of the following? A.) Few if any transitional species will be found in the fossil record. B.) Evolution proceeds slowly and gradually over time. C.) The environment has always changed slowly over time. D.) Species adapt gradually to changes throughout time. E.) Transitional species should be the most abundant fossils found.

Few if any transitional species will be found in the fossil record.

What essential role does water play in photosynthesis?

First electron donor Replenishes chlorophyll Forms an electron train Replaces electrons lost in

In what way is burning gasoline in your car similar to your eating foods rich in fats and carbohydrates?

Food - oxidizes lipids Gasoline - oxidizes fossil fuel When you burn gas, Carbon Dioxide, water and heat/energy are released. This is similar to eating food rich in fats and carbs because you are oxidizing and releasing energy/heat. They are identical chemical reactions. A very controlled, slow burn.

In what ways is burning gasoline in your car similar to your eating foods rich in fats and carbohydrates?

Food- oxidizes lipids Gasoline- oxidizes fossil fuel

The property that you named above (4 covalent bonds) gives carbon a lot of versatility. Provide some examples of this versatility.

Form single, double, and triple bonds Form chains, branched chains, and rings when connected to other carbon atoms

Phospholipids function as...

Form the membrane that enclose cells

When only a very few individuals give rise to a new population in a new area of favorable habitat, the new population might possess different genotype frequencies for a certain trait than the larger population from which it separated. This effect is called the __________ effect.

Founder

What are the goals of functional genomics?

Functional genomics wants to know how the many variant forms of genes on a chromosome affect the phenotype.

Normal cells may enter this stage of the cell cycle, but cancer cells typically do not. A.) G0 B.) G1 C.) G2 D.) S E.) M

G0

In what phase of the cell cycle would cells be synthesizing proteins, lipids, and carbohydrates, as well as doubling their numbers of organelles? A.) S B.) G1 C.) Cytokinesis D.) G0 E.) Mitosis

G1

State the major events that occur in each of the following stages of the eukaryotic cell cycle G1- S- G2- M (mitosis)- C (cytokinesis)-

G1- Cell recovers from previous cell division S- DNA synthesis (Each chromosome is now composed of two sister chromatids) G2- Preparation for division M (mitosis)- Division of nucleus (Daughter cells have the same number of chromosomes as parent cells.) C (cytokinesis)- Division of cytoplasm

State the major events that occur in each of the following stages of the eukaryotic cell cycle: G1 S G2 M (mitosis) C (cytokinesis)

G1: Cell recovers from previous cell division S: DNA synthesis (each chromosome is not composed of two sister chromatids) G2: Preparation for division M (mitosis): Division of nucleus (Daughter cells have the same number of chromosomes as parent cells) C (cytokinesis): Division of cytoplasm

A cell placed in a hypotonic solution will... A.) Neither gain nor lose water B.) Gain water. C.) Lose water.

Gain water.

In sexual reproduction, haploid __________ will fuse to form a new diploid called a zygote. A.) Gametes B.) Bacteria C.) Chromosomes D.) Somatic cells

Gametes

Each ______ is a specific segment of the DNA with the code for production of one functional product. A.) Intron B.) Exon C.) Gene D.) Operator

Gene

Making identical copies of the same gene is referred to as... A.) Proteome cloning B.) Polymorphic cloning C.) Genome cloning D.) Gene cloning E.) Plasmid cloning

Gene cloning

The use of DNA information to direct the production of a protein product is called... A.) Transcription. B.) Translation. C.) Gene expression. D.) Replication.

Gene expression.

A lone wolf that has been rejected by its pack joins another pack. What has occurred? A.) Directional selection B.) A bottleneck selection C.) Stabilizing selection D.) Gene flow E.) Genetic drift

Gene flow

The _____________ is the total alleles of all the genes in all the members of a population. A.) Genetic drift B.) Gene flow C.) Gene pool D.) Gene migration E.) Gene selection

Gene pool

Define the terms: gene and allele.

Gene- Unit of hereditary existing as alleles on the chromosomes; in diploid organisms, typically two alleles are inherited-one from each parent. Allele- Alternative form of a gene; alleles occur at the same locus on homologues.

Which of the following statements about how genotype determines phenotype is correct? A.) Proteins determine the sequence of genes. B.) Amino acids are located on chromosomes and determine what genes will be expressed. C.) Associations of proteins from polypeptides, which contain genes. D.) Genes encode a sequence of amino acids for a particular polypeptide or protein. E.) Genes are located on proteins, which encode a sequence of amino acids.

Genes encode a sequence of amino acids for a particular polypeptide or protein.

Molecular genetics predicts that the longer two species have been evolving separately on their own, the greater the number of ________ that will accumulate between them. A.) Genetic differences B.) Genetic similarities C.) Convergences D.) Homologous structures

Genetic differences

A biologist is study changes in allele frequency of a gene pool due to chance rather than selection by the environment, called what? A.) Disruptive drift B.) Heterozygous drift C.) Stabilizing drift D.) Directional drift E.) Genetic drift

Genetic drift

Population bottlenecks can lead to major evolutionary events because... A.) Genetic drift has the greatest effects on very small populations. B.) Small populations are more successful at breeding than large populations. C.) Small populations are better able to resist a harsh and changing environment. D.) Natural selection only works on small populations.

Genetic drift has the greatest effects on very small populations.

In small populations, allele frequencies can change drastically by chance alone. This phenomenon is called... A.) Migration B.) Density-independent effects. C.) Density-dependent effects. D.) Genetic drift.

Genetic drift.

A friend asks you a question, "What's genetic engineering?" Your answer is,... A.) Genetic engineering is the ability of scientists to remove a nucleus from a cell and place it into another cell. B.) Genetic engineering is taking proteins from one organism and placing them into another organism of a different species. C.) Genetic engineering is moving genes from one organism to another. D.) Genetic engineering is removing structural components, for example, a lysosome, from one part of a cell and placing those structures elsewhere in that cell.

Genetic engineering is moving genes from one organism to another.

The particular alleles that an individual has are the individual's ______________, whereas the appearance of an individual is called its _____________.

Genotype, phenotype

A scientist wants to study the effect of vitamin C on colds. He recruits 100 people with colds and gives the experimental group 1000 mg of vitamin C per day. What would be an appropriate control? A.) Give the control group nothing. B.) Give the control group 2000 mg of vitamin C per day. C.) Give the control group orange juice every day. D.) Give the control group a pill similar to vitamin C but containing sugar (a placebo). E.) Give the control group 1000 mg of another brand of vitamin C per day.

Give the control group a pill similar to vitamin C but containing sugar (a placebo).

The main products of photosynthesis are...

Glucose (simple sugar) and oxygen gas

The carbon dioxide we exhale results from the oxidation of... A.) Oxygen B.) Water C.) Coenzyme A. D.) Glucose and other organic molecules. E.) NADH and FADH2

Glucose and other organic molecules.

In bacteria, the lac operon is maximally expressed when... A.) Glucose is high and lactose is present. B.) Glucose is high and lactose is absent. C.) Glucose is low and lactose is present. D.) Glucose is low and lactose is absent. E.) Glucose is low, regardless of the presence or absence of lactose.

Glucose is low and lactose is present.

What organic molecules can be created in plants from G3P, the product of the Calvin cycle?

Glucose phosphate (starch, cellulose), fatty acid synthase, and amino acid synthase are organic molecules that can be created from G3P.

C6H12O6 is the chemical formula of what monosaccharide?

Glucose, galactose, and mannose

In the overall reactions for cellular respiration, _______ is oxidized, and ________ is reduced. A.) Glucose, water B.) Oxygen, glucose C.) Oxygen, water D.) Glucose, carbon dioxide E.) Glucose, oxygen

Glucose, oxygen

A highly branched polysaccharide of glucose that animals (e.g. humans) use to store energy is... A.) Starch. B.) Cellulose. C.) Chitin. D.) Glycogen.

Glycogen.

All of the following are lipids except... A.) Waxes. B.) Cholesterol. C.) Triglycerides. D.) Phospholipids. E.) Glycogen.

Glycogen.

Because of its use in all living organisms, _______ is thought to have been one of the earliest metabolic pathways developed by life, although the process may have taken millions of years to evolve completely. A.) Glycolysis B.) Photosynthesis C.) Aerobic respiration D.) Fermentation

Glycolysis

The first stage of cellular respiration, called _______, takes place in the cytoplasm of the cell and needs no oxygen. A.) Glycolysis B.) Krebs cycle C.) Photorespiration D.) Oxidation

Glycolysis

The order of the major pathways and reactions of cellular respiration is A.) Preparatory reaction - glycolysis - electron transport chain - Krebs cycle. B.) Electron transport chain - glycolysis - preparatory reaction - Krebs cycle. C.) Glycolysis - electron transport chain - preparatory reaction - Krebs cycle. D.) Krebs cycle - glycolysis - preparatory reaction - electron transport chain. E.) Glycolysis - preparatory reaction - Krebs cycle - electron transport chain.

Glycolysis - preparatory reaction - Krebs cycle - electron transport chain.

Select the correct sequence depicting the breakdown of glucose during cellular respiration. A.) Glycolysis - pyruvate - acetyl - CoA - electron transport chain - Krebs cycle B.) Glycolysis - pyruvate - acetyl - CoA - Krebs cycle - electron transport chain C.) Glycolysis - acetyl - CoA - pyruvate - electron transport chain - Krebs cycle D.) Glycolysis - acetyl - CoA - pyruvate - Krebs cycle - electron transport chain

Glycolysis - pyruvate - acetyl - CoA - electron transport chain - Krebs cycle

What would be the immediate result if the hydrogen ion concentration in the intermembrane space and the matrix reached equilibrium? A.) The conversion of NAD+ and NADH would stop. B.) Most ATP production would stop. C.) The conversion of FAD and FADH2 would stop. D.) Glycolysis and the Krebs cycle would stop. E.) Most ATP production would increase.

Glycolysis and the Krebs cycle would stop.

Does glycolysis require oxygen gas?

Glycolysis does not require oxygen.

Which of the following statements accurately reflects the process of glycolysis? A.) Glycolysis uses molecular oxygen, however it occurs in anaerobic environments. B.) Glycolysis uses molecular oxygen and occurs in aerobic environments. C.) Glycolysis uses no molecular oxygen and occurs in the mitochondria. D.) Glycolysis uses no molecular oxygen, however it can occur in aerobic environments.

Glycolysis uses no molecular oxygen; however, it can occur in aerobic environments.

What is the advantage in genetically engineering plants to be resistant to glyphosate (the active ingredient in Roundup)? A.) Glyphosate can then be applied and only kill pests on some plants. B.) The genetically engineered plants will then be resistant to the glyphosate virus. C.) Glyphosate will then only be able to enhance the growth of the genetically engineered plants. D.) Glyphosate can then be applied and stop the growth of unwanted plants, like weeds.

Glyphosate can then be applied and stop the growth of unwanted plants, like weeds.

Given that a cell's structure will often reflect its function, which of the following organelles would you predict to be larger or more numerous in cells whose main function is to secrete large amounts of enzymes out of the cell and into the stomach lumen for digesting food? A.) Nucleus B.) Cytoskeleton C.) Chloroplasts D.) Golgi apparatus

Golgi apparatus

Lysosomes are produced from this organelle A.) Nucleus B.) Rough endoplasmic reticulum C.) Smooth endoplasmic reticulum D.) Mitochondria E.) Golgi apparatus

Golgi apparatus

______ function in the collection, packaging, and distribution of molecules made in the cell. A.) Mitochondria B.) Ribosomes C.) Golgi bodies D.) Vesicles

Golgi bodies

Many protists are __________ for most of the their lives. A.) Diploid B.) Germ-line C.) Haploid D.) Cross over

Haploid

In humans, gametes are ______ while somatic cells are ________. A.) Diploid / diploid B.) Haploid / diploid C.) Haploid / haploid D.) Haploid / triploid E.) Triploid / haploid

Haploid / diploid

The second meiotic division is essentially a mitotic division and the cells produced are... A.) Somatic. B.) Diploid. C.) Haploid. D.) Autosomal. E.) Polyploidy.

Haploid.

Which is incorrect about inducible operons? A.) Have genes turned off by a buildup of end product B.) Are often for catabolic pathways C.) Are normally turned off D.) Are turned on by the substrate of the enzyme E.) Include the lac operon

Have genes turned off by a buildup of end product

The sickle-cell allele is maintained in African populations even though when two sickle-cell alleles are inherited it can be fatal because A.) Having two sickle-cell alleles makes the person immune to malaria. B.) Having two non-sickle-cell alleles makes a person immune to malaria. C.) Having a non-sickle-cell allele and a sickle-cell allele makes the person resistant to malaria. D.) The sickle-cell allele is dominant.

Having a non-sickle-cell allele and a sickle-cell allele makes the person resistant to malaria.

During DNA replication, which enzyme is responsible for unzipping and unwinding the original molecule of DNA? A.) DNA polymerase B.) DNA ligase C.) Helicase D.) DNA primer

Helicase

All of the following are correct about DNA replication except A.) DNA polymerase is the enzyme that adds nucleotides to one another. B.) DNA polymerase can only synthesize DNA in the 5' to 3' direction. C.) Helicase is the enzyme which rewinds the DNA after replication is finished. D.) DNA polymerase can only add nucleotides to an existing strand. E.) The lagging strand has many primers.

Helicase is the enzyme which rewinds the DNA after replication is finished.

Two true-breeding pea plants are crossed, one with purple flowers and the other with white. Their offspring are... A.) Heterozygous with the dominant phenotype. B.) Homozygous with the recessive phenotype. C.) A mixture of purple and white flowers with varying genotypes. D.) Still true-breeding, with the same parental genotypes.

Heterozygous with the dominant phenotype.

In eukaryotes, DNA within chromosomes is wrapped around proteins with positive charges called... A.) Chromatids. B.) Histones. C.) Centromeres. D.) Kinetochores.

Histones.

All living things, whether they are unicellular or complex multicellular organisms, are able to maintain stable internal conditions. This property is called...

Homeostasis

During the anaphase stage of mitosis, sister chromatids separate and become the daughter chromosomes. What happens during anaphase I of meiosis? A.) Sister chromatids separate. B.) Chromosomes condense. C.) Homologous chromosomes separate. D.) The nuclear membrane disappears. E.) Recombination occurs.

Homologous chromosomes separate.

Pairs of chromosomes that have the same size, shape, and function are ___________.

Homologous pairs

Pairs of chromosomes that have the same size, shape, and function are called ________________.

Homologous pairs

Over time, the same bones in different vertebrates were put to different uses (example- the bones in a bat wing, human arm, and porpoise fin.) This falls under the category of... A.) Heterologous structures. B.) Vestigial organs. C.) Analogous structures. D.) Homologous structures.

Homologous structures.

Structures that are anatomically similar because they are inherited from a recent common ancestor are called A.) Vestigial structures. B.) Analogous structures. C.) Transitional structures. D.) Homologous structures.

Homologous structures.

A diploid individual contains two alleles, for each gene. If the two alleles are the same, the individual is said to be ______________; if the two alleles are different, the individual is said to be _____________.

Homozygous, heterozygous

If you were studying the gene which controls the number of appearance of repeating structures along the main body axes in vertebrates, you would be studying which of the following genes? A.) Tbx5 B.) Evo9 C.) Pax6 D.) Hox E.) Pax2

Hox

If the human gene that encodes for the protein insulin is transferred into a bacteria cell's chromosome with a proper bacterial promoter, the transgenic bacteria would produce... A.) Human insulin. B.) A protein with a different amino acid sequence than human insulin. C.) Either human insulin protein or a different protein depending on the type of bacteria. D.) Nothing from that gene since bacteria do not use DNA.

Human insulin.

Which of the following disorders is a result of an autosomal dominant allele? A.) Sickle-cell disease B.) Huntington disease C.) Tay-Sachs disease D.) Influenza E.) Cystic fibrosis

Huntington disease

Base pairs in double-stranded nucleic acids are held together by ______ bonds. A.) Peptide B.) Nonpolar covalent C.) Polar covalent D.) Hydrogen

Hydrogen

In addition to electrons, which of the following is also involved in electron transfer in redox reactions? A.) ADP B.) Glucose C.) Carbon D.) Hydrogen

Hydrogen

This type of bond is responsible for holding the two strands of a DNA double helix together. A.) Covalent B.) Ionic C.) Hydrogen D.) Van der Waal

Hydrogen

In a DNA molecule, the polar "G" nucleotide on one strand always bonds with the polar "C" nucleotide on the other strand. What kind of bonds are these? A.) Hydrophobic interactions B.) Hydrogen bonds C.) Ionic bonds D.) Covalent bonds

Hydrogen bonds

Which of the following functional groups, when added to a carbon chain, will transform it into an alcohol molecule? A.) Phosphate B.) Hydroxyl C.) Carbonyl D.) Carboxyl E.) Sulfhydryl

Hydroxyl

In science, a possible explanation for a natural event is known as a(n) _______. It is based on existing knowledge, so it is an informed notion, but it has yet to be tested or fully supported.

Hypothesis

In the light-dependent reactions in plants, electrons flow from photosystem __________ to photosystem _________.

I, II

A woman who can roll her tongue (presumably dominant) is married to a man who cannot. Two of their four children can roll their tongues and two cannot. What is the genotype of the parents?

If A = roll tongue and a = cannot roll tongue, then the woman would be Aa and the man would be aa.

What is the significance of the G1 check point? A.) If the cell doesn't pass the G1 checkpoint, it will specialize B.) If the cell doesn't pass the G1 checkpoint, it will become cancer. C.) If the cell passes the G1 checkpoint, it will become cancer. D.) If the cell passes the G1 checkpoint, it is committed to divide. E.) IF the cell doesn't pass the G1 checkpoint, it is committed to divide.

If the cell passed the G1 checkpoint, it is committed to divide.

Which of the following is NOT a feature of the scientific method? A.) Formulating a hypothesis B.) Devising a testable prediction C.) Conducting a careful experiment D.) Analyzing experimental data E.) Ignoring empirical evidence

Ignoring empirical evidence

Explain the difference between artificial selection and natural selection. Be able to give examples of each.

In artificial selection, a breeder selects for the desired characteristics. In natural selection, conditions in nature determine which kinds of individuals in a population are the most fit.

In eukaryotic cells, what controls where translation will initiate on a mRNA?

In eukaryotic and prokaryotic, the translation always initiates with the amino acid methionine, usually encoded by AUG. Ribosome binds to 5' cap until AUG is encountered where initiation begins

In a new treatment of skin cancer, viruses are used to deliver the p53 gene directly into cancer cells, which then causes apoptosis of these infected cells. What type of treatment is this? A.) Xenotransplantation gene therapy B.) Transcriptional gene therapy. C.) In vivo gene therapy. D.) Translated gene therapy. E.) Ex vivo gene therapy.

In vivo gene therapy.

In the presence of lactose, the Lac repressor is... A.) Active and can bind to the operator. B.) Active and cannot bind to the operator. C.) Inactive and can bind to the operator. D.) Inactive and cannot bind to the operator.

Inactive and cannot bind to the operator.

Which of the following is incorrect about termination codons? A.) Where the bond between the final tRNA and the finished polypeptide chain is broken B.) Include AUG C.) Include UAA, UAG, and UGA D.) Do not have corresponding tRNA E.) Can also be called nonsense codons

Include AUG

In snapdragons, pink-flowered plants are produced when red-flowered plants are crossed with white-flowered plants. This type of inheritance can best be described as... A.) Simple dominant and recessive traits. B.) Codominance. C.) Incomplete dominance. D.) Sex-linked traits.

Incomplete dominance.

Which of the following changes would cause an atom to become a different element? A.) Increase the number of neutrons B.) Increase the number of electrons C.) Increase the number of protons D.) Decrease the number of neutrons E.) Decrease the number of electrons

Increase the number of protons

Which of the following would decrease one's risk of getting cancer? A.) Increased exposure to radiation B.) Increased consumption of antioxidants C.) Increased frequency of smoking D.) Increased body weight E.) Increased consumption of alcohol

Increased consumption of antioxidants

What role do transcription factors play in the positive control of eukaryotic gene expression? A.) Increasing RNA polymerase binding to a promoter to start transcription B.) Binding to an operator to prevent transcription C.) Forming heterochromatin to prevent transcription D.) Binding to a promotor of a gene to start translation E.) Binding to a repressor site to prevent transcription

Increasing RNA polymerase binding to a promoter to start transcription

What is inductive reasoning? When is it used in science?

Inductive reasoning are isolated facts combined into a cohesive whole to think creatively. It is used in science to develop your hypothesis.

Compare and contrast the initiation of transcription and translation. Compare and contrast the termination of transcription and translation.

Initiation of Transcription- Occurs when RNA polymerase protein binds to the promoter in DNA and forms a transcription initiation complex. Promotor directs the exact location for the initiation of transcription. Initiation of Translation- Occurs when ribosome subunits, initiation factors and t-RNA bind to mRNA near the AUG start codon. Termination of Transcription- RNA transcript is released, and polymerase detaches from DNA. DNA rewinds itself into a double-helix and is unaltered throughout this process. Termination of Translation- When the ribosome encounters one of the three stop codons, it disassembles the ribosome and releases the polypeptide.

The step which brings all the translation elements together is referred to as.... A.) Elongation. B.) Initiation. C.) Transcription. D.) Translocation. E.) Replication.

Initiation.

Crop plants are made resistant to insect pests by... A.) Injecting mRNA for a protein into the root of the plant. B.) Inserting the Bt protein gene into the plant. C.) Coating the plant with a gel that acts as a barrier. D.) All of these are correct

Inserting the Bt protein gene into the plant.

A cell spends most of its time in which of the following phases? A.) G2 B.) Interphase C.) G1 D.) Mitosis E.) S

Interphase

The G1, S, and G2 phases of the cell cycle are collectively known as ___________.

Interphase

The G1, S, and G2 phases of the cell cycle are collectively known as _________________.

Interphase

Which of the following is NOT a stage of mitosis? A.) Interphase B.) Prophase C.) Anaphase D.) Metaphase E.) Telophase

Interphase

The DNA sequence of a gene that do not code for amino acids in a protein are called... A.) Promoters. B.) Introns. C.) Transposons. D.) Inducers. E.) Exons.

Introns

When an electron is transferred from one atom to the next, and the two atoms are then electrically attracted to one another, the type of bond is a(n) _______ bond. A.) Hydrogen B.) Covalent C.) Kinetic D.) Ionic

Ionic

What type of chemical bond holds the atoms of sodium and chloride together in crystals of table salt?

Ionic bond

When salt crystals dissolve in water, which chemical interactions in the salt crystals are being dissociated by the water molecules? A.) Nonpolar covalent bonds B.) Hydrogen bonds C.) Polar covalent bonds D.) Ionic bonds

Ionic bonds

Atoms that bear a positive or negative charge are known as...

Ions

Which of the following elements is important in smaller quantities than is true for the six major elements in living things? A.) Carbon B.) Iron C.) Oxygen D.) Nitrogen E.) Hydrogen

Iron

The lactose repressor protein... A.) Is transcribed with the structural lac genes. B.) Is activated by binding lactose. C.) Is inactivated by binding lactose. D.) Requires lactose for its transcription.

Is activated by binding lactose.

The pairing of homologous chromosomes all along their lengths... A.) Occurs during anaphase II. B.) Occurs during telophase I. C.) Is called crossing over. D.) Is called synapsis.

Is called synapsis.

Glucose, galactose, and mannose are all ________ of one another (they all have the same molecular formula C6H12O6).

Isonomers

The term normal saline is commonly used to refer to an aqueous solution of 0.9% NaCl. It is also referred to as _______ saline because it closely approximates the level of tonicity in cells. A.) Hypotonic B.) Hypertonic C.) Isotonic

Isotonic

What is the benefit of converting pyruvate to lactate when oxygen is not available? A.) It allows the electron transport chain to continue. B.) It allows chemiosmosis to continue. C.) It allows the electrons transport chain to produce oxygen. D.) It allows substrate-level ATP synthesis to continue. E.) It allows the Krebs cycle to produce oxygen.

It allows substrate-level ATP synthesis to continue.

Which of the following statements about differential gene expression is true? A.) It would have unchanged the size of a particular structure. B.) It might maintain the function of an organ. C.) It could identically color skin. D.) It might change leg length between species.

It might change leg length between species.

What is the function of the translation repressor protein? A.) It binds to a mRNA and prevents translation from that mRNA. B.) It binds to a proteasome and prevents translation. C.) It binds to a master developmental regulatory gene and prevents translation. D.) It binds to a gene on the DNA and prevents translation. E.) It binds to amino acids and prevents them from forming the protein chain.

It binds to a mRNA and prevents translation from that mRNA.

What is the significance of the stroma of the chloroplast? A.) It contains metabolic enzymes used to reduce carbon dioxide to carbohydrate. B.) It is where the chlorophyll is stored. C.) It is where oxygen leaves the plant. D.) It helps absorb water. E.) It is where carbon dioxide enters the leaf.

It contains metabolic enzymes used to reduce carbon dioxide to carbohydrate.

Hydrochloric acid is classified as an acid because A.) It absorbs excess hydroxide ions from the solution. B.) It dissociates to release hydroxide ions. C.) It absorbs excess hydrogen ions from the solution. D.) It dissociates to release hydrogen ions.

It dissociates to release hydrogen ions.

Which is incorrect about fermentation? A.) It is anaerobic. B.) It generates oxygen. C.) In yeast, ethanol is produced. D.) When human muscles are lacking oxygen, lactate is produced. E.) It recycles NADH back to NAD+.

It generates oxygen.

What property does carbon have that makes it so central to biological chemistry?

It has 6 electrons, 4 of which are in its outer shell. So, it can form four covalent bonds.

If a molecule has been reduced, which of the following statements must be true? A.) It has lost electrons. B.) The number of electrons in the molecule has stayed the same. C.) It has lost oxygen. D.) It has gained electrons. E.) It has gained oxygen.

It has gained electrons.

Which of the following is least an obstacle to cloning animals? A.) There are ethical concerns associated with it. B.) It has a high failure rate. C.) It increases the genetic variability of a species. D.) It is very expensive. E.) Mutation rate is higher in clones.

It increases the genetic variability of a species.

A plant with yellow flowers was crossed with a plant with red flowers. All of the numerous offspring had red flowers. What does this show about the genotype of the parental plant with yellow flowers? A.) It is homozygous dominant. B.) It is homozygous dominant or heterozygous. C.) It is heterozygous dominant. D.) It is homozygous recessive or heterozygous. E.) It is homozygous recessive.

It is homozygous recessive.

Which of the following statements is least accurate about modern corn? A.) It comes from a wild grass called teosinte. B.) It is little changed from teosinte. C.) It is America's number one field crop. D.) It is an allotetraploid. E.) Its artificial selection began between 4,000 and 3,000 B.C.

It is little changed from teosinte.

Hemophilia is a group of different inherited blood-clotting disorders. Which is true about hemophilia? A.) It causes sterility. B.) It is more common in females than in males. C.) It is more common in males than in females. D.) It is inherited only on the autosomes.

It is more common in males than in females.

What happens to pyruvate, the end product of glycolysis, before it enters the Krebs cycle? A.) It is reduced. B.) It is phosphorylated. C.) It is carboxylated. D.) It is oxidized.

It is oxidized.

Which organelle acts as the powerhouse of the cell? A.) Mitochondria B.) Nucleus C.) Rough endoplasmic reticulum D.) Lysosome

Mitochondria

Which of the following statements is true about fermentation? A.) It produces more ATP then cellular respiration with oxygen. B.) It doesn't need enzymes. C.) It occurs without oxygen. D.) It produces pyruvate as its end product. E.) It is very efficient.

It occurs without oxygen.

Histone acetylation can disrupt the interaction between adjacent nucleosomes because A.) Acetylated nucleosomes are targeted for degradation. B.) It prevents histones from interacting tightly with one another and with the DNA. C.) It prevents histone H1 from forming a bridge between itself and the centromere. D.) Acetylated histones lead to condensation of the DNA.

It prevents histones from interacting tightly with one another and with the DNA.

Which of the following is true of a protein which has been denatured by high heat? A.) It can be reformed by changing the pH. B.) It will no longer be able to perform its normal function. C.) It will still work the same as before the denaturation process. D.) It can be reformed by further heating. E.) It can be reformed by cooling it back to room temperature.

It will no longer be able to perform its normal function.

Which scientists are credited for deducing the true double helix structure of DNA in cells? A.) Maurice Wilkins and Rosalind Franklin B.) Martha Chase and Frederick Griffith C.) James Watson and Francis Crick D.) Rachel Carson and Oswald Avery

James Watson and Francis Crick

Gametes and spores... A.) Are both produced by animals. B.) Are autosomal. C.) Are haploid. D.) Are diploid. E.) Join in fertilization.

Join in fertilization.

Objects that are moving are said to possess ________ energy.

Kinetic

The second stage of cellular respiration, called the _________, takes place in the mitochondria and generates NADH and FADH2.

Krebs cycle

The second stage of cellular respiration, called the ______________________, takes place in the mitochondria and generates NADH and FADH2.

Krebs cycle

During which stage(s) of cellular respiration is CO2 produced? A.) Glycolysis and Krebs cycle B.) Krebs cycle only C.) Electron transport chain D.) Oxidation of pyruvate to acetyl-CoA and Krebs cycle E.) Krebs cycle and electron transport chain

Krebs cycle only

Some common end products of fermentation include ________ and __________.

Lactate and ethanol

Some common end products of fermentation include ___________ and ____________.

Lactate and ethanol

Yogurt is a food produced by fermentation of milk. Fermentation of lactose by bacteria produces __________, which acts on milk proteins to give yogurt its texture and characteristic tang. A.) Sulfate B.) Propane C.) Glucose D.) Lactic acid E.) Water

Lactic acid

All of the following are monosaccharides EXCEPT: A.) Lactose B.) Galactose C.) Deoxyribose D.) Glucose

Lactose

If pairs of factors separate independently of other pairs of factors, you are dealing with the... A.) Law of independent assortment B.) Law of codominance C.) Law of dominance D.) Law of segregation E.) Law of multiple allele

Law of independent assortment

In guinea pigs, B = black, b = brown, S = short hair, s= long hair. If a BbSs animal mated with a BbSs animal, the offspring were: 9 black short hair, 3 black long hair, 3 brown short hair, 1 brown long hair. Mendel's results from these sorts of dihybrid crosses supported his... A.) Law of independent assortment. B.) Law of segregation. C.) Law of natural selection. D.) Law of gene linkage.

Law of independent assortment.

The ________ strand is synthesized continuously while the ________ is synthesized in a series of short fragments called Okazaki fragments. A.) Original; primary B.) Lagging; leading C.) Leading; lagging D.) Primary; secondary

Leading; lagging

Water is split and oxygen is released during which process in photosynthesis? A.) Electron Transport Chain B.) Cyclic Electron Pathway C.) Calvin Cycle D.) Light Reactions E.) ATP synthesis

Light Reactions

During the light reactions of photosynthesis, ATP molecules are generated by...

Light energy is converted into electrical energy which is converted into ATP.

After hydrogens and electrons are stripped from NADPH, the regenerated NADP+ can be used again as a reactant in the... A.) Antenna complex. B.) Calvin cycle. C.) Krebs cycle. D.) Light-dependent reactions. E.) Light-independent reactions.

Light-dependent reactions.

Because genes close together on a chromosome tend to be inherited together, they are said to form a... A.) Pleiotropic association. B.) Linkage group. C.) Genotype association. D.) Gene locus. E.) Polygenic group.

Linkage group.

What is the composition of a cell membrane? A.) Lipids, proteins, glycolipids, and cholesterol B.) An oil film of lipids only C.) Cellulose, proteins, and cholesterol D.) A network of cellulose and glycolipids

Lipids, proteins, glycolipids, and cholesterol

The best fat burning exercise would be... A.) Short and aerobic. B.) Short and anaerobic. C.) Long and aerobic. D.) Long and anaerobic.

Long and aerobic.

A cell placed in a hypertonic solution will... A.) Neither gain nor lose water. B.) Gain water. C.) Lose water.

Lose water.

Enzymes are biological catalysts because they function to... A.) Lower activation energy. B.) Raise activation energy. C.) Supply activation energy. D.) Supply the reactants.

Lower activation energy.

Organelles called ______ contain enzymes that digest worn out parts of cells. A.) Golgi bodies B.) Lysosomes C.) Vacuoles D.) Secretory vesicles

Lysosomes

Where inside an animal cell would engulfed bacteria, debris, and macromolecules be broken down? A.) Rough endoplasmic reticulum B.) Lysosomes C.) Peroxisomes D.) Smooth endoplasmic reticulum E.) Mitochondria

Lysosomes

Describe 3 functions of the cell's cytoskeleton

Maintenance of cell shape, hold organelles, and assist in moving materials in and out of the cell are three functions of the cell's cytoskeleton.

Enzymes function to... A.) Add structural support. B.) Provide defense. C.) Transport molecules. D.) Make chemical reactions more likely to occur.

Make chemical reactions more likely to occur.

What is the value of the G1, M, and G2 checkpoints in the cell cyle?

Make sure the cell is ready. If not, it'll go back through the cycle. Stop abnormal cells from growing. The cell cycle has checkpoints that delay the cycle until all is well.

Which of the following issues would be least likely to be solved by applying the scientific method? A.) Determing the most effective safety products for use in contact sports B.) Evaluating the relationship between violent video games and criminal behavior. C.) Making public policy on whether to legalize same-sex marriage D.) Comparing the effectiveness of two different potential cancer drugs E.) Developing more effective standardized tests (ACT or SAT)

Making public policy on whether to legalize same-sex marriage

Considering the major functions of the 4 types of biological macromolecules (carbohydrates, proteins, lipids, and nucleic acids), which of the following is NOT a function of the carbohydrates in living organisms? A.) Energy source B.) Energy storage C.) Genetic inheritance D.) Structural framework for parts of the cell E.) Making up cell walls

Making up cell walls

Whose DNA was used for the Human Genome Project? A.) Males from Asia and the Americas B.) Females from North and South America C.) Males and females from Europe, Africa, Asia, Americas D.) Males from Europe and Africa E.) Males from Europe, Africa, Asia, the Americas

Males and females from Europe, Africa, Asia, Americas

All of the following are correct about cancer except A.) Malignant tumors are invasive. B.) Malignant tumors can produce metastases. C.) Cancer cells can sometimes spread into the bloodstream. D.) An oncogene is a cancer cell. E.) Cancers are the direct result of mutations in growth-regulating genes.

Malignant tumors can produce metastases.

For good heart health, you should avoid saturated fats which are primarily found in________. A.) Fungi B.) Fish C.) Mammals D.) Plant oils

Mammals

Genes which are involved in pattern formation and organization of body parts are referred to as A.) Regulatory regions of structural genes B.) Master developmental regulatory genes C.) Developmental enhancer genes D.) Nucleosome regulatory genes

Master developmental regulatory genes

The Hardy-Weinberg equilibrium is based on a number of assumptions. Which of the following is NOT one of those? A.) Mating occurs non-randomly, and mates exert preferences. B.) The size of the population is large. C.) There is no migration. D.) There are no mutations.

Mating occurs non-randomly, and mates exert preferences.

During which stage of meiosis does crossing over between homologous chromosomes occur?

Meiosis (Prophase I)

What are three key differences between meiosis and mitosis?

Meiosis requires two nuclear divisions, but mitosis requires only one nuclear division. Meiosis results in four haploid daughter cells. Mitosis results in two diploid daughter cells. Following meiosis, the daughter cells are genetically dissimilar to each other and to the parent cell. Following mitosis, the daughter cells are generally identical to each other and to the parent cell.

Where is the process of meiosis happening in the human body?

Meiosis takes place in the ovaries and the testes (the sex cells) in the human body.

Genetic variation in sexual organisms is produced by... A.) Mitosis. B.) Meiosis.

Meiosis.

The process that serves to decrease cellular chromosome number by half is... A.) Mitosis. B.) Meiosis. C.) Binary fission. D.) Both mitosis and meiosis decrease chromosome number by half. E.) Mitosis, meiosis, and binary fission all decrease chromosome number by half.

Meiosis.

The RNA molecules that carry amino acids to the ribosomes during protein synthesis are called... A.) Ribosomal RNA. B.) Messenger RNA. C.) Transfer RNA. D.) Primer RNA. E.) Ribozymes.

Messenger RNA.

Define the following terms Metabolism- Catabolism- Anabolism-

Metabolism- defined as the sum of all the chemical reactions in an organism Catabolism- energy-releasing process (exergonic reactions) Anabolism- energy-using process (endergonic reaction)

During mitosis, chromosomes are lined up at the equator of the cell during the phase called ...

Metaphase

During mitosis, chromosomes are lined up at the equator of the cell during the phase called...

Metaphase

Independent assortment of homologous chromosomes occurs at... A.) Prophase I. B.) Metaphase I. C.) Anaphase II. D.) Metaphase II.

Metaphase I.

Which component of the cytoskeleton controls amoeboid movement-the crawling of cells along a surface?

Microfilaments (actin filament)

Which technique would a biologist use to insert genes into eggs by hand? A.) DNA ligase insertion B.) Restriction enzyme insertion C.) Vortex mixing D.) Polymerase chain reaction E.) Microinjection

Microinjection

Which cytoskeletal fibers are found in the internal structure of the cilia and flagella? A.) Keratin filaments B.) Actin filaments C.) Microtubules D.) Myosin E.) Intermediate filaments

Microtubules

___________ causes evolutionary change when individuals move between populations and affect the genetic composition of the receiving population. A.) Migration B.) Death C.) Vegetation D.) Stagnation

Migration

What type of mutation chances the base of a single DNA nucleotide, which then causes a chance in a specific codon?

Missense mutation

Cyanide directly poisons _________ thereby preventing production of _____ molecules. A.) Nuclei / DNA B.) Mitochondria / ATP C.) Ribosomes / protein D.) Lysosomes / sugar E.) Actin / actinomycrete

Mitochondria / ATP

Once a plant has produced glucose from photosynthesis, the glucose can be catabolized for energy (to make ATP) in the plant cell's... A.) Nucleus. B.) Lysosome. C.) Plasma membrane. D.) Mitochondria. E.) Chloroplasts.

Mitochondria.

In eukaryotic cells, the process of oxidative phosphorylation (respiratory chain) takes place in the ________, whereas the Krebs cycle takes place in the _______. A.) Cytoplasmic membrane / cytoplasm B.) Mitochondria matrix / cytoplasm C.) Mitochondrial inner membrane / mitochondrial matrix D.) Chloroplast inner membrane / chloroplast stroma E.) Endoplasmic reticulum / Golgi apparatus

Mitochondrial inner membrane / mitochondrial matrix

In plants like the fern, haploid cells produced by meiosis divide by _________ to form a multicellular haploid phase. A.) Meiosis B.) Binary fission C.) Mitosis D.) Budding

Mitosis

Mitosis and meiosis differ in that A.) Meiosis does not change the number of chromosomes per cell. B.) Mitosis doubles the number of chromosomes per cell. C.) Meiosis doubles the number of chromosomes per cell. D.) Mitosis does not change the number of chromosomes per cell. E.) Mitosis triples the number of chromosomes per cell.

Mitosis does not change the number of chromosomes per cell.

In pools below waterfalls, where predation is low, male guppies are _________. However, in pools above waterfalls where predation is high, male guppies are ________. A.) More colorful / drab colored B.) Drab colored / more colorful C.) Faster / slower D.) Sorry, predators have no effect on guppies

More colorful / drab colored

What is a possible advantage of being heterozygous for the sickle-cell gene?

More resistance to malaria and a higher chance of living long enough to reproduce

What is the function of secretory vesicles inside a cell? A.) Move proteins between the nucleus and the Golgi apparatus B.) Move proteins between the endoplasmic reticulum and the cell membrane C.) Move proteins between the Golgi apparatus and the plasma membrane D.) Move proteins between the endoplasmic reticulum and the Golgi apparatus E.) Move proteins between the nucleus and the endoplasmic reticulum

Move proteins between the Golgi apparatus and the plasma membrane

What is the function of the transport vesicles inside a cell? A.) Move proteins between the SER And the RER B.) Move proteins between the endoplasmic reticulum and the Golgi apparatus C.) Move proteins between the nucleus and the endoplasmic reticulum D.) Move proteins between the endoplasmic reticulum and the cell membrane

Move proteins between the endoplasmic reticulum and the Golgi apparatus

Which of the following activities does NOT require cells to expend energy? A.) Swimming movements of some types of cells B.) Driving endergonic reactions C.) Movement of organelles within cells D.) Diffusion of water across the cell membrane

Movement of organelles within cells

Human height shows a continuous variation from the very short to the very tall. Height is most likely controlled by... A.) A single gene. B.) Sex-linked genes. C.) The environment. D.) Multiple genes.

Multiple genes.

When an organism's DNA is permanently altered, we say that is A) Mutates. B) Sequences. C) Metabolizes. D) Varies. E) Reproduces.

Mutates.

Changes in DNA and genes are introduced into populations through ______________, the ultimate source of variation. A.) Selection B.) Genetic drift C.) Mutation D.) Migration

Mutation

Which of the following refers to a change in the DNA sequence of a gene, which may cause a change in the amino acid sequence of the encoded protein? A.) Mutation B.) Complementation C.) Nucleotide D.) Carrier

Mutation

A permanent, inheritable chance in the genetic information is called... A.) Translation. B.) Transcription. C.) Mutation. D.) Alteration. E.) Regeneration.

Mutation.

The raw material for evolutionary change is A.) Gene flow. B.) Genetic drift. C.) Mutation. D.) Nonrandom reproduction. E.) Natural selection.

Mutation.

Cancer cells arise from... A.) An inhibition of chlorophyll function. B.) An inhibition of glycolysis. C.) Mutations in the DNA of growth factor genes. D.) A disruption of the cell membrane. E.) Cells to undergo cell death.

Mutations in the DNA of growth factor genes.

Which of these is an electron carrier molecule? A.) NADH B.) ATP C.) Adenylate kinase D.) Carbonic acid

NADH

From the standpoint of the cell, what are the most important products of the Krebs cycle?

NADH and FADH2 are the most important products of the Krebs cycle.

In the light-independent ("synthesis") reactions of photosynthesis, glucose is formed from carbon dioxide. The source of electrons and hydrogen to form the new covalent bonds in glucose come from... A.) ATP B.) NADPH C.) Sunlight D.) Glucose E.) Salt

NADPH

What product from the light reactions receives electrons that originated in chlorophyll and carries them to the Calvin cycle?

NADPH

What is(are) the end product(s) of the light reactions in photosynthesis? A.) ATP only B.) NADP+ and ATP C.) NADP+ and ADP D.) NADPH and ADP E.) NADPH and ATP

NADPH and ATP

Which of the following did Darwin suggest was the mechanism for evolution? A.) Natural selection B.) Analogous selection C.) Artificial selection D.) Founder selection E.) Bottleneck selection

Natural selection

Does glycolysis require oxygen gas?

No

Could a scientist know if a treatment worked or not in an experiment without a control group?

No, every experiment must have a control group to compare to one or more experimental groups.

Can there be evolution without mutation?

No, mutation is required for there to be evolution.

When you add oil to water, the oil does not readily dissolve in the liquid. This shows that oil molecules are.... A.) Polar and hydrophilic. B.) Cohesive. C.) Covalent. D.) Neither hydrophobic nor hydrophilic. E.) Non-polar and hydrophobic.

Non-polar and hydrophobic

What is non-random mating? Give two examples of non-random mating.

Non-random mating occurs when individuals with certain genotypes mate with one another either more or less commonly than would be expected by chance. There are two types of non-random mating: assortative mating and sexual mating. Two examples of sexual mating include the Raggiana Bird of Paradise and lions.

A geneticist is studying the failure of chromosomes to separate properly during meiosis. Which of the following would he be studying? A.) Crossing-over B.) Synapsis C.) Translocation D.) Inversion E.) Nondisjunction

Nondisjunction

When a scientist observes a cell undergoing meiosis, microscopic examination shows too much DNA in one gamete and too little in another. The most likely explanation is that there was an occurrence of... A.) Nondisjunction. B.) Misinheritance. C.) Synapsis. D.) Telophase I. E.) Mitosis.

Nondisjunction.

What kind of access does RNA Polymerase have to heterochromatin? A.) None B.) Moderate C.) Easy D.) Varied by chromosome E.) Permanent

None

Which chemical bond results from the equal sharing of electrons between two atoms? A.) Adhesive B.) Hydrogen C.) Ionic D.) Polar E.) Nonpolar

Nonpolar

Inbreeding within a population is an example of A.) Mutation. B.) Genetic drift. C.) Gene flow. D.) Nonrandom mating. E.) Natural selection.

Nonrandom mating.

At which of these sites would you expect to find the highest number of ribosomes inside the cell? A.) Cytoskeleton B.) Nuclear envelope C.) Smooth endoplasmic reticulum D.) Rough endoplasmic reticulum E.) Lysosome

Nuclear envelope

Proteins and mRNAs can pass between the nucleus and the cytoplasm by moving through these structures in the nuclear envelope. A.) Nuclear pores B.) Cytoplasmic pores C.) Aquaporins D.) Worm holes

Nuclear pores

Which polymer serves as the information storage molecule for cells? A.) Carbohydrate B.) Nucleic acid C.) Protein D.) Lipids

Nucleic acid

Which of the following cell structures is NOT found in bacteria? A.) Ribosomes B.) Plasma membrane C.) Nucleus D.) Cell wall E.) DNA

Nucleus

Provide a short description of each of the following eukaryotic cell structures or organelles. Nucleus Cytoskeleton Mitochondria Endoplasmic Reticulum Golgi apparatus Lysosome

Nucleus- Also known as the "control center". This is where DNA is found (chromatin). Cytoskeleton- Protein filaments that provide structure and movement. The vesicular traffic is regulated by cytoskeleton proteins. Mitochondria- Also called the "powerhouse of the cell". An organelle that harvests energy for the cell by oxidation of glucose to make ATP. It uses oxygen and releases carbon dioxide in the process. Endoplasmic Reticulum- Organelle that produces and modifies molecules to be exported to other parts of the organism. It also breaks down toxic chemicals and cellular by-products. Golgi apparatus- This is where everything the ER makes gets transported to. It is a stack of slightly curved but flat saccules that sends biomolecules to their final destination. Lysosome- They hydrolyze macromolecules to carry out intracellular digestion.

The identity of any atom can be determined by the ... A.) Number of electrons. B.) Atomic charge. C.) Atomic mass. D.) Number of protons. E.) Number of neutrons.

Number of protons.

What would occur if all the decomposers were removed from an ecosystem? A.) Energy would no longer be lost in the form of heat. B.) Nutrients from dead organisms could not be recycled into the ecosystem. C.) The ecosystem would lose its ultimate source of energy. D.) Nutrient recycling from dead organisms would increase. E.) There would be no effect on the ecosystem.

Nutrients from dead organisms could no longer be recycled into the ecosystem.

All of the following pertain to transcription except... A.) Occurs on a ribosome in the cytoplasm. B.) Occurs before translation. C.) Requires RNA polymerase. D.) Requires a template DNA strand. E.) Proceeds in the 5' to 3' direction of the growing mRNA molecule.

Occurs on a ribosome in the cytoplasm.

Where are genes located in the cell? A.) On chromosomes B.) On the endoplasmic reticulum C.) In the Golgi complex D.) On the plasma membrane

On chromosomes

Mutated proto-oncogenes become cancer-causing genes called _________.

Oncogenes

Mutated proto-oncogenes become cancer-causing genes called __________.

Oncogenes

Which are cancer-causing genes? A.) Inducible genes B.) Tumor suppressor genes C.) Oncogenes D.) Proto-oncogenes

Oncogenes

A basic difference between spermatogenesis and oogenesis is that in oogenesis A.) Four functional eggs are produced. B.) Only one cell eventually complete meiosis II. C.) Both sperm and egg are produced. D.) Four cells are produced and fertilization is required for completion. E.) One cell is produced and fertilization is required for completion.

One cell is produced and fertilization is required for completion.

The antiparallel arrangement within DNA molecules refers to... A.) Each base bonding at the 1' position of the sugar. B.) A purine always bonding to a pyrimidine. C.) One helix strand that runs from the 5' to 3' direction and the other strand runs from the 3' to 5' direction. D.) An original parent DNA strand and one newly synthesized DNA strand comprising a new DNA molecule. E.) None of the choices is correct.

One helix strand that runs from the 5' to 3' direction and the other strand runs from the 3' to 5' direction.

What is the error rate in the DNA replication process, for eukaryotes? A.) One in 1 billion nucleotides B.) One in 1 hundred nucleotides C.) One in 1 thousand nucleotides D.) One in 10,000 nucleotides E.) One in 1 million nucleotides

One in 1 billion nucleotides

In semiconservative DNA replication, each new double helix formed will have... A.) Three new strands in one helix and three old strands in the second helix. B.) One new strand and one old strand in each helix. C.) Two new strands and two old strands in each helix. D.) Two new and one old strand in one helix and two old and one new in the second helix. E.) Only new strands in each helix.

One new strand and one old strand in each helix.

How can two pea plants identical in phenotype, both with yellow pea color, have different genotype for the pea color trait? A.) Genotype has no relation to phenotype. B.) One of the two plants could be homozygous for the dominant allele while the other could be homozygous for the recessive allele. C.) One of the two plants could be homozygous for the recessive allele while the other could be heterozygous. D.) One of the two plants could be homozygous for the dominant allele while the other could be heterozygous.

One of the two plants could be homozygous for the dominant allele while the other could be heterozygous.

If an organism shows a recessive phenotype, such as short pea plants, it can be... A.) Either TT or Tt. B.) Only tt. C.) Either TT or tt. D.) Only TT. E.) Only Tt.

Only tt.

Replication of DNA begins at a(n) ____________, which is a specific adenine-thymine rich area in the DNA. A.) Origin of replication B.) Promoter C.) Terminator D.) Start codon

Origin of replication

In cells, DNA replication begins at a site on the DNA called the... A.) Cleavage furrow. B.) Origin of replication. C.) Cell plate. D.) Binary fusion.

Origin of replication.

The electron transport chain (also called the respiratory chain) produces ATP molecules via a process known as __________ phosphorylation.

Oxidative

Nicotinamide adenine dinucleotide (NAD) exists in two forms: an oxidized and reduced form. What is the oxidized form? What is the reduced form?

Oxidized- NAD+ Reduced- NADH

Cellular respiration... A.) Oxidizes glucose and reduces oxygen. B.) Oxidized oxygen and reduced glucose. C.) Releases energy and reduces water. D.) Requires energy and releases NADH. E.) Releases energy and oxygen.

Oxidizes glucose and reduces oxygen.

Which products made during photosynthesis are needed b animals to live and grow?

Oxygen - last electron acceptor of cell respiration needed to produce ATP Organic matter - starting point of cell respiration

A plant cell's central vacuole functions in A.) Manufacturing sugars. B.) Capturing energy from sunlight. C.) Packaging molecules for export from the cell. D.) Storing the genetic information of the cell. E.) Storing water, sugars, and ions.

Packaging molecules for export from the cell.

If you followed the breakdown of long chain fatty acids, in what organelle would they be digested first, before they could be used to make cellular energy in the mitochondria? A.) Rough endoplasmic reticulum B.) Mitochondria C.) Smooth endoplasmic reticulum D.) Nucleus E.) Peroxisomes

Peroxisomes

When a protozoan engulfs bacteria cells for food, it does so by using this type of endocytosis

Phagocytosis

Which of the following terms is represented by word descriptions, such as brown or tall? A.) Phenotype B.) Homozygous C.) Allele D.) Genotype E.) Dominance

Phenotype

ATP is composed of a ribose sugar, the organic base adenine, and three ________ groups.

Phosphate

Which of these functional groups would be associated with nucleotides and phospholipids? A.) Hydroxyl B.) Phosphate C.) Sulfhydryl D.) Carbonyl E.) Carboxyl

Phosphate

What molecule makes up the polar head group of a phospholipid? What molecule makes of the nonpolar tails?

Phosphate makes up the head group of the phospholipid. Fatty acids make up the nonpolar tails.

What are the three parts which make up a nucleotide?

Phosphate, pentose sugar, and a nitrogen-containing base are the three parts that make up a nucleotide.

Which of the following is not a correct association of a lipid molecule and its specific function? A.) Beeswax: main structural component of the bee honeycomb B.) Phospholipid: main component of fats and oils C.) Wax: waterproof covering of plant leaves D.) Testosterone: steroid involved in muscle development in males E.) Cholesterol: stabilize cell membrane of animal cells

Phospholipid: main component of fats and oils

The membrane lipid responsible for forming the cell plasma membranes is A.) Phospholipids B.) Cell surface proteins C.) Transmembrane proteins D.) Carbohydrate chains

Phospholipids

Nitrogen (N) and phosphorus (P) are needed to make cell biomolecules. List 3 types of biomolecules that require either nitrogen or phosphorus for their synthesis.

Phospholipids, nucleic acid, and proteins are three types of biomolecules that require either nitrogen or phosphorus for their synthesis.

What types of living organisms are capable of performing photosynthesis?

Photoauotrophs Plants, algae and certain types of bacteria

Light energy arrives at earth in tiny packets called... A.) Rays. B.) Photons. C.) Light waves. D.) Wavelengths.

Photons.

Which of the following is not a pathway that cells use to get energy from food/nutrient molecules? A.) Cellular respiration B.) Fermentation C.) Photosynthesis D.) Anaerobic respiration

Photosynthesis

Molecules that absorb light energy are called _______.

Pigments

Molecules that absorb light energy are called ________________________________.

Pigments

Which of the following is not a type of cytoskeletal fiber? A.) Microtubules B.) Actin microfilaments C.) Intermediate filaments D.) Pili

Pili

Which process imports liquid and any dissolved solutes in bulk into cells? A.) Pinocytosis B.) Facilitated transport C.) Phagocytosis D.) Exocytosis

Pinocytosis

Based on observations that a cell has a nucleus, a cell wall, and a large central vacuole, you can conclude that this cell is most likely a(n)... A.) Archaean cell B.) Animal cell C.) Bacterial cell D.) Plant cell E.) Prokaryotic cell

Plant cell

Which kingdom contains photosynthetic multicellular organisms that live on the land? A.) Fungi B.) Plantae C.) Animalia D.) Protista E.) Archaea

Plantae

Why do deciduous leave turn red and yellow in the fall?

Plants break down and reabsorb chlorophyll; this lets the colors of other pigments show through. There is less light reflected which makes the pigments change in the leaves. There is also less water and humidity. Deciduous leaves produce sugar in the Spring and shut down metabolism in the fall.

What is the overall chemical reaction of photosynthesis?

Plants use energy fro sunlight to react Carbon Dioxide and water to produce sugar (glucose) and Oxygen Carbon Dioxide + Water + Light (yields =) Glucose (sugar) + Oxygen 6 Co2 + 6 H2O + Light = C6H12O6+ 6O

What is the function of the plasma membrane? What type of lipid is responsible for forming the cell plasma membrane?

Plasma membrane- Imports and regulates (separates and regulates) Phospholipids make up a cell plasma membrane

What are the membrane-lined channels that allow direct exchange of materials between two adjacent plant cells? A.) Extracellular matrix B.) Plasmodesmata C.) Intermediate filament D.) Tight junctions E.) Anchoring desmosome

Plasmodesmata

If several traits are affected by the same allele, the allele is said to be... A.) Recessive. B.) Dominant. C.) Epistatic. D.) Pleiotropic.

Pleiotropic.

If you were studying the phenomena of one gene having more than one effect, you would be studying which of the following? A.) Gene linkage B.) Codominance C.) Incomplete dominance D.) Genotype dominance E.) Pleiotropy

Pleiotropy

When two or more sets of alleles affect the same trait, it is referred to as... A.) Codominant. B.) A double-trait cross. C.) Polygenic inheritance. D.) Polygenic dominance. E.) A single-trait cross.

Polygenic dominance.

A chemist is in the lab studying a large organic molecule with many subunits, which would best be termed a... A.) Isomer. B.) Hydromer. C.) Polymer. D.) Hydrophilic. E.) Monomer.

Polymer.

When scientists wish to amplify (make many copies) of a gene, they employ the _______ technique. A.) Centrifuge B.) Pepsin activation reaction C.) Polymerase chain reaction D.) Restriction enzymes

Polymerase chain reaction

Which of the following levels of organization goes beyond an individual organism? A.) Organ B.) Cell C.) Tissue D.) Organ system E.) Population

Population

Which of these statements describes the structure of an atom? A.) Neutral protons and negative neutrons in the nucleus are surrounded by positive electrons. B.) Positive protons and negative neutrons in the nucleus are surrounded by neutral electrons. C.) Negative protons and positive neutrons in the nucleus are surrounded by neutral electrons. D.) Positive protons and neutral neutrons in the nucleus surrounded by negative electrons. E.) Positive protons and negative electrons in the nucleus are surrounded by neutral neutrons.

Positive protons and neutral neutrons in the nucleus are surrounded by negative electrons

When insulin is initially synthesized, the polypeptide chain is too long and must have a part cleaved off. What type of regulation of gene expression is this? A.) Post-transcriptional control B.) Post-translational control C.) Chromatin regulation D.) Translational control E.) Transcriptional control

Post-translational control

Which type of gene expression control occurs after the protein has been synthesized? A.) Translational control B.) Posttranslational control C.) Inducible control D.) Posttranscriptional control E.) Transcription control

Posttranscriptional control

Objects that are not actively moving but have the capacity to do so are said to possess ________ energy.

Potential

As a rock rolls down a hill, which of the following statements would best explain what is occurring? A.) Potential energy increases as heat energy increases. B.) Kinetic energy decreases as potential energy increases. C.) Kinetic energy decreases as heat energy increases. D.) Potential energy decreases as kinetic energy increases. E.) Potential energy and kinetic energy do not change.

Potential energy decreases as kinetic energy increases.

Bacteria populations grow using asexual reproduction. This has some advantage over sexual reproduction, but it also has some disadvantages. One disadvantage of asexual reproduction is that it... A.) Takes more time than sexual reproduction does. B.) Takes more effort than sexual reproduction does. C.) Produces offspring with less genetic diversity than with sexual reproduction. D.) Is much riskier than sexual reproduction. E.) Sorry, there are no disadvantages to asexual reproduction.

Produces offspring with less genetic diversity than with sexual reproduction.

Which of the following best represents the "synthesis" part of photosynthesis? A.) Production of glucose from CO2, ATP, and NADPH B.) Production of glucose from light and H2O C.) Production of ATP and NADPH from light and H2O D.) Production of ATP from glucose, H2O, and O2 E.) Sorry, nothing is synthesized during photosynthesis.

Production of glucose from CO2, ATP, and NADPH

The direction of and site at which the transcription of a gene begins is controlled by the position and orientation of the ___________. A.) Promoter B.) Origin of replication C.) Start codon D.) Ribosome

Promoter

What defines the beginning and the end of a gene?

Promoter sequence starts the gene while the terminator sequence ends it.

The region of DNA to which RNA polymerase binds to initiate transcription is called the... A.) Promoter. B.) Operator. C.) Operon. D.) Exon.

Promoter.

During which stage of mitosis do replicated chromosomes condense and the nuclear envelope disappear?

Prophase

Explain the pros and cons of regulating an enzyme's activity at the post-translational level (i.e. feedback inhibition) versus the transcriptional level.

Pros of Post-Translational- proteases protect the cell by being confined to lysosomes or special structures called proteasomes, proteasomes contribute to the development of cancer if they digest which helps control the cell cycle and prevent tjis cancer Pros of Translational- the long life of mRNAs that code for hemoglobin in mammalian red blood cells is attributed to the persistence of their caps and their poly-A tails, translation can be inhibited either by the presence of miRNAs attached to the mRNA or by the presence of translation repressor proteins Cons of Post-Translational- last chance a cell has for influencing gene expression, some proteins are not immediately active after synthesis, just how long a protein remains in a cell is usually regulated by the use of proteases, the inability of proteasomes to degrade other proteins is implicated in Alzheimer, Parkinson, and mad cow disease Cons of Translational- Involves the ability of an mRNA for translation at the ribosome, presence or absence of the 5' cap and the length of the 3' poly-A tail of a mature mRNA transcript determines whether translation takes place and how long the mRNA is active

Most enzymes are... A.) Proteins B.) Nucleic acids C.) Carbohydrates D.) Lipids

Proteins

Normal cellular genes that code for proteins that stimulate the cell cycle or prevent cell death: A.) Proto-oncogenes B.) Oncogenes C.) Tumor suppressor genes D.) Inducible genes

Proto-oncogenes

In chemiosmosis, a(n) ________ gradient drives the production of ATP using ATP synthase channels. A.) Pigment B.) Chlorophyll C.) Proton D.) Electron E.) NADPH

Proton

This subatomic particle carries a positive charge (+1) and is found in the nucleus of an atom

Proton

Which of the following is NOT a use of water by living organisms? A.) External transportation for chemicals B.) Help exchange heat C.) Provides nutrient energy for metabolism D.) Provides a medium for movement

Provides nutrient energy for metabolism

In the electron transport chain, electrons are passed from one carrier to another, providing energy to accomplish which of the following? A.) Convert NAD+ and NADH B.) Convert FAD and FADH2 C.) Convert ADP and ATP D.) Pump hydrogen ions into the matrix E.) Pump hydrogen ions out of the matrix

Pump hydrogen ions out of the matrix

The model of speciation that states evolution has long periods of no change followed by brief periods of rapid change is referred to as the... A.) Sympatric model. B.) Allopatric model. C.) Punctuated equilibrium model. D.) Gradualistic model.

Punctuated equilibrium model.

What are the pyrimidines? What are the purines? Which ones are used in DNA and which ones are used in RNA?

Pyrimidines- cytosine, thymine, uracil Purines- adenine, guanine DNA uses cytosine, thymine, adenine, and guanine; whereas RNA uses cytosine, uracil, adenine, and guanine

In the process of glycolysis, glucose is cleaved into a pair of three-carbon molecules called

Pyruvate

Which of the following levels of protein structure refers to assemblies of multiple polypeptide chains together into a complex? This level of structure is found in hemoglobin and many enzymes. A.) Primary B.) Secondary C.) Tertiary D.) Quaternary

Quaternary

During the process of translation, the information in... A.) DNA is converted into RNA information. B.) RNA is converted into protein information. C.) DNA is converted into protein information. D.) Protein is converted into RNA information. E.) RNA is converted into DNA information.

RNA is converted into protein information.

Gene regulation can involve a protein repressor that blocks _______ from initiating transcription. A.) DNA polymerase I B.) DNA polymerase III C.) RNA polymerase D.) mRNA

RNA polymerase

Which is the correct association of a cell membrane protein type and its function? A.) Receptor proteins: have a binding site for a specific molecule B.) Channel proteins: assist in cell-to-cell communication C.) Carrier proteins: carry out metabolic reactions D.) Cell recognition proteins: help substance move across the cell membrane E.) Enzymatic proteins: allow molecules to move across membrane

Receptor proteins: have binding site for a specific molecule

Bovine somatotropin (BST) or bovine growth hormone (BGH) is a peptide hormone produced by cows' pituitary gland that stimulates growth and milk production. In the United States, public opinion led some manufacturers and retailers to market milk that is free of rBST. This rBST refers to... A.) Reduced BST, which has had electrons added to it. B.) Rat BST, purified from rats. C.) Rancid BST, which has spoiled. D.) Recombinant BST, which is produced via biotechnology.

Recombinant BST, which is produced via biotechnology.

In genetic engineering, the combination of DNA from two or more sources into one molecule is called... A.) 2-DNA technology B.) Human growth hormone technology C.) Recombinant DNA technology D.) Biotechnology

Recombinant DNA technology

Plants absorb ______ and ______ wavelengths of light and reflect _______ wavelengths. A.) Green, red, blue B.) Green, blue, red C.) Red, blue, green D.) White, green, blue E.) White, green, red

Red, blue, green

Because oxidation and reduction go hand in hand, the entire reaction is called a _______ reaction.

Redox (oxidation-reduction)

A biologist is studying a gene which is located outside the operon and codes for a repressor protein that can deactivate the operon. This biologist is studying which of the following genes? A.) Structural gene B.) Promoter gene C.) Regulatory gene D.) Operator gene

Regulatory gene

During the S-phase of the cell cycle, the chromosomes are __________, forming two identical copies of each chromosome called sister chromatids. A.) Degraded B.) Oxidized C.) Cleaved D.) Replicated

Replicated

Dolly (born July 1996 - died February 2003) was a female domestic sheep, and the first mammal ever cloned using the process of...

Reproductive cloning

Ian Wilmut successfully cloned Dolly the sheep from a fully differentiated sheep cell. This type of cloning is known as... A.) Reproductive cloning. B.) Therapeutic cloning. C.) Stem cell cloning. D.) Gene therapy cloning.

Reproductive cloning.

A protein encoded by a tumor-suppressor gene normally acts to... A.) Mutate the cell. B.) Stimulate cell division. C.) Restrict cell division. D.) Promote metastases.

Restrict cell division.

Removing genes from molecules of DNA requires the use of special... A.) Digestive enzymes. B.) Restriction enzymes. C.) Enzymes from peroxisomes. D.) Microscopic scalpels.

Restriction enzymes.

The sugar found in a nucleotide of RNA is...

Ribose

Which of the following are found within bacteria cells? A.) Ribosomes B.) Endoplasmic Reticulum C.) Nucleus D.) Mitochondria

Ribosomes

Which of the following would you expect to be found in larger numbers inside cells that are producing increased amounts of proteins compared with cells producing less? A.) Centrioles B.) Lysosomes C.) Ribosomes D.) Mitochondria

Ribosomes

Which of the following does not conduct photosynthesis? A.) Algae B.) Leaves of plants C.) Some bacteria D.) Roots of plants

Roots of plants

Compare and contrast the functions of the rough ER and the smooth ER.

Rough ER- protein synthesis (ribosomes) Smooth ER- lipid and carbohydrate synthesis

Which of the following crosses represents a dihybrid cross? A.) RRrr x DdDd B.) RR x Dd C.) RrDd x RRdd D.) RD x Rd E.) Rd x RR

RrDd x RRdd

____________________ catalyzes the first step in the Calvin cycle and is also the most abundant enzyme on Earth

Rubisco Ribulose Caroxylase

What is the main use of the transgenic mice created in the past decade?

Scientific and medical research Basic research Understanding basic gene transferring process

The terms alpha helix and pleated sheet are associated with which level of protein structure?

Second level

When a protein is denatured, which level of protein structure is affected? (select all that apply) A.) Primary B.) Secondary C.) Tertiary D.) Quaternary

Secondary, Tertiary, and Quaternary

Which choice describes how the DNA molecule is replicated in cells? A.) Semiconservative B.) Dispersive C.) Conservative D.) Either dispersive or conservative depending on the organism.

Semiconservative

Turner syndrome and Klinefelter syndrome result from...

Sex chromosomes anomalies are the direct cause of Turner syndrome and Klinefelter syndrome. The chromosomes fail to separate properly during meiosis (nondisjunction). Nondisjunction of chromosomes during Meiosis I of oogenesis, followed by fertilization, will result in these syndromes.

X and Y chromosomes are examples of... A.) Autosomes. B.) Sex chromosomes.

Sex chromosomes.

Meiosis is a lot more complicated than mitosis. Consider why some living things go to so much trouble to reproduce. What is the main advantage of sexual reproduction over asexual reproduction?

Sexual reproduction leads to offspring that are genetically different from one another and either parent.

Which repetitive DNA is a standard for DNA fingerprinting analysis of criminal cases? A.) Short tandem repeats B.) Transposons C.) Interspersed repeats D.) Unique noncoding DNA E.) Introns

Short tandem repeats

What types of mutation alters the base, but not the amino acid being coded for?

Silent mutation

Define each of the following types of mutations: silent, missense, nonsense, and frame-shift.

Silent- no change in amino acid Missense- changes one amino acid to another Nonsense- changed to a stop codon Frame-shift- deletions and insertions cause dramatic changes in the DNA coding sequence

Analogous structures results from... A.) Similar selective pressures. B.) Differing selective pressures. C.) Chance alone. D.) Directional mutation.

Similar selective pressures.

Anatomical features that are similar in organisms that are not evolutionarily related are due to... A.) Similar selective pressures. B.) Differing selective pressures. C.) Chance alone. D.) Directional mutation.

Similar selective pressures.

Which is incorrect about meiosis? A.) Crossing over occurs in prophase I B.) Meiosis I results in 2 haploid cells C.) Sister chromatids separate in anaphase I D.) Meiosis II is like mitosis E.) Chromosomes replication only occurs in interphase I

Sister chromatids separate in anaphase I

Which of these events happens during anaphase II? A.) Sister chromatids separate. B.) Chromosomes condense. C.) Homologous chromosomes separate. D.) The nuclear membrane disappears. E.) Recombination occurs.

Sister chromatids separate.

Replicate copies of each chromosome are called _________ and are joined at the ___________. A.) Homologues; centromere B.) Sister chromatids; kinetochore C.) Sister chromatids; centromere D.) Homologues; kinetochore

Sister chromatids; centromere

Which of the following types of organisms would be found in unusually great numbers in the Burgess Shale? A.) Fish B.) Soft-bodied invertebrates C.) Dinosaurs D.) Amphibians E.) Mammals

Soft-bodied invertebrates

When water is mixed with a salt in a glass, the salt would be considered the _______ and the water would be considered the _____.

Solute, solvent

Explain why mutations are not all bad.

Some are silent mutations where there is no change in the amino acid sequence. Some mutations can even benefit the organism (sickle-cell resistant to malaria).

Water has some unique properties that make it very important for life. List some of these properties? What chemical bonds occurring between individual water molecules make these properties possible?

Some properties of water include heat storage, ice formation, high heat of vaporization, cohesion, and high polarity. The chemical bond occuring between individual water molecules which make these properties possible is a hydrogen bond.

Red-green colorblindness is a sex-linked recessive trait. Imagine a case of a woman with normal color vision, whose father was colorblind. She mates with a colorblind man. What chances do each of the children have being colorblind?

Son - 25% Daughter - 50% Overall, all children will have a 50% chance of being colorblind.

Most of the oxygen in the Earth's atmosphere comes from photosynthesis. From which of the following is the oxygen derived? A.) Splitting of carbon dioxide B.) Splitting of water C.) Splitting of glucose D.) Splitting of chlorophyll E.) Splitting of NADH

Splitting of water

In human infants, there has long been evolution toward having the highest survival rate at a 6- to 7-pound birth weight. This is an example of A.) Directional selection. B.) Disruptive selection. C.) Stabilizing selection. D.) Random chance.

Stabilizing selection.

The frequency of sickle-cell disease in Central African humans is an example of... A.) Directional selection. B.) Disruptive selection. C.) Stabilizing selection. D.) Random selection.

Stabilizing selection.

The lipid group that serves as an energy source and a long-term energy storage molecule is... A.) Phospholipid B.) Triglyceride C.) Sterol D.) Wax

Sterol

Proto-oncogenes encode proteins that... A.) Stimulate cell division. B.) Suppress tumors. C.) Restrict cell division. D.) Control programmed cell death.

Stimulate cell division.

Functions of carbohydrates include...

Storing energy and forming structural parts for cells

Within chloroplasts, the fluid-like matrix in which the Calvin cycle occurs is called... A.) Thylakoids. B.) Grana. C.) Stroma. D.) Mesophyll.

Stroma.

Lemon Juice has a pH of 2.3 and should be classified as a... A.) Strong acid. B.) Buffering solution. C.) Strong base.

Strong acid.

During glycolysis, an enzyme catalyzes the transfer of a phosphate from phosphoenolpyruvate to ADP forming ATP. This way of making ATP is referred to as... A.) Enzyme ATP synthesis. B.) Active site ATP synthesis. C.) Substrate-level ATP synthesis. D.) Enzyme ADP synthesis. E.) Substrate-level ADP synthesis.

Substrate-level ATP synthesis.

The disaccharide which is transported in plants and is also used as table sugar is... A.) Lactose B.) Glucose C.) Sucrose D.) Fructose

Sucrose

The polysaccharide that plants use to store energy for is called... A.) Starch. B.) Cellulose. C.) Chitin. D.) Sucrose.

Sucrose.

What is the overall chemical reaction of photosynthesis?

Sunlight + water + carbon dioxide --> oxygen + sugar

List 6 general functions played by proteins in animals

Support, metabolism, transport, defense, regulation, and motion are six general functions played by proteins in animals.

In human intestinal cells, what specialized feature increases the surface-area-to-volume ratio and allows increased absorption of materials? A.) Very few organelles inside cells B.) Larger cell size C.) Many cilia D.) Many flagella E.) Surface folds called microvilli

Surface folds called microvilli

"Survival of the fittest" may be a misleading phrase to describe the process of evolution by natural selection because A.) If it impossible to determine the fittest individuals in nature. B.) Fitness has little to do with natural selection. C.) Natural variation in a population is not influenced by differential reproductive success. D.) Survival is not the only factor in natural selection affecting reproductive success.

Survival is not the only factor in natural selection affecting reproductive success.

A biologist is studying the speciation of organisms without geographical barriers. That is... A.) Allopatric speciation. B.) Microevolution. C.) Sympatric speciation. D.) Ecological speciation. E.) Hybrid speciation.

Sympatric speciation.

In the cell cycle the "S" phase is named for A.) Sexual reproduction. B.) Sister chromatids. C.) Synthesis of DNA. D.) Synapsis of DNA.

Synthesis of DNA.

DNA polymerase... A.) Is needed for adding nucleotides during mRNA synthesis. B.) Synthesizes new DNA only in the 5' to 3' direction. C.) Cannot add nucleotides to the lagging strand. D.) Synthesizes an RNA primer.

Synthesizes new DNA only in the 5' to 3' direction.

If one strand of a DNA molecule has the base sequence ATTGCAT, its complementary strand will have the sequence...

TAACGTA

The branch of biology that deals with the identification and naming of organisms is... A.) Biology. B.) Herpetology. C.) Taxonomy. D.) Physiology.

Taxonomy.

During this stage of mitosis, the mitotic spindle disassembles, the nuclear membrane re-forms, and the chromosomes uncoil/decondense

Telophase

During which mitotic stage is the spindle disassembled, the nuclear membrane re-formed, and the chromosomes uncoiled?

Telophase

During DNA replication, each parent DNA strand serves as a ______________ for synthesis of new DNA strands.

Template

During transcription, only one strand serves as a __________ for synthesis of new RNA strands.

Template

The three-dimensional shape of a polypeptide (protein) is called its _______ structure.

Tertiary

Why is a molecules that binds to and blocks the action of ATP synthase lethal to plants? A.) ATP synthase splits water and release oxygen; without the production of oxygen, the plants will die. B.) ATP synthase is the enzyme responsible for carbon dioxide fixation. C.) The ATP produced by ATP synthase powers the Calvin cycle, the ultimate source of all organic molecules made by the plant. D.) When ATP synthase is blocked, the electron transport chain moves electrons backwards to the reaction center. E.) If ATP synthase is blocked, chemiosmosis cannot occur, and as a result the plant will run out of water and dehydrate.

The ATP produced by ATP synthase powers the Calvin cycle, the ultimate source of all organic molecules made by the plant.

Describe the ATP-ADP cycle in living things

The ATP-ADP cycle has everything to do with the storage and use of energy in living things.

What is the Central Dogma of Molecular Biology (it is also called the Central Dogma of Genetics)?

The Central Dogma of Molecular Biology involves a gene composed of DNA that is first copied into a molecule of RNA. This is then read into a protein.

Most of the carbon dioxide that is made during aerobic respiration is from... A.) Glycolysis. B.) The Krebs cycle. C.) The electron transport chain. D.) The Calvin cycle.

The Krebs cycle.

What is the "way of learning" in science?

The Scientific Method

What is explained in the Theory of Evolution?

The Theory of Evolution is the unifying theory of biology. It is the scientific explanation for the unity and diversity of life on Earth.

Until the 19th century, all peppered moths (Biston betularia) observed around Manchester, England, were light in color. In 1845, a single black peppered moth was reported. As Manchester became more industrialized, the frequency of black moths increased greatly. The change in proportion of light and dark forms most likely resulted from A.) The black moths evading predators more and thus reproducing more often. B.) The black moths being more easily seen by members of the opposite sex. C.) The black moths being better able to find food at night. D.) The black moths being more resistant to starvation. E.) Random genetic drift.

The black moths evading predators more and thus reproducing more often.

What is Genomics?

The branch of molecular biology concerned with the structure, function, evolution, and mapping of genomes. Genomics is a modern field (in the last 20 years) that compares the entire DNA content of different organisms.

If a cell did not have RNA molecules, what would be the result? A. The DNA would take over its job. B. The cell would not have genes. C. There would be no effect on the cell. D. The cell could not make energy. E. The cell would not be able to make proteins.

The cell would not be able to make proteins.

If a poison that specifically inhibited ribosome function were added to cells, the cells would die directly from the fact that... A.) The cells would not be able to digest any bacteria for food. B.) The cells would not be able to generate any cell energy. C.) The cells would not be able to make any new proteins. D.) The cell would no longer be able to move around.

The cells would not be able to make any new proteins.

The goal of comparative genomics is... A.) The determination of how species are related. B.) The structure, function, and interactions of cellular proteins. C.) The structure, function, and interactions of genes. D.) The application of computer technologies to study the genome. E.) The application of computer technologies to study human proteins.

The determination of how species are related.

Why is biodiversity generally a good thing for a species or an ecosystem?

The direct values of biodiversity are medical values, agricultural values, and consumptive use values. The indirect values of biodiversity are to help dispose of waste, provide fresh water, prevent soil erosion, and regulate climate.

Does the disaccharide lactose induce or repress the expression of the lac operon? How does lactose carry out this effect?

The disaccharide lactose induces the expression of the lac operon. The lac operon is usually repressed but can be turned on in the presence of the inducer allolactose.

The full range of energy in sunlight can best be described as... A.) The electromagnetic spectrum. B.) Visible light. C.) Ultraviolet light. D.) Infrared radiation.

The electromagnetic spectrum.

The evolution of bacteria would best be described by A.) The biological species concept. B.) Prezygotic isolation mechanism. C.) The evolutionary species concept. D.) Postzygotic isolation mechanism. E.) Sympatric speciation.

The evolutionary species concept.

The species concept which recognizes that every species has its own evolutionary history is referred to as A.) The biological species concept. B.) The reproductive species concept. C.) The adaptive species concept. D.) The evolutionary species concept. E.) The prezygotic species concept.

The evolutionary species concept.

What is biotechnology?

The exploitation of biological processes for industrial and other purposes, especially the genetic manipulation of microorganisms for the production of antibiotics, hormones, etc. Use of cells or cell components to make a product (Foods, antibiotics, vitamins, enzymes, etc.)

Which of the following would have the greatest impact on an organism? (1) a mistake made during DNA replication placing an incorrect nucleotide into a certain position in a gene or (2) a mistake made during transcription placing an incorrect nucleotide into one mRNA molecule made from that gene. Explain.

The first one because result would be a mutation which would be passed down because it serves as a template for future replication. Whereas in the second case, the mRNA would only be temporary and not passed down.

What is the benefit of genetically engineering plants to be resistant to glyphosate (the active ingredient in the product Roundup)?

The glyphosate resistance enables farmers to wipe out most weeds from the fields without damaging their crops. Glyphosate is a powerful herbicide that kills most actively growing plants and is used to control weeds. A gene from the bacterium that gives resistance to glyphosate is inserted into crop plants. The glyphosate can then be applied to fields and orchards where it retards weed growth but not crop growth.

With operons under negative transcriptional control (e.g. lac operon), how does the presence of an inducer (lactose) affect transcription? A.) The inducer binds to the operator, leading to inhibition of transcription. B.) The inducer falls off the operator, leading to increased transcription. C.) The inducer causes a repressor to bind the operator, leading to inhibition of transcription. D.) The inducer prevents a repressor from binding to the operator, leading to increased transcription. E.) None of the above, operons under negative control do not have inducers.

The inducer prevents a repressor from binding to the operator, leading to increased transcription.

Which is incorrect about photosynthesis? A.) The light reactions take place in the cytoplasm and the dark reactions occur in the chloroplast. B.) The light-independent reactions don't require light but can occur in its presence. C.) The light-dependent reactions produce NADPH and ATP. D.) The light-dependent reactions release oxygen. E.) Photosynthesis act as antennae to capture photons.

The light reactions take place in the cytoplasm and the dark reactions occur in the chloroplast.

The energy to power the Calvin cycle comes from... A.) Cellular respiration. B.) The light-dependent reactions of photosynthesis. C.) The light-independent reactions of photosynthesis. D.) Oxygen. E.) Chlorophyll.

The light-dependent reactions of photosynthesis.

What is the monomer (building block) for building a DNA or RNA strand? What is each monomer composed of?

The monomer for building DNA or RNA strand are nucleotides. They are composed of a pentose sugar and a phosphate and a nitrogen-containing base.

The more unsaturated a lipid is, A.) The more glycerol it contains. B.) The more liquid it is at room temperature. C.) The more solid it is at room temperature. D.) The more hydrogen it contains. E.) The fewer kinks it has.

The more liquid it is at room temperature.

The spindle is... A.) The new cell membrane as it reforms across the cytoplasm during cytokinesis. B.) A point of constriction on each chromosome containing repeated DNA sequences. C.) The network of protein cables that will pull the chromatids to opposite ends of the cell. D.) The protein that DNA winds itself around.

The network of protein cables that will pull the chromatids to opposite ends of the cell.

Mitosis is the part of the eukaryotic cell cycle in which... A.) The DNA is transcribed into mRNA B.) Cells grow in size preparation for division C.) The nucleus divides into two D.) The cell / cytoplasm is divided into two E.) Sorry, mitosis is not part of the cell cycle

The nucleus divides into two.

The process of independent assortment refers to... A.) The random meeting of an egg and sperm. B.) The orientation of homologous chromosomes at metaphase I. C.) The orientation of chromosomes at metaphase II. D.) The random choice of which cell in germ-line tissue will begin meiosis. E.) Whether sexual or asexual reproduction will occur.

The orientation of homologous chromosomes at metaphase I.

In humans, germ-line cells are located in... A.) The brain. B.) The muscles. C.) The kidneys. D.) The ovaries and testes. E.) The eyes.

The ovaries and testes.

Today, the insulin protein used by diabetics to treat their disease is recombinant human insulin. Before the era of Biotechnology, the insulin used in treatment was obtained from... A.) Human blood donations. B.) Naturally occurring plants. C.) Transgenic bacteria. D.) The pancreas of animals used in the food industry.

The pancreas of animals used in the food industry.

The primary function of a cell's plasma membrane is to...

The primary function of a cell's plasma membrane is to enclose all cells, act as a boundary, separate intracellular components from the environment, and regulate what comes in and what goes out.

What is gene pharming? A.) The production of pesticides by plants. B.) The production of pesticides by animals. C.) The production of insecticides by animals. D.) The production of insecticides by plants. E.) The production of pharmaceuticals by animals.

The production of pharmaceuticals by animals.

If you were studying functional genomics, you would be most interested in... A.) The application of computer technology to study human proteins. B.) The role of the genome in cells or organisms. C.) The evolution of a species and how genes and noncoding regions function. D.) The structure, function, and interactions of noncoding genes. E.) The application of computer technologies to study the genome.

The role of the genome in cells or organisms.

What is bioinformatics?

The science of collecting and analyzing complex biological data such as genetic codes.

Which is most directly responsible for the sequence of amino acids in a protein? A.) The proteins associated with rRNA B.) The number of codons in mRNA C.) The sequence of codons in mRNA D.) The enzyme that attaches the amino acid to tRNA

The sequence of codons in mRNA

When water is mixed with a salt in a beaker, the salt would be considered... A.) The solvent. B.) The solute. C.) The solution. D.) The buffer. E.) The ion.

The solute.

What is proteomics?

The study of proteomes and their functions. Study the structure, function, and interaction of a cell's proteins

Directly or indirectly, all organisms ultimately get their energy from A.) The sun. B.) Inorganic matter. C.) Organic matter. D.) Water. E.) Nutrient cycling.

The sun.

Which of the following factors does NOT influence the activity of an enzyme? A.) The temperature of the cell cytoplasm B.) The time of day. C.) The pH of the cytoplasm. D.) The amount of coenzymes in the cell.

The time of day.

What is Gene Therapy?

The transplantation of normal genes into cells in place of missing or defective ones in order to correct genetic disorders. Possible to cure a genetic disease, one that comes from a person having a faulty allele of a gene, by inserting a correct version of the gene into their cells. The two types of gene therapy are Ex vivo and In vivo.

Diploid cells have homologous pairs of chromosomes. Which of the following is an example of a homologous pair of chromosomes? A.) The two chromosome 1's you have in each of your body cells B.) The chromosome 1 and chromosome 22 you have in each of your body cells C.) The X and Y chromosome D.) Maternal chromosome 1 after it is has been duplicated and joined together at the centromere E.) Human chromosome 1 and mouse chromosome 1

The two chromosome 1's you have in each of your body cells

Certain alleles of BRCA1 and BRCA2 are associated with what type(s) of cancer? What is the function of the BRCA1 and BRCA2 proteins?

The type of cancer associated with BRCA1 and BRCA2 is breast cancer. These alleles shut down the cell cycle and control expression of genes.

Generally, as a cell increases in size, how does the ratio of surface area to volume change? A.) The surface area increases more than the volume B.) The volume and surface area increase in equal amounts C.) The volume stays the same but the surface area increases D.) The surface area stays the same but the volume increases E.) The volume increases more than the surface area

The volume increases more than the surface area

Which of the following structures is not homologous to the others? A.) The wing of a bird B.) The wing of a bat C.) The fin of a whale D.) The wing of an insect E.) The leg of a cat

The wing of an insect

Which of the following does NOT correctly match a term with an accurate definition as it applies in scientific speech? A.) Theory- a proposed, but untested, explanation for an observed phenomenon B.) Biological literacy- ability to understand, think about, communicate, and integrate biological information as you read and hear about it in your daily lives C.) Scientific method- a process employed in science disciplines for examination and discovery D.) Psuedoscience- scientific-sounding claims that are not supported by methodical scientific studies

Theory- a proposed, but untested, explanation for an observed phenomenon

The method of turning ADP into ATP is called chemiosmosis because ATP production is tied to a hydrogen ion gradient across a membrane. TRUE or FALSE?

True

The method of turning ADP into ATP is called chemiosmosis because ATP production is tied to a hydrogen ion gradient across a membrane. True or False?

True

Why do deciduous leaves turn red and yellow in the fall?

There is less light reflected which makes the pigments change in the leaves. There is also less water and humidy. Deciduous leaves produce sugar in the spring and shut down metabolism in the fall.

The period between meiosis I and meiosis II, called interkinesis, differs from interphase in that... A.) There is no S phase (DNA replication). B.) The cell is metabolically inactive. C.) Synapsis occurs. D.) The cells are diploid.

There is no S phase (DNA replication).

Which of the following is incorrect about saturated fats? A.) They are solid at room temperature. B.) They contain the maximum number of hydrogen atoms. C.) They contain glycerol covalently bonded to 3 fatty acids. D.) They are nonpolar molecules that do not mix with water. E.) They are generally the type of fat found in plants and fish.

They are generally the type of fat found in plants and fish.

All of the following are correct about genetically modified crops EXCEPT A.) They are not allowed to be grown in the U.S. B.) They can be more nutritious than regular crops. C.) Their use can lessen the amounts of chemical pesticides used for raising the crop. D.) Their use can reduce the amount of tillage and soil erosion.

They are not allowed to be grown in the U.S.

Why can't certain solute molecules, like sugars and proteins, pass freely across the lipid bilayer of plasma membranes?

They are too large and/or too polar.

How do miRNAs control gene expression?

They bind to the 3'-UTR of their target mRNAs and repress protein production by destabilizing the mRNA and translation silencing.

What is the fate of the NADH and FADH2 made during glycolysis and the Krebs cycle? A.) They carry electrons to the electron transport chain. B.) They carry electrons to water. C.) They carry electrons to pyruvate. D.) They carry electrons to the Calvin cycle to reduce carbon dioxide. E.) They carry electrons to the cytoplasm.

They carry electrons to the electron transport chain.

Fats and oils are referred to as triglycerides because A.) They have three glycerol molecules. B.) They have three fatty acid molecules. C.) They have three amino acids. D.) They have three phospholipids.

They have three fatty acid molecules.

What would occur if large pores were inserted into the thylakoid membranes making them highly permeable to hydrogen ions?

They lose chemiosmosis which means no ATP would be produced.

How can enzymes speed up chemical reactions? A.) They lower the activation energy needed for the reaction. B.) They increase the pH of the reaction. C.) They lower the temperature of the reaction. D.) They lower the rate of ATP breakdown in the reaction. E.) They increase the amount of substrate in the reaction.

They lower the activation energy needed for the reaction.

Transgenic bacteria have been used to help clean up beaches after a massive oil spill. True or False?

True

Why is it that eukaryotic cells have to undergo the individual steps of mitosis while prokaryotic cells can more simply divide in half by binary fission?

They must preserve the nucleus, separate DNA from the spindle made of microtubulin When there are a number of chromosomes, as there are in most eukaryotic cells, special measures are needed to ensure that each daughter cell has a copy of each and every parental chromosome. Prokaryotes have only one circular chromosome that attaches to the cell membrane. Its replicates are separated when the cell pulls apart and becomes two.

What is the value of the checkpoints in the cell cycle? A.) They prevent cells from advancing in the cell cycle until ready to complete the next stage. B.) They ensure a balanced number of cells throughout the body. C.) They aid in cell-cell communication. D.) They allow cells to be pigmented. E.) They allow the nuclear envelope to be a double membrane.

They prevent cells from advancing in the cell cycle until ready to complete the next stage.

Darwin better understood the mechanism for natural selection after he read a book by ___________ on the rate of growth of populations. A.) Alfred Wallace B.) Charles Lyell C.) Thomas Malthus D.) Jean Lamarck

Thomas Malthus

How is the increase in the numbers of antibiotic resistant bacterial diseases over the last century related to evolution and natural selection?

Those bacteria which have a mutation allowing them to survive will live on to reproduce. This will be passed on to their offspring.

Photosynthetic pigments are embedded into the membrane of flattened sacs within chloroplasts called ________. A.) Matrices. B.) Golgi bodies. C.) Stroma. D.) Cristae. E.) Thylakoids.

Thylakoids.

Epithelium has __________ to prevent the passage of molecules and ions through the space between the plasma membranes of adjacent cells. A.) Plasmodesmata B.) Anchoring junctions C.) Tight junctions D.) Gap junctions E.) Extracellular matrix

Tight junctions

A group of cells in an organism with a common structure and function is referred to as a(n)...

Tissue

Presence of the amino acid tryptophan represses expression of the trp operon because tryptophan binds to the... A.) Trp Repressor protein, inhibiting it from binding to the operator. B.) Trp Repressor protein, activating it to bind to the operator. C.) Trp Activator protein, inhibiting it from binding to the enhancer. D.) Trp Activaor protein, activating it to bind to the enhancer.

Trp Repressor protein, activating it to bind to the operator.

A cell which has irreparably damaged DNA would be eliminated by apoptosis. True or False?

True

A scientist studying two daughter cells resulting from mitosis would expect to find them to be genetically identical. True or False?

True

Homologues (or homologous chromosomes) are corresponding chromosomes with genes in the same order, though alleles may differ between them. True or False?

True

Although high doses of radiation are harmful to cells, how can low levels of radioactive isotopes be used medicinally? A.) As tracers in imaging organs using X-rays and PET scans B.) To reduce obesity and diabetes in teenagers C.) To create drugs that work faster than normal D.) To destroy aging and unwanted cells E.) To prevent ultraviolet damage from the sun

To destroy aging and unwnted cells

Which of these reasons for cell division goes beyond the normal requirements for it? A.) To make new immune cells when the organism is sick B.) To replace old cells C.) To make extra copies of DNA to store for later use D.) To make new cells as the organism grows E.) To repair damaged tissue

To make extra copies of DNA to store for later use

The purpose of meiosis I is... A.) To separate homologous chromosomes. B.) To replicate chromosomes. C.) To create 4 haploid cells. D.) To separate sister chromatids. E.) To repair errors in DNA synthesis.

To separate homologous chromosomes.

At times when a certain gene product is not needed, repressing gene expression at this level offers the greatest energy savings for a cell A.) Transcriptional B.) Post-transcriptional C.) Translational D.) Post-translational E.) All of the above offer equivalent energy savings

Transcriptional

Which molecules regulate the process of transcription by binding genes and turning on their expression? A. Translation factors B. Posttranslational control promoters C. Heterochromatin factors D. Repressible control factors E. Transcriptional factors

Transcriptoinal factors

If a biologist wanted to manually cross-fertilize two pea plants, which of the following steps must be taken? A.) Transfer pollen from one plant to the other B.) Transfer a pea from one plant to the other C.) Transfer the egg from one plant to the other D.) Transfer the stigma from one plant to the other E.) Transfer the ovule from one plant to the other

Transfer pollen from one plant to another

A new technique puts animal eggs and DNA together in an agitator with tiny silicon-carbide needles. After fertilization of these treated eggs, what would be produced? A.) Microinjected animals B.) Therapeutically cloned animals C.) Transgenic animals D.) Reproductively cloned animals E.) Xenotransplanted animals

Transgenic animals

If you were studying the control of gene expression which begins when processed mRNA reaches the cytoplasm and before there is a protein product, you would be studying A.) Transcriptional control B.) Posttranscriptional control C.) Posttranslational control D.) Translation control E.) Inducible control

Translational control

The control of gene expression which occurs in the nucleus and involves mRNA processing is referred to as A.) Posttranscriptional control B.) Transcription activator control C.) Translation control D.) mRNA induction E.) Posttranslational control

Translational control

What process results in the assembly of a protein chain? A.) Replication B.) Translation C.) Translocation D.) Mutation

Translocation

A geneticist is studying the movement of a chromosome segment from one chromosome to another. This scientist would be studying a(n) A.) Duplication. B.) Trisomy. C.) Inversion. D.) Deletion. E.) Translocation.

Translocation.

If an enzyme was denatured, it would no longer bind effectively with its substrate efficiently. True or False?

True

In double-stranded DNA, adenine (A) base pairs with thymine (T) and guanine (G) base pairs with cytosine (C). True or False?

True

The attraction of an atom for the electrons in a covalent bond is called electronegativity. True or False? Which is more electronegative: oxygen or hydrogen?

True Oxygen is more electronegative than hydrogen.

In bacteria, the mRNA codon UGG codes for the amino acid tryptophan. What amino acid does UGG code for in animals?

Tryptophan

In bacteria, the mRNA codon UGG codes for the amino acid tryptophan. What amino acid does the codon UGG code for in animals?

Tryptophan (Trp)

How does the amino acid tryptophan repress expression of the trp operon?

Trytophan functions as a corepressor, which is required for the Trp Repressor protein to bind to its operator.

If a diploid parent cell undergoes mitosis, then afterwards there will be... A.) Two diploid daughter cells B.) Four haploid daughter cells C.) Four diploid daughter cells D.) One haploid daughter cell and one diploid daughter cell E.) Two haploid daughter cells

Two diploid daughter cells

If a haploid parent cell undergoes mitosis, then afterwards there will be... A.) Two diploid daughter cells B.) Four haploid daughter cells C.) Four diploid daughter cells D.) One haploid daughter cell and one diploid daughter cell E.) Two haploid daughter cells

Two haploid cells

Fats composed of fatty acids that have double bonds within the fatty acids and thus have fewer than the maximum number of hydrogen atoms are... A.) Saturated. B.) Unsaturated and liquids at room temperature. C.) Saturated and solid at room temperature. D.) Generally found in animal fats.

Unsaturated and liquids at room temperature

Xenotransplants... A.) Involve the injection of viruses into humans. B.) Are animal or plant clones. C.) Use animal organs for transplants. D.) Involve the replacement of whole sets of chromosomes in humans. E.) Involve the injection of plasmids into humans.

Use animal organs for transplants.

Distilled white vinegar is acidic with a pH of around pH 2-3. Which of the following statements is correct about vinegar? A.) Vinegar is a non-polar molecule. B.) Vinegar has an increased concentration of hydrogen ions (H+) compared with pure water. C.) Vinegar has an increased concentration of hydroxide ions (OH-) compared with pure water. D.) Most bacteria, protozoa, and fungi can grow well in vinegar.

Vinegar has an increased concentration of hydrogen ions (H+) compared with pure water.

The portion of the electromagnetic spectrum that is visible to the human eye and is also the source of energy harvested by plants is known as _________. A.) Gamma rays B.) X-rays C.) UV light D.) Visible light E.) Infrared

Visible light

The portion of the electromagnetic spectrum that is visible to the human eye and is also the source of energy harvested by plants is known as...

Visible light

Genetically engineered "golden" rice contains... A.) Vitamin C. B.) Vitamin A. C.) Nitrogen. D.) Golden bacteria.

Vitamin A.

Most of the atmospheric oxygen occurs as a result of photosynthesis. From which of the following molecules is the oxygen derived? A.) Carbon dioxide B.) Water C.) Glucose D.) Chlorophyll E.) NADH

Water

Which of the following is NOT an organic chemical? A.) Water B.) Glucose C.) Protein D.) Fatty acid E.) DNA

Water

What essential role does water play in photosynthesis?

Water is the source of electrons to chlorophyll.

Water is split and oxygen is released during which process in photosynthesis? A. Electron Transport Chain B. Cyclic Electron Pathway C. Calvin Cycle D. Light Reactions E. ATP synthesis

Water molecule are split to release H+ ions and oxygen.

Why is water split during photosynthesis?

Water molecule are split to release H+ ions and oxygen.

Certain insects, such as a water strider, can walk across the surface of a pond. Which property of water allows this? A.) Water molecules are able to stick to other non-water molecules. B.) Water repels hydrophobic materials. C.) Water molecules are able to stick to other water molecules. D.) Frozen water is less dense than liquid water. E.) Water can dissolve polar and ionic compounds

Water molecules are able to stick to other water molecules.

Which statement regarding the translational regulation of the iron-storing protein ferritin is correct? A.) An iron-responsive element (IRE) in the 5' untranslated region of the ferritin mRNA binds directly to iron leading to translational repression. B.) An iron-responsive element (IRE) in the 5'untranslated region of the ferritin mRNA encodes for an enzyme for the synthesis of iron. C.) When iron is present, the iron-responsive element (IRE)-binding protein binds to an enhancer in the ferritin gene and induced transcription. D.) When iron is present, the iron-responsive element (IRE)-binding protein dissociates from the ferritin mRNA thereby allowing its translation.

When iron is present, the iron-responsive element (IRE)-binding protein dissociates from the ferritin mRNA thereby allowing its translation.

What advantage is it to a cell to have small size?

When the surface area to volume ratio is greater, there will be greater efficiency in metabolism and growth.

Why do eukaryotic cells have to undergo the individual steps of mitosis while prokaryotic cells can merely pull in half by binary fission?

When there are a number of chromosomes, as there are in most eukaryotic cells, special measures are needed to ensure that each daughter cell has a copy of each and every parental chromosome. Prokaryotes have only one circular chromosome that attaches to the cell membrane. Its replicates are separated when the cell pulls apart and becomes two.

Which scientists' work first suggested that DNA had the shape of a helix? A.) Wilkins and Franklin B.) Watson and Crick C.) Hershey and Chase D.) Avery, Macleod, and MCarty E.) Campbell and Wilmut

Wilkins and Franklin

In general, are X-linked recessive disorders more common in males or females? Why is this the case?

X-linked recessive disorders are more common in males. Females have two X's and males have one X. Males cannot mask a recessive trait but will express the "recessive" allele. Females can mask recessive allele on the other X. Therefore, males have one chance and females have two chances to get that X chromosome which carriers the gene.

Can a change/substitution in the primary sequence of a polypeptide (protein) affect its secondary and tertiary structure?

Yes

Is structure important for a protein's function?

Yes

Is it possible for a woman to exhibit an X-linked recessive trait? If it is, how can this occur?

Yes, a woman can have a X-linked recessive trait if both he father has the trait and her mother is homozygous for the trait. Yes, a woman can have a X-linked recessive trait if both he father has the trait and her mother is heterozygous for the trait.

A woman with blood type B has a baby with blood type O. A man she says is the baby's father has blood type A, but he denies having fathered the child. Could he be the father? Explain.

Yes, he can be the father. That is the great thing about blood type O; it is complete with every blood type. That is why it is most wanted in blood banks and the most common used blood type used in blood transfusions. AO is the blood type the father can be that will produce an O child. There is a probability he can be the father.

Combining your knowledge of the regulation of gene expression and biotechnology, would it be possible to produce a transgenic plant (genetically modified organism) in which the transgene is only expressed in the leaves and not in the fruit? Explain why or why not.

Yes, scientists can find the gene's promoter / enhancer region only in the leaves and not in the fruit. This will express the transgene only in the leaves.

Where does your body heat come from?

Your body heat comes from all of your anabolic reactions and your cell metabolism.

When a haploid sperm cell fuses with a haploid egg cell, the new cell that is formed is diploid and is called a(n)...

Zygote

With regard to regulation of the lac operon in response to glucose levels, when in a complex with _______, the CAP protein binds to the CAP site and _________ the expression of the lac operon. A.) Glucose ; switches on B.) Glucose ; switches off C.) Lactose ; switches on D.) cAMP ; switches on E.) cAMP ; switches off

cAMP; switches on

The overall purpose of the Calvin cycle is to...

energy + matter = sugar Convert Carbon Dioxide and water into Glucose Build organic molecules

At what pH range do human enzymes work best at?

pH 7 (stomach enzymes pH 1 or 2, inside the cell (lysosome) acidic pH)

When a person exercises strenuously and their muscle cells are not receiving enough oxygen, the cells may begin to produce _____ via fermentation.

lactic acid

Which molecule carries genetic information from the nucleus to where new proteins are manufactured? A.) tRNA B.) RNA polymerase C.) rRNA D.) mRNA

mRNA

A newly discovered level of gene expression in vertebrates involves the DNA areas that used to be called "junk DNA". Actually these areas can encode... A.) microRNAs for RNA interference. B.) Repressor RNAs for halting transcription. C.) Enhancer RNAs for activating transcription. D.) Hox mRNAs for changing splicing.

microRNAs for RNA interference.

One gene has exons ABCDE. In nerve cells, the mRNA molecules from this gene contain the exons ABC. In muscle cells, the mRNA molecules from this contain BCE. What causes the difference between the mRNA molecules in different tissue? A.) DNA activation B.) DNA processing C.) mRNA posttranscriptional control D.) mRNA activation E.) pre-mRNA splicing

pre-mRNA splicing

Why is it impossible for unexplained supernatural and religious phenoma to be explained by biology?

Cannot be tested in science

Tortoises which eat mostly shrubs have notches in their shell that allow their necks to extend upward, while tortoises which eat ground-level vegetation have no notches. The notch is an example of _____. A.) A selective agent. B.) A species. C.) An adaption. D.) An experimental variable. E.) An acquired characteristic.

An adaption.

A biologist is studying where a species lives and how it interacts with other species, which is a study of what? A.) A geographical niche. B.) An allopatric niche. C.) A sympatric niche. D.) Microevolution. E.) An ecological niche.

An ecological niche.

A biologist finds structures that serve the same function but are not considered similar, nor do they share a recent common ancestor. These are A.) Vestigial structures. B.) Analogous structures. C.) Transitional structures. D.) Directional structures. E.) Homologous structures.

Analogous structures.

Scientists must analyze _____ collected from experiments or observations in order to form conclusions. A.) Data B.) Fonts C.) Genes D.) Hypotheses

Data

Sizes of natural population remain relatively stable over time because _________ limits population numbers. A.) Death B.) Birth C.) Immigration D.) Emigration

Death

Theory

Deemed to be right, not an educated guess A concept that seems to be true or proven to be right

Choose the highest, most inclusive category (or taxon). A.) Order B.) Family C.) Genus D.) Phylum E.) Domain

Domain

Humans are members of which Domain of life? A.) Bacteria B.) Archaea C.) Eukarya D.) All of the above E.) None of the above

Eukarya

The unity and diversity of living organisms on our planet is explained by what biology theory?

Evolution

Why is the study of biology central to the understanding and solving of the world's great environmental problems?

Evolutionary biological techniques can be used to help solve global challenges in agriculture, medicine, and environmental sciences.

In the scientific method, the condition which is deliberately changed is referred to as the... A.) Control group. B.) Experimental variable. C.) Control variable. D.) Hypothesis. E.) Test.

Experimental variable.

Which kingdom contains non-photosynthetic multicellular organisms that digest their food externally and then absorb it? A.) Fungi B.) Plantae C.) Animalia D.) Protista E.) Archaea

Fungi

Which theory helps us understand why various species differ from one another, tells us to what diseases we are predisposed, and enables us to produce drugs for a variety of diseases? A.) Cell B.) Gene C.) Ecosystem D.) Homeostasis

Gene

What does the Gene Theory state?

Genes are hereditary units composed of DNA.

Some of these terms could be used with both natural and artificial selection, but which applies only to artificial selection? A.) Adaption B.) Opportunistic C.) Selective advantage D.) Evolution E.) Goal-directed

Goal-directed

According to the biological species concept, which of the following is true? A.) It relies on traits to define a species. B.) It recognizes that every species has an evolutionary history. C.) It can only be applied to prokaryotes. D.) It can only be applied to asexually reproducing species. E.) It relies primarily on reproductive isolation.

It relies primarily on reproductive isolation.

The purpose of the voyage of the HMS Beagle was to... A.) Discover new routes to the New World. B.) Survey the living creatures on islands only. C.) Map navigational routes around the coasts of South America. D.) Search for gold.

Map navigational routes around the coasts of South America.

When animal genitalia or plant floral structures are incompatible between species it is referred to as... A.) Genetic isolation. B.) Mechanical isolation. C.) Behavioral isolation. D.) Habitat isolation. E.) Temporal isolation.

Mechanical isolation.

The process of using and transforming energy by a living organism is... A.) Response to stimulation. B.) Complexity. C.) Metabolism. D.) Homeostasis.

Metabolism.

Define the terms: microevolution and macroevolution

Microevolution is any allele frequency change within the gene pool of a population. Macroevolution is the evolutionary change leading to origin of new taxonomic groups (speciation). Macroevolution requires the origin of species, also called speciation. Speciation is the splitting of one species into two or more, or a transformation into a new species over time. Speciation is the final result of changes in gene pool allele and genotype frequencies. Speciation results from accumulation of independent microevolutionary changes in different populations of the same species.

The problem with the biological species concept is that it assumes that organisms regularly... A.) Undergo mitosis. B.) Reproduce asexually. C.) Outcross. D.) Inbreed.

Outcross.

Hypothesis

Possible explanation for a natural event Input from various sources is used to formulate a testable statement Must be testable

Isolation mechanism which occur after the formation of a zygote are called... A.) Gamete isolation mechanisms. B.) Temporal isolation mechanisms. C.) Postzygotic isolation mechanisms. D.) Mechanical isolation mechanisms. E.) Prezygotic isolation mechanisms.

Postzygotic isolation mechanisms.

While examining a drop of pond water under the microscope, you find a unicellular organism with a nucleus and structures for photosynthesis. To what kingdom would you assign this organism? A.) Animals B.) Plants C.) Fungi D.) Protists E.) Archaea

Protists

Which of these groups does not fit well into the biological species concept? (select all that apply) A.) Animals B.) Plants C.) Protists D.) Bacteria E.) Archaea

Protists, Bacteria, and Archaea

Do all organisms fit equally well under the biological species concept? Explain.

Since the biological species concept defines species based on their ability to interbreed to generate viable, fertile offspring, those organisms that reproduce asexually and those that are extinct cannot be easily characterized by this definition. For example, protists, archaea, and bacteria all produce asexually and do not fit equally well under the biological species concept. The biological species concept cannot be applied to fossils.

Which of the following statements about fossils is least accurate? A.) Fossils are mostly from the hard parts of organisms. B.) Fossils are found in sedimentary rock. C.) Soft body parts are often found along with the hard parts of the fossils. D.) In general, the older a fossil is, the deeper it is buried. E.) The fossil record indicates that life has generally progressed from simple to more complex.

Soft body parts are often found along with the hard parts of the fossils.

A study of the concept of one species splitting into two or more new species would concern what? A.) Polyploidy B.) Speciation C.) Prezygotic isolation D.) Postzygotic isolation E.) Alloploidy

Speciation

A ________ is a separate group of organisms incapable of interbreeding with other such groups.

Species

Choose the lowest, least inclusive category (or taxon). A.) Order B.) Family C.) Genus D.) Phylum E.) Species

Species

The scientific name for a gray squirrel is Sciurus carolinensis. This name represents the animal's taxonomic genus and ______.

Species

When a plant bends toward the light, it is responding to a stimulus. True or False?

True

Structures which are fully developed in one group of organisms but reduced or possibly nonfunctional in a similar group are referred to as... A.) Vestigial structures. B.) Analogous structures. C.) Transitional structures. D.) Homologous structures.

Vestigial structures.

A biologist wants to test the effectiveness of a new food additive on causing growth in mice. An effective control group would be one that... A.) Ate a higher concentration of food additive. B.) Was kept in different conditions across the city. C.) Was fed the same ration without the food additive. D.) Ate a lower concentration of the food additive.

Was fed the same ration without the food additive.

Which of these is an example of homeostasis? A.) Birds, bats, and butterflies all have wings. B.) Venomous coral snakes and harmless scarlet king snakes both have bands of red, yellow, and black. C.) When a dog feels hot, it hangs it tongue out and pants. D.) Hummingbirds prefer to feed on nectar from red flowers. E.) A shark can detect even minute quantities of blood in the water.

When a dog feels hot, it hangs it tongue out and pants.

Why are donkeys and horses considered different species?

When a horse and donkey mate, they produce hybrid offspring called mules. Although they are perfectly healthy, mules are infertile and cannot have their own offspring.


Kaugnay na mga set ng pag-aaral

Unit 4 Final Exam (entrepreneurship)

View Set

Domain 9: Networking Fundamentals

View Set

AP US History II: Chapter 29 + 30 (Jaden Brescia)

View Set

Live Virtual Machine Lab 5.4: Module 05 Troubleshooting Cable Connectivity

View Set

World Geography - Unit 3- Latin America

View Set

Varacalis Mental Health Chapter 8

View Set